Sunteți pe pagina 1din 175

[Type text]

INTRODUCTION
Kishore Vaigyanik Protsahan Yojana (KVPY) is a scholarship program funded by the Department of Science and
Technology of the Government of India, aimed at encouraging students to take up research careers in the areas of
basic sciences, engineering and medicine. It offers scholarship and contingency grants up to the pre-Ph.D. level to
selected students. Started in 1999, it is administered by the Indian Institute of Science.The Kishore Vaigyanik
Protsahan Yojana (KVPY) is an on-going National Program of Fellowships in Basic Sciences, initiated and funded
by the Department of Science and Technology, Government of India, to attract exceptionally talented and highly
motivated students for pursuing basic science courses and research career in science.
The selection of students is made from those studying in XI standard to 1st year in any undergraduate Program in
BasicSciencesnamelyB.Sc./B.S./B.Stat./B.Math./Int.M.Sc./M.S.in Mathematics, Physics, Chemistry and Biology ha
ving aptitude for scientific research. Special groups / committees are set up at IISc, to screen the applications and
conduct an aptitude test at various centres in the country. Based on the performance in the aptitude test, short-listed
students are called for an interview which is the final stage of the selection procedure. For receiving a fellowship,
both aptitude test and interview marks are considered.
The merit list is based on 75% weight-age for Aptitude Test marks and 25% weight-age for the interview marks in
the streams SA, SB and SX. SA can be written by students studying in class 11. SX can be written by students
studying in class 12
The students who qualify KVPY Exam are eligible for the fellowship. The students receive fellowship from 1st year
onwards of their graduation. The KVPY fellows receive Rs. 5000 monthly fellowship from 1st to 3rd year of B.Sc/
B.Stat/ B.S./ B.Maths/ Integrated M.S./M.Sc and an annual contingency grant of Rs.20,000. After three years the
fellowship is enhanced to Rs.7,000 per month during M.Sc/ 4th & 5th year of Integrated M.S./M.Sc and an annual
contingency grant of Rs. 28,000.

CHEMISTRY

1. STOICHIOMETRY

2. ATOMIC STRUCTURE

3. PERIODIC PROPERTIES

4. CHEMICAL BONDING

5. THERMODYNAMICS AND THERMOCHEMISTRY

6. GASEOUS STATE

7. CHEMICAL EQUILIBRIUM
[Type text]

8. IONIC EQUILIBRIUM

9. S BLOCK and P BLOCK

10. GENERAL ORGANIC CHEMISTRY

11. HYDROCARBONS

12. ALKYL AND ARYL HALIDES

13. ALCOHOL ETHERS AND PHENOLS

14. ALDEHYDES AND KETONES

15. CARBOXYLIC ACID AND ITS DERIVATIVES

16. AMINES

17. SOLID STATE

18. LIQUID SOLUTIONS

19. SURFACE CHEMISTRY AND POC

20. CHEMICAL KINETICS

21. ELECTROCHEMISTRY

22. QUALITATIVE ANALYSIS

23. transition elements and COORDINATION CHEMISTRY


[Type text]

STOICHIOMETRY

The Mole

In 1971, the General conference on weights and measures, adopted mole as the seventh basic SI
unit of the amount of a substance.

The term ‘mole’ is a Latin word which means heap or pile. A mole of atom is a collection of
atoms whose total weight is equal to the gram atomic weight. As equal number of moles of
different elements contain equal number of atoms, it is very easy to express the amount of a
substance in terms of moles. Just as a dozen means twelve objects, in the similar fashion, a mole
is a collection of a ‘definite number’ of particles, viz., atoms, molecules, ions or electrons. This
‘definite number’ is called Avogadro number which is equal to 6.023 ? 1023. Thus, a mole can be
defined as, “the amount of a substance containing as many atoms, molecules, ions, electrons or
other elementary entities as there are carbon atoms in exactly 12g of C-12”.

Now-a-days, gram-molecules and gram-atom are termed as a mole of molecules and a mole of
atoms respectively, e.g., 1 gram-molecule of chlorine and 1 gram atom of chlorine are
expressed as 1 mole of Cl2 and 1 mole of Cl respectively.

The number of moles can be calculated by a number of formulae depending upon the data given:
[Type text]

Wt. of substance in g
No. of moles of molecules =
Mol. wt. of substance
Wt. of element in g
No. of moles of atoms =
At. wt. of the element
Volume of the gas at STP
No. of moles of gas =
Standard molar volume (i.e. 22.4L)

The Limiting Reagent


The reactions in which more than one reactants are reacting and if they are not present in the
same molar ratio as the balanced equation requires, then one has to find out the limiting reagent
i.e., the reagent which is completely consumed in the reaction. All quantitative calculations are
carried out with the help of limiting reagent only. Now, how to decide the limiting reagent of a
reaction? Consider the following example.

Suppose you are given 5 moles of PbS and 18 moles of H2O2 and you have to calculate the
maximum amount of PbSO4 being produced.

The balanced equation is:

PbS + 4H2O2?? PbSO4 + 4H2O


Initial moles 5 mol 18 mol

Now since 1 mol of PbS reacts completely with 4 moles of H2O2 to produce 1 mol of PbSO4.
Therefore, 5 moles of PbS will react with 20 moles of H2O2. But since moles of H2O2 is only 18.
So, H2O2 will be the reagent which will be consumed first, and hence H2O2 is the limiting
reagent. If PbS have been the limiting reagent 5 mole of it would have given 5 moles of PbSO4
but it is not so. As one can see from the balanced chemical equation that 1 mol of PbSO4 is
produced from 4 moles of H2O2. So 18 moles of H2O2 will produce 4.5 moles of PbSO4.
So, one can define limiting reagent in another way,

“The reagent producing the least number of moles of products is the limiting reagent”.

Gravimetric Analysis
Gravimetric analysis is an analytical technique based on the measurements of mass of solid
substances and/or volume of gaseous species. Gravimetric analysis is divided into three parts.

i) Mass-Mass relationship
ii) Mass-volume relationship
iii) Volume-volume relationship
[Type text]

i) Mass-Mass Relationship:It relates the mass of a species (reactant or product) with


the mass of another species (reactants or product)

Let us consider a chemical reaction,



2NaHCO3(s) → Na2CO3(s) + H2O + CO2(g)

Suppose the mass of NaHCO3 being heated is ‘a’ g and we want to calculate the weight
of Na2CO3 being produced by heating of ‘a’ g of NaHCO3.

a
The moles of NaHCO3 =
84

According to the above balanced equation 2 moles of NaHCO3 upon heating gives 1 mole
of Na2CO3. So,
1 a
The no. of moles of Na2CO3 produced = ×
2 84
Thus, wt. of Na2CO3 produced = moles of Na2CO3? Molecular weight of Na2CO3
1 a 53a
= × × 106 = gms
2 84 84

ii) Mass-Volume Relationship:It relates the mass of a species (reactant or product)


and the volume of a gaseous species (reactant or product) involved in a chemical
reaction. Suppose we are provided with ‘a’ gms of NaHCO3 in a vessel of capacity V L
and the vessel is heated, so that NaHCO3 decomposes as

2NaHCO3 → Na2CO3 + H2O + CO2

Now, we want to calculate the volume of CO2 gas being produced.


a
Moles of NaHCO3 taken =
84
Now, since 2 moles of NaHCO3 gives 1 mole of CO2 at STP. Thus
1 a
Moles of CO2 produced = ×
2 84
As we know that 1 mole of any gas at STP occupies a volume of 22.4 L.
1 a 
So, volume of CO2 produced =  × × 22.4 L
 2 84 

iii) Volume-Volume Relationship: It relates the volume of gaseous species (reactants


or products) with the volume of another gaseous species (reactant or product) involved in
a chemical reaction.
after the reaction.

Volumetric Analysis
[Type text]

Titration:The process of determination of concentration of a solution of unknown


concentration with the help of a solution of known concentration i.e., standard solution, is known
as TITRATION.

Before approaching towards the depth of volumetric analysis one must know the following terms
very clearly.

Molarity (M):The no. of moles of solute present in one litre of solution is called the
molarity (M).

No. of moles of solute


Molarity =
Volume of solution in litre
Wt. of solute/Molecular wt. of solute
or Molartiy =
Volume of solution in litre
So, no. of moles of solute = Volume of solution (in litre) ? Molarity of solution

Molality (m):No. of moles of solute present in one kilogram of solvent is know as molality

No. of moles of solute


Molality (m) =
Wt. of solvent (in kg)

Normality (N):No of equivalents of solute present in one litre of the solution is known as
Normality (N).

No. of equivalents of solute


Normality (N) =
Volume of solution in litre
Wt. of solute/equivalent weight of solute
=
Volume of solution in litre

No. of equivalent of solute = Volume of solution (in litre) ? normality of solution

No. of milli equivalents of solute = Normality of solution ? Volume of solution in


millilitres

Equivalent Weight:Number of parts by mass of an element which reacts or displaces from a


compound 1.008 parts by mass of hydrogen, 8 parts by mass of oxygen and 35.5 parts by mass of
chlorine, is known as the equivalent weight of that element e.g.

2Mg + O 2 
→ 2MgO
48g 32g
12g 8g
∴ 32g of O2 reacts with 48g of Mg
48 × 8
∴ 8g of O2 = = 12g
32
[Type text]

∴ Equivalent weight of Mg = 12
Similarly, Zn + H 2 SO 4 → ZnSO 4 + H 2
65.5g 2 × 1.008
Q 2 × 1.008 g of hydrogen is displaced by 65.5g
65.5
∴ 1.008g of H = = 32.75g
2 ×1.008
65.5
∴ Equivalent weight of Zn = = 32.75
2
3
Al + Cl 2  → AlCl 3
2
3
32g × 71g Q 111.5g chlorine reacts with 27g of Al
2
27 × 35.5
∴ 35.5 g chlorine reacts with = 9.0g of Al
111.5
27
∴ Equivalent weight of aluminium = = 9.0
3

As we can see from the above examples that equivalent weight is the ratio of atomic weight and
a factor (say n-factor) which is in above three cases is their respective valencies. So, equivalent
atomic weight
weight =
n − factor

In a similar way, the equivalent weight of acid/base is the ratio of molecular weight and the
basicity/acidity and for oxidizing agents and reducing agent it is the ratio of molecular weight
and the number of moles of electrons gained or lost. So in case of acid/base the n-factor is
basicity/acidity (i.e. number of dissociable H+ ions/number of dissociable OH– ion and in case of
oxidizing agent/reducing agent, n-factor is number of moles of electrons gained/lost per mole of
oxidizing agent/reducing agent. Therefore, in general, we can write.
Atomic or molecualr weight
Equivalent weight (E) =
n − factor
Wt W W
No. of equivalents of solute = = =
Eq. wt. E M / n
No. of equivalents of solute = No. of moles of solute ? n-factor

And also Normality = n-factor ? molarity of solution

Calculation of n-factor
For calculating the n-factor of any reactant in any reaction. One must know the kind of reaction it
is, the reactions may be classified into the following three types.

1. Acid-Base Reactions/Neutralization reactions


2. Redox Reactions
[Type text]

3. Precipitation Reactions/Double decomposition reactions

Acid-Base Reaction:According, to the Arrhenius, an acid is a substance that furnishes


H+ ion(s) in solution, a base is a substance that furnishes OH– ion(s) in solution and
neutralization is a reaction in which H+ ion furnished by acid combines with OH– ions furnished
by base. The number of H+ ion(s) furnished per molecule of the acid is its n-factor also called
basicity. Similarly the number of OH– ion(s) furnished by the base per molecule is its n-factor,
also called acidity.

Some Examples

→ H + + Cl −
HCl 
( n =1)

→ H + + HSO −4
H 2SO 4 
( n =1)

→ 2H + + SO 24−
H 2SO 4 
( n =2 )

→ H + + H 2 PO −4
H 3 PO 4 
( n =1)

→ 3H + + PO 34−
H 3 PO 4 
( n = 3)

→ H + + H 2 PO 3−
H 3 PO 3 
( n =1)

→ 2H + + HPO3−
H 3PO3 
( n =2)

The n-factor of H3PO3 cannot be 3 as it has only two dissociable O


H+ ions. So, its n-factor or dissociable protons is 1 or 2 as one of the HO P OH
H-atoms is linked with P atom directly.
H
− +
Similarly, CH 3COOH 
→ CH 3COO + H
( n =1)

n-factor of CH3COOH is 1, because it contains only one dissociable H+ ion.

Now, we will consider the n-factor of some bases.


→ Na + + OH −
NaOH 
( n =1)

→[Ba(OH)]− + OH −
Ba(OH) 2 
( n =1)

→ Ba 2+ + 2OH −
Ba(OH) 2 
( n =2 )

→ Al3+ + 3OH −
Al(OH) 3 
( n =3)

Similarly, n-factor of Al(OH)3 can also be 1 or 2 or 3, depending upon the number of OH–
released.
[Type text]

Redox Reactions
Those reactions which involve the exchange of electrons are called redox reactions. For the
calculation of n-factor of oxidising agent or reducing agent, the method depends upon the change
in oxidation state of the species considered. We will discuss them one by one.
i) When only one atom undergoing either reduction or oxidation e.g.
+7 +2
+
MnO −4 →
H
Mn 2+
n =5
In such a case, we consider the change in oxidation state of atom undergoing oxidation or
reduction change per molecule as the n-factor of the species.
n-factor = |(+2) ?1 – (+7) × 1| = 5
+2 +3
Fe 2+ 
→ Fe3+ n-factor = |(+3) × 1 – (+2) × 1| = 1
n =1
+3×2 +4×2
−2
C O 
2 4→ 2CO 2 n-factor = |(+4) × 2 – (+3) × 2| = 2
n =2
+6×2 +3× 2
2−
Cr O2 7 → 2Cr 3+
 n-factor = |(+3) × 2 – (+6)× 2| = 6
n =6

ii) Salts which reacts in such a way that only one atom undergoes change in oxidation state
but appears in two products with the same oxidation state:

In such a case the method of calculation of n-factor remains the same i.e., we will
calculate the change in oxidation state of the atom per mole of that substance (reactant).
+6×2 +3 3+
Cr2 O72− 
→ Cr 3+ + Cr 3+

In this example, oxidation state of Cr changes from +6 to +3 in both the products. So


n-factor = |(+6) × 2 – (+3) × 2| = 6

iii) Salts which react in such a way that only one atom undergoes change in oxidation state
but goes in two products with different oxidation state as a result of either only oxidation
or only reduction.
+7 +2 +6
3MnO −4 
→ 2 Mn 2 + + Mn + 6

In such a case, it is impossible to calculate the n-factor until and unless one knows that
how much of MnO −4 is changing to Mn2+ and how much to Mn6+ and if one knows the
balanced equation then there is no need of calculation of n-factor. Nevertheless in such
case the n-factor can be calculated by deducing the total change in oxidation state divided
by total number of atom undergoing reduction/oxidation change. So, for the calculation
of n-factor in the above example, out of three moles of MnO −4 , two moles are being
[Type text]

converted to Mn2+ and one mole changes to Mn6+. So total decrease in oxidation state
of Mn.

= | [2 × (+2) – 2 × (+7)] | + | [1 × (+6) – 1 × (+7)]|


= | 4 – 14 | + | 6 – 7 | = 11
11
So, n-factor =
3

iv) Salts which react in such a way that only one atom undergoes change in oxidation state
in two product, in one product with changed oxidation state and in other product with
same oxidation state as that of reactant.

In such case also one cannot calculate the n-factor without knowing the balanced
chemical equation because one must know how much of atom has changed its oxidation
state. For example.

K2Cr2O7 + 14HCl ?? 2KCl + 2CrCl3 + 3Cl2 + 7H2O

Let us calculate the n-factor of HCl. Out of 14 moles of Cl– (in HCl) only 6 moles of Cl–
are changing its oxidation state from –1 to 0 in the product Cl2 and the oxidation state of
remaining 8 Cl– ions remains same in KCl and CrCl3. So, total no. of moles of electrons
lost by 14 moles of HCl is 6. So each mole of HCl takes up 6/14 i.e., 3/7 moles of
electrons and hence n-factor of HCl is 3/7.

v) Salts which react in such a way that two or more atoms in the salt undergoes change in
oxidation states as a result of either oxidation or reduction. Let us consider the following
example,

FeC2O4?? Fe3+ + 2CO2

In this case, the oxidation of both Fe2+ and C3+ are changing from + 2 and +3 to +3 and
+4 respectively. In such a case we will calculate the n-factor of the salt as the total
increase or decrease in oxidation state per mole of the salt. As one can see that one mole
of FeC2O4 contains one mole of Fe2+ and one mole of C2O42– (i.e. 2 carbon atoms per
mole of C2O42–.

Total change in oxidation state

= | 1 × (+2) – 1 × (+3) | + | 2 × (+3) – 2 × (+4) |


=1+2=3
So, n-factor of FeC2O4 is 3

vi) Salts which react in such a way that two atoms undergoing change in oxidation state but
one undergoing oxidation and other reduction reaction. In such a case one has to
calculate the change in oxidation state of either the atom being oxidized or the atom
being reduced. For example.
[Type text]

−3× 2 +6× 2 0× 2 +3× 2


(N H 4 ) 2 Cr 2 O 7 
→ N 2 + Cr2 O3 + 4H 2 O

In this reaction, the oxidation state of N is increasing by 6 units and that of Cr is


decreasing by 6 unit. So, we can consider either oxidation or reduction product for the
calculation of n-factor and it will be the same.

n-factor of (NH4)2Cr2O7 considering oxidation


= |(-3) ×2 – (0) × 2| = 6

n-factor of (NH4)2Cr2O7 considering reduction


= |(+6) × 2 – (+3) × 2| = 6

vii) Species which undergoes disproportionation reaction: Those reaction in which oxidant
and reductant are the same species or the same element from the species is getting
oxidized as well as reduced.

When the number of moles of atoms being oxidized is equal to the number of moles of
atoms being reduced.

The n-factor can be calculated by knowing the balanced chemical equation and
considering any of the change taking place. Say for example,
2H2O2?? 2H2O + O2

Out of 2 moles of H2O2 consumed in the reaction, one mole of H2O2 is being oxidized
(H2O2?? O2) and one mole of H2O2 is being reduced (H2O2?? 2H2O). First consider the
oxidation reaction
( −1)× 2 2×0
H2O2 
→ O 2
n-factor = |2 × 0 – (–1) × 2| = 2

Again, considering reduction reaction


( −1)×2 ( −2 ) × 2
H2O2 
→ 2H 2 O
n-factor = |(-2) × 2 – (-1) × 2| = 2

So, n-factor of H2O2 either considering oxidation or reduction reaction is same


i.e. 2.

Precipitation/Double Decomposition Reactions: In such reaction, there is no


change in oxidation state of any atom. The n-factor of the salt can be calculated by multiplying
the oxidation state of the cation/anion by total no. of atoms per molecule of the salt. For example
[Type text]

BaCl 2 + Na 2SO 4 
→ BaSO 4 ↓ +2 NaCl
n=2 n=2
for BaCl2

n-factor = Oxidation state of Ba atom in BaCl2 × number of Ba atoms in 1 molecule of


BaCl2
= (+2) × 1 = 2

for Na2SO4
n-factor = Oxidation state of Na × number of Na-atoms in 1 molecule of Na2SO4
= (+1) × 2 = 2

Titration
The process of determination of concentration of a solution with the help of a solution of known
concentration (standard solution) is called titration.

Titration is divided into following four categories.

i) Simple Titration
ii) Double Titration
iii) Back Titration
iv) Iodimetric and Iodometric Titration

Simple Titration: A known volume of the solution of unknown concentration is taken in a


flask and required reagents are added to it. The solution of known concentration is added from
the burette in the solution of unknown concentration till the latter reacts completely. This process
is called titration. At the end point (equivalence point) the equivalents or milliequivalents of the
two reacting substances are equal.
Volume of solution (A) = VA litres
Normality of solution (A) = NA
Equivalents of substance (A) = NAVA

Similarly, equivalents of substance (B) = NBVB

At the equivalence point (end point) the equivalents (not the moles) of the two substance are
equal.
NAVA (litre) = NB × VB (litre)

Double Titration: If an aqueous solution contains a mixture of any two of the three NaOH,
NaHCO3 and Na2CO3 and it has to be titrated against an acid HCl or H2SO4, it will require two
indicators to determine the strength of the bases present. The two indicators used are
phenophthalein and methyl orange.
[Type text]

Method:A given volume of the aqueous solution of the bases is taken and phenophthalein
indicator is added to it. This solution is titrated with an acid of known normality to the end point
the volume of the acid used is noted. This end point is called first end point. Now in the same
solution methyl orange is added and gain titrated with an acid of known strength to the end point.
It is called second end point. The volume of acid, used in the second end point is also noted. The
normality of the bases present is then calculated.

Principle: In the presence of phenolphthalein indicator

i) NaOH is completely neutralized by the acid.

ii) Only half of the milliequivalents of Na2CO3 present are titrated as the phenolphthalein
will show the colour change when only NaHCO3 (weak base) and neutral species are left
in the reaction mixture. The following reactions take place,
NaOH + HCl → NaCl + H2O
Na2CO3 + HCl 
→ NaHCO 3 + NaCl
weak base

Since phenolphthalein is a weak organic acid, and it changes its colour in weakly basic medium
(NaHCO3), so as soon as the Na2CO3 is converted to NaHCO3 phenophthalein shows the colour
change indicating the completion of the reaction.

In the presence of methyl orange, all the basic substances left in the mixture will be neutralized
by acid and methyl orange will show the colour change when the medium is weakly acidic
(H2O + CO2 i.e. H2CO3).

1. Titration of the solution containing both NaOH and Na2CO3: A given


volume of the aqueous solution is titrated with an acid of known normality using
phenophthalein indicator. Suppose ‘a’ milli equivalents of acid are used in the first end
point then,

milli equivalent of NaOH + ½ milli equivalent of Na2CO3


= milli equivalent of acid = a …(1)

Now in the same already titrated solution methyl orange indicator is added and again
titrated to the end point. Suppose ‘b’ milli equivalents of the acid are used at the second
end point.

½ milli equivalents of Na2CO3 = milli equivalents of acid = b …(2)

From equation (1) and (2)

Milli equivalents of acid used by Na2CO3 = 2b


≡ milli equivalents of Na2CO3

Milli equivalents of acid used by NaOH =a–b


= milli equivalent of NaOH
[Type text]

Knowing the milli equivalents of Na2CO3 or NaOH and the volume of the solution
titrated, their normality can be calculated.

2. Titration of the solution containing both Na2CO3 and NaHCO3:Given


volume of the solution is titrated by an acid using phenolphthalein indicator. Suppose ‘a’
milli equivalents of acid are used in the first end point. Then

1/2 milli equivalent of Na2CO3 = milli equivalents of acid = a …(1)

Now in the same already titrated solution methyl orange indicator is added and again
titrated to the end point. Suppose ‘b’ milli equivalents of the acid are used at the second
end point.Then

1/2 milli equivalents of Na2CO3 + milli equivalents of NaHCO3


= milli equivalents of acid=b ..(2)
From equation (1) and (2)

Milli equivalents of acid used by Na2CO3 = 2a


≡ milli equivalents of Na2CO3
Milli equivalents of acid used by NaHCO3 =b–a
= milli equivalent of NaHCO3
Knowing the milli equivalents of the base and volume of the solution titrated, the
normality (strength) of the bases can be calculated.

Back Titration: Let us consider that we have an impure solid substance ‘Z’ weighing ‘w’ g
and we are required to calculate the percentage purity of ‘Z’ in the sample. We are also provided
with two solutions ‘X’ and ‘Y’, where the concentration of ‘Y’ is known (N1) and that of ‘X’ is
unknown. For the back titration to work, following conditions are to be satisfied
a) Compounds ‘X’, ‘Y’ and ‘Z’ should be such that ‘X’ and ‘Y’ reacts with each other.
b) ‘X’ and pure ‘Z’ also reacts with each other but the impurity present in ‘Z’ does not react
with ‘X’.
Z + X (excess) ?? Product 1
Remaining (X) + Y ?? Product 2
Note: Product 1 should not react with Y
Milli equivalent of Y = N2V2
Where N2 and V2 (ml) is the normality and volume of Y
Initial mili equivalent of X = N1V1
Where N1 and V1 (ml) is the normality and volume of X
Remaining milli equivalents of X after reacting with Y = N1V1– N2V2
Remaining milli equivalents of X = milli equivalents of Z
a × 1000
N1V1 – N2V2 =
Equivalent weight
[Type text]

Where ‘a’ is the weight of pure Z which is reacted.


Molecular weight (N1V1 − N 2 V2 )
a=
n − factor

(N1V1 − N1V2 ) Molar Mass of `Z'


? Percentage purity of ‘Z’ = × ×100
n − factor W

Iodimetric and Iodometric Titrations


The redox-titration using iodine directly or indirectly as an oxidizing agent are called Iodine
Titrations. These are of two types.

1. Iodimetric Titrations:Iodimetric titrations are defined as those iodine titrations in


which a standard iodine solution is used as an oxidant and iodine is directly titrated
against a reducing agent. Iodimetric procedures are used for the determination of strength
of reducing agent like thiosulphates, sulphites, arsenites and stannous chloride etc., by
titrating them against standard solution of iodine taken in a burette. Some cases of
oxidation-reduction reactions are given as under:
i) 2Na2S2O3 + I2 → Na2S4O6 + 2NaI
ii) Na2SO3 + I2 + H2O 
→ Na2SO4 + 2HI
iii) Na3AsO3 + H2O → Na3AsO4 + 2HI

2. Iodometric Titrations:Iodometric titrations are defined as those iodine titrations in


which some oxidizing agent liberates iodine from an iodine solution and then lilberated
iodine is titrated with a standard solution of a reducing agent added from a Burette. In
such titrations, a neutral or an acidic solution of oxidizing agent is employed. The amount
of iodine liberated from an iodide, (i.e. KI) is equivalent to the quantity of the oxidizing
agent present. Iodometric titrations are used for the determination of CuSO4, K2Cr2O7,
KMnO4, ferric ions, antimonite ions, H2O2, MnO2, bromine and chlorine etc. The
equations for some of the reactions are as follows:

i) 2CuSO4 +4KI 
→ Cu2I2+ 2K2SO4 + I2
ii) 2KMnO4 + 3H2SO4  → K2SO4 + 2MnSO4 + 3H2O + 5O
10KI + 5H2SO4 + 5O 
→ 5K2SO4 + 5H2O + 5I2
iii) K2Cr2O7 + 4H2SO4 
→ K2SO4 + Cr2(SO4)3 + 4H2O + 3O
6KI + 3H2SO4 + 3O → 3K2SO4 + 3H2O + 3I2

In the above reactions, the liberated iodine is titrated with a standard sodium thiosulphate.

2Na2S2O3 + I2 
→ Na2S4O6 + 2NaI

Volume Strength of H2o2 Solution


[Type text]

The concentration of H2O2 is usually represented in terms of volume. If a sample of H2O2 is


labeled as ‘x volume’, it means that 1 volume of H2O2 solution gives ‘x volumes’ of O2 gas at
STP on complete decomposition.

Consider the decomposition of H2O2 as



2H 2 O 2 
→ 2H 2 O + O 2
2×34 g 22.4 L at STP

22400 ml of O2 gas is liberated by 68g of H2O2 solution


68x 17 x
? x ml of O2 gas will be liberated by = = g of H 2 O 2
22400 5600
17 x
It means that g of H2O2 will be present in 1 ml of solution.
5600
17 x 17 x
? 1000 ml of solution contains H2O2 = × 1000 =
5600 5. 6

Strength (g L–1) = Normality ? Equivalent weight


17 x 34
= N× (Q n-factor of H2O2 = 2)
5.6 2

x = 5.6 ? N

i.e., Volume strength of H 2 O 2 = 5.6 × Normality

Percentage Labeling of Oleum


Oleum or fuming sulphuric acid contains SO3 gas dissolved in sulphuric acid. When water is
added to oleum, SO3 reacts with H2O to form H2SO4, thus mass of the solution increases.

SO3 + H2O?? H2SO4

The total mass of H2SO4 obtained by diluting 100g of sample of oleum with desired amount of
water, is equal to the percentage labeling of oleum.

?% labeling of oleum = Total mass of H2SO4 present in oleum after dilution.

= mass of H2SO4 initially present + mass of H2SO4


produced on dilution.
NaOH solution for neutralization. Calculate the mass % of SO3 in oleum.

Hardness of Water
Hardness of water is due to the presence of Ca++ and Mg++ in it. Hardness of water is of two
types:
[Type text]

1. Temporary Hardness:It is due to the presence of dissolved Ca(HCO3)2 and


Mg(HCO3)2 in water. To determine the temporary hardness a given volume of hard water
is titrated with a strong acid (HCl or H2SO4) using methyl orange indicator.

2. Permanent Hardness:It is due to the presence of dissolved CaCl2, CaSO4, MgCl2


and MgSO4 in water. A known volume of hard water is taken and an excess of known
equivalents of Na2CO3 are added in it. Na2CO3 reacts with Ca++ and Mg++ forming
precipitates of CaCO3 and MgCO3. These precipitates are filtered off. The filtrate is
titrated with a strong acid (HCl or H2SO4). Knowing the equivalents of Na2CO3 added
and left unreacted, the equivalents of Na2CO3 consumed by hard water is known. The
equivalents of Na2CO3 consumed is equal to the total equivalents of Ca++ and Mg++ ions
present in hard water.

Hardness of water is represented in ppm (mg/litre) of CaCO3 i.e. milli grams of CaCO3
present per litre of hard water. But hard water does not contain CaCO3. Hard water
contains CaCl2, MgCl2, Ca(HCO3)2 etc.
One mole CaCl2? one mole CaCO3

or 111g CaCl2? 100g CaCO3

Similarly, 120g MgSO4? 100g CaCO3

Thus mass of CaCO3 corresponding to the mass of CaCl2, MgSO4 etc., present in hard water is
calculated. Milligrams of CaCO3 per litre of hard water is called hardness of water in ppm.

Solved Problems

LEVEL I

Problem 1: Calculate the mass of 90% pure MnO2 to produce 35.5g of Cl2 according to
the following reaction.
MnO2 + 4HCl ?? MnCl2 + Cl2 + 2H2O

Solution: MnO2 + 4HCl ?? MnCl2 + Cl2 + 2H2O


87g 71g
Q 71g Cl2 is produced by 87g of MnO2
[Type text]

87 × 35.5
?35.5g Cl2 is produced = = 43.5g
712
Q 90g pure MnO2 is present in 100g sample
100 × 43.5
? 43.5g pure MnO2 = = 48.3g
90

Problem 2: 8 gm of methane is burnt with 4.48L of O2 at STP. Find out the volume of
CO2 gas produced at STP and also the weight of CO2 gas.

Solution: The balanced chemical equation is


CH4 + 2O2?? CO2 + 2H2O
1 mol 2 mol 1 mol
16 gm 2 ? 22.4 L 22.4 L
44 gm
8
No. of moles of CH4 = = 0.5 mol
16
4.48L
No. of moles of O2 = = 0.2mol
22.4L
Now since 1 mole of CH4 requires 2 mol (i.e. 44.8 L) of O2 for complete
combustion. But the given moles of O2 is only 0.2 mol. So, O2 is the limiting
reagent.
Again, since 2 moles of O2 reacts with 1 mol of CH4 to give 22.4 L of CO2
at STP.
So 0.2 mole of O2 will react with 0.1 mol of CH4 to give 2.24 L of CO2.
Wt. of CO2 produced = 0.1 mol ? 44
= 4.4 gms of CO2

Problem 3: By heating 10g of CaCO3, 5.6g CaO is formed. What is the weight of CO2
obtained in this reaction?

Solution: CaCO3?? CaO + CO2


Molecular weight 100 56 44
Q 100g CaCO3 gives 56g CaO and 44g CO2
? 10g CaCO3 gives 5.6g CaO and 4.4g CO2

Problem 4: Calculate the volume of hydrogen liberated at 27°C and 760 mm pressure by
treating 1.2g of magnesium with excess of hydrochloric acid.

Solution: The balanced equation is


Mg + 2HCl = MgCl2 + H2
1 mole 1 mole
[Type text]

24g 22.4 litre at NTP


24g of Mg liberate hydrogen = 22.4 litre
22.4
1.2g of Mg will liberate hydrogen = × 1.2 = 1.12 litre
24
Volume of hydrogen under given condition can be calculated by applying
P1V1 P2 V2
=
T1 T2
P1 = 760 mm P2 = 760 mm
T1 = 273 K T2 = (27 + 273) = 300K
V1 = 1.12 litres V2 = ?
760 × 1.12 300
V2 = × = 1.2308litres
273 760

Illustration 5: What volume of oxygen gas at NTP is necessary for complete combustion of
20 litre of propane measured at 0°C and 760 mm. pressure.

Solution: The balanced equation is


C 3 H 8 + 5O 2 = 3CO 2 + 4H 2 O
1 vol 5 vol
1 litre 5 litre

1 litre of propane requires = 5 litre of oxygen


20 litre of propane will require = 5 ×20 =100 litre of oxygen at 760 mm
pressure and 0°C.

Problem 6: 40 ml N/10 HCl and 60 ml N/20 KOH are mixed together. Calculate the
normality of the acid or base left. What is the normality of the salt formed in
the solution?

1× 40
Solution: Milli equivalents of HCl = N × V (ml) = =4
10
1× 60
Milli equivalents of KOH = N × V (ml) = =3
20
One milli equivalent of an acid neutralizes one milli equivalent of a base
Milli equivalent of HCl left =4–3=1
Total volume of the solution = 40 + 60 = 100 ml
Milli equivalents of HCl = N × V (ml)
1 = N × 100
Normality (N) of HCl left in solution = 0.01
Salt formed = Milli equivalent of acid or base neutralized
Milli equivalents of the salt formed = N × V (ml)
3 = N × 100
Normality (N) of salt formed = 0.03
[Type text]

Problem 7: NaOH and Na2CO3 are dissolved in 200 ml aqueous solution. In the
presence of phenolphthalein indicator. 17.5 ml of 0.1 N HCl are used to
titrate this solution. Now methyl orange is added in the same solution
titrated and it requires 2.5 ml of the same HCl. Calculate the normality of
NaOH and Na2CO3 and their mass present in the solution.

Solution: Milli equivalents (a) of HCl used in the presence of phenolphthalein indicator.
= N ? V (ml)
= 0.1 ? 17.5 = 1.75
1.75 (a) = milli. eq. of NaOH + 1/2 milli eq. of Na2CO3 ...(1)
Milli eq. (b) of HCl used in the presence of methyl orange indicator
= N ? V (ml)
= 0.1 ? 2.5 = 0.25
0.25 (b) = 1/2 milli equivalents of Na2CO3 …(2)
For Na2CO3 solution: From equation(2)
Milli eq. of acid used by Na2CO3 = 2b
= 2 ? 0.25 = 0.5
Volume of Na2CO3 solution = 200 ml
Suppose, Normality of Na2CO3 =N
Milli equivalents of Na2CO3 = N ? V (ml) = 200N
Putting equivalents of acid and Na2CO3 equal.
200N = 0.5
1
or (Normality of Na2CO3 solution) N =
400
Mass of Na2CO3 = N ? E ? V (litre)
1
(E for Na2CO3 = 53) = × 53 × 0.2
400
= 0.0265 gram
For NaOH solution: From equation (1) and (2)
Milli eq. acid used by NaOH = a – b = 1.75 – 0.25
= 1.50
Volume of NaOH solution = 200 ml
Suppose, Normality of NaOH solution = N
Mili eq. of NaOH = N ? V (ml) = 200 N
Putting the mili eq. of NaOH and acid used equal
200 N = 1.5
1.5
(Normality of NaOH solution) N =
200
Mass of NaOH = N ? E ? (V litres)
[Type text]

1.5
= × 40 × 0.2
200
(E for NaOH = 40) = 0.06g

Problem 8: Both Na2CO3 and NaHCO3 are present in an aqueous solution. In the
presence of phenolphthalein indicator 10 ml of this solution requires 2.5 ml
of 0.1M H2SO4 for titration. After this methyl orange is added in the same
solution and titration requires 5 ml M H2SO4. Calculate the concentration
of Na2CO3 and NaHCO3 in g/litre.

Solution: Milli eq. (a) of H2SO4 used in the presence of phenolphthalein indicator
= N ? V (ml) = 0.1 ? 2 ? 2.5 = 0.5
1
a = 0.5 = milli equivalents of Na2CO3 …(1)
2
Milli. eq. (b) of NaHCO3 + ½ milli eq. of Na2CO3 …(2)
For Na2CO3 solution: From equation (1)
Milli eq. of acid used by Na2CO3 = 2 ? 0.5 = 1
Suppose, Normality of Na2CO3 solution = N
Volume of Na2CO3 solution taken = 10 ml
Milli eq. of Na2CO3 taken = N ? V (ml) = 10 N
Putting the milli eq. of H2SO4 and Na2CO3 equal,
1 = 10 N
or (Normality of Na2CO3) N = 0.1
Strength (S) in g/litre =N?E
= 0.1 ? 53 (E for Na2CO3 = 53)
= 5.3 g/litre
For NaHCO3 solution: From equations (1) and (2) milli eq. of acid used by
NaHCO3 = b – a = 1.0 – 0.5 = 0.5
Suppose, Normality of NaHCO3 solution =N
Volume of NaHCO3 solution taken = 10 ml
Milli equivalents of NaHCO3 taken = 10 N
Putting the milli eq. of H2SO4 and NaHCO3 equal,
0.5 = 10 N
or (Normality of NaHCO3 solution) N = 0.05
Strength (S) in g/litre =N?E

(E for NaHCO3 = 84) = 0.05 ? 84 = 4.2g/litre Illustration 9:


20 g of a sample of Ba(OH)2 is dissolved in 10 ml. of 0.5N HCl solution.
The excess of HCl was titrated with 0.2N NaOH. The volume of NaOH used
was 10 cc. Calculate the percentage of Ba(OH)2 in the sample.

Solution: Milli eq. of HCl initially = 10 ? 0.5 = 5


[Type text]

Milli eq. of NaOH consumed = Milli eq. of HCl in excess


= 10 ? 0.2 = 2
? Milli eq. of HCl consumed = Milli eq. of Ba(OH)2
= 5-2=3
? eq. of Ba(OH)2 = 3/1000 = 3 × 10-3
Mass of Ba(OH)2 = 3 × 10-3 × (171/2) = 0.2565 g.
% Ba(OH)2 = (0.2565/20) × 100 = 1.28%

Problem 10: 0.5 g sample containing MnO2 is treated with HCl, liberating Cl2. The Cl2 is
passed into a solution of KI and 30.0 cm3 of 0.1 M Na2S2O3 are required to
titrate the liberated iodine. Calculate the percentage of MnO2 in sample. (At.
Wt. of Mn = 55).

Solution: MnO 2 HCl


→ Cl 2 →
KI
I 2 Na
2S
→ NaI + Na 2 S 4 O 6
2O 3

Redox change are: → 2I −


2e + I 02 
+
2S 22+ 
→ S(45 / 2) + 2e
2 e + Mn 4 + 
→ Mn 2 +
The reactions suggest that,
Meq. of MnO2 = Meq. of Cl2 formed = Meq. of I2 liberated
= Meq. of Na2S2O3 used
w
∴ × 1000 = 0.1 × 1 × 30
M/2
[ Q N Na 2S2 O3 = M Na 2S2 O3 since valency factor = 1, see redox changes for
Na2S2O3]
0.1×1× 30 × M 0.1× 1× 30 × 87
or w MnO2 = = (Q M MnO2 = 87)
2000 2000
w MnO 2 = 0.1305
0.1305
∴ Purity of MnO2 = × 100 = 26.1%
0.5

Problem 11: A sample of H2O2 is x% by mass. x ml of KMnO4 are required to oxidize one
gram of this H2O2 sample. Calculate the normality of KMnO4 solution.

Solution: Suppose, Mass of H2O2 solution = 100g


Mass of H2O2 present = x gram
Mass of H2O2 solution taken = 1 gram
x
Mass of H2O2 present in 1 gram solution =
100
w
Equivalents of H2O2 =
E
[Type text]

x
= …(1)
100 ×17
(E for H2O2) = 17
Equivalents of KMnO4 = N × V (litre) = N × x × 10–3
Putting equivalents of H2O2 and KMnO4 equal,
x
= N × x ×10−3
100 ×17
N = 0.59 (Normality of H2SO4)

Problem 12: Calculate the composition of 109% oleum.


Solution: Let the mass of SO3 in the sample be ‘w’ g, then the mass of H2SO4 would be
(100 – w)g.
On dilution,
SO3 + H2O ?? H2SO4
80g 18g
w
Moles of SO3 in oleum = = Moles of H2SO4 formed after dilution.
80
98w
?Mass of H2SO4 formed on dilution =
80
98w
Total mass of H2SO4 present in oleum after dilution = + (100 − w ) = 109
80
? w = 40
Thus oleum sample contains 40% SO3 and 60% H2SO4.

LEVEL 2
SUBJECTIVE

Problem 1: P and Q are two elements which forms P2Q3 and PQ2. If 0.15 mole of P2Q3
weighs 15.9g and 0.15 mole of PQ2 weighs 9.3g, what are atomic weights of
P and Q?

Solution: Let atomic weight of P and Q are a and b respectively


? Molecular weight of P2Q3 = 2a + 3b
and Molecular weight of PQ2 = a + 2b
Now given that 0.15 mole of P2Q3 weigh 15.9g
15.9  wt. 
(2a + 3b) = ∴ = mole 
0.15  mol. wt. 
9.3
Similarly, (a + 2b) =
0.15
Solving these two equations
[Type text]

b = 18
a = 26

Problem 2: Potassium selenate is isomorphous with potassium sulphate and contains


45.52% selenium by weight. Calculate the atomic weight of selenium. Also
report the equivalent weight of potassium selenate.

Solution: Potassium selenate is isomorphous to K2SO4 and thus its molecular formula is
K2SeO4.
Now molecular weight of K2SeO4 = (39 ? 2 + a + 4 ? 16)
= (142 + a)
where a is atomic weight of Se
(142 + a)g K2SeO4 has Se = ag
a × 100
? 100g K2SeO4 has Se =
142 + a
% of Se = 45.52
a × 100
? = 45.52
142 + a
? a = 118.2
Mol. wt. 2 × 39 + 118.2 + 64
Also equivalent of K2SeO4 = = = 130.1
2 2

Problem 3: A sample of H2SO4 (density 1.787g mL–1) is labeled as 86% by weight. What
is molarity of acid? What volume of acid has to be used to make 1 litre of
0.2m H 2SO4?

Solution: H2SO4 is 86% by weight


? Weight of H2SO4 = 86g
Weight of solution = 100g
100 100
? Volume of solution = mL = litre
1.787 1.787 × 1000
86
? M H 2SO 4 = = 15 .68
100
98 ×
1.787 × 1000
Let V mL of this H2SO4 areused to prepare 1 litre of 0.2 M H2SO4
? mM of conc. H2SO4 = mM of dilute H2SO4
V ? 15.68 = 1000 ? 0.2 ? V = 12.75 mL

Problem 4: The molecular mass of an organic acid was determined by the study of its
barium salt 4.290g of salt was quantitatively converted to free acid by the
reaction with 21.64 ml of 0.477 M H2SO4. The barium salt was found to
[Type text]

have two mole of water of hydration per Ba2+ ion and the acid is mono
basic. What is molecular weight of anhydrous acid?

Solution: Meq. of barium salt = Meq. of acid


4.290
× 1000 = 21.64 ? 0.4777 ? 2
M/2
Molecular weight of salt = 415.61
415.61 − 137 − 36
Molecular weight of anion = = 121.31
2
? Molecular weight of acid = 121.31 + 1 = 122.31

Problem 5: 25 mL of a solution of Na2CO3 having a specific gravity of 1.25g mL–1


required 32.9 mL of a solution of HCl containing 109.5gm of the acid per
litre for complete neutralization. Calculate the volume of 0.84 N H2SO4 that
will be completely neutralized by 125g of Na2CO3 solution.

109.5
Solution: N HCl = =3
36.5 × 1
Since Na2CO3 is completely neutralized by HCl
? Meq. of Na2CO3 = Meq. of HCl
N ? 25 = 32.9 ? 3
? N Na 2CO3 = 3.948
Now Na2CO3 fresh solution reacts with H2SO4
125
? Volume of Na2CO3 solution = = 100ml
1.25
? Meq. of H2SO4 = Meq. of Na2CO3
0.84 ? V = 100 ? 3.948
? Volume of H2SO4= 470 mL

Problem 6: Borax in water gives:


B 4 O 72− + 7 H 2 O 
→ 4H 3 BO 3 + 2OH −
How many gram of borax (Na2B4O7.10H2O) are required to?
a) Prepare 50 mL of 0.2 M solution
b) neutralize 25 mL of 0.1934 M of HCl and H2SO4 separately

 Normality 
Solution: Q Molarity = - 
 No. of replaceable OH 
? N=M?2
Thus Meq. of borax in solution = 50 ? 0.2 ? 2 = 20
[Type text]

w
? × 1000 = 20
M/2
w
? × 1000 = 20 ? w = 3.82g
382 / 2
For neutralization of HCl
Meq. of HCl = Meq. of borax
w
25 × 0.1934 = × 1000
382 / 2
? Weight of borax = 0.09235g
For neutralization of H2SO4
Meq. of borax = Meq. of H2SO4
w
× 1000 = 25 × 0.1934 × 2
382 / 2
? Weight of borax = 1.847g

Problem 7: A mixture containing As2O3 and As2O5 required 20.10 mL of 0.05N iodine
for titration. The resulting solution is then acidified and excess of KI was
added. The liberated iodine required 1.1113g hypo (Na2S2O3?5H2O) for
complete reaction. Calculate mass mixture. The reactions are:
As2O3 + 2I2 + 2H2O ?? As2O5 + 4H+ + 4I–
As2O5 + 4H+ + 4I–?? As2O3 + 2I2 + 2H2O

Solution: Meq. of I2 used = 20.10 ? 0.05 = 1.005


Let Meq. of As2O3 and Meq. of As2O5 in mixture be a and b respectively. On
addition of I2 to mixture, As+23 is converted to As+25 .
? Meq. of As2O5 = Meq. of I2 to mixture used = 1.005 – Meq. of As5+ formed.
or a = 1.005 …(1)
After the reaction with I2, mixture contains all the arsenic in +5 oxidation state
which is then titrated using KI + hypo. Thus,
Meq. of As2O3 as As+5 + Meq. of As2O5 as As+5 = Meq. of liberated I2
= Meq. of hypo used
1.113
or a+b= ×100
248
or a + b = 4.481
By equations (1) and (2),
b = 4.481 – 1.005 = 3.476
Meq. × Eq.Wt 1.005 ×198
? Wt. of As2O3 = = = 0.0497g
1000 4 ×100
3.476 × 230
and Wt. of As2O5 = = 0.1999g
4 ×1000
[Type text]

? Wt. of mixture = 0.0497 + 0.1999 = 0.2496g

Problem 8: Chile salt peter, a source of NaNO3 also contains NaIO3. The NaIO3 can be
used as source of iodine, produced in the following reactions.
IO3– + 3HSO3–?? I– + 3H+ +3SO42– …(1)
5I– + IO3– + 6H+?? 3I2(g) + 3H2O …(2)
One litre of chile salt peter solution containing 5.80g NaIO3 is treated with
stoichiometric quantity of NaHSO3. Now an additional amount of same
solution is added to reaction mixture to bring about the second reaction.
How many grams of NaHSO3 are required in step I and what additional
volume of chile salt peter must be added in step II to bring in complete
conversion of I– to I2?

5.8
Solution: ? Meq. of NaHSO3 = Meq. of NaIO3 = N ? V = ×1000
198 / 6
[Et. wt. of NaI = M/6 because I5+ + 6e ?? I–]
Meq. of NaHSO3 = 175.76
175.76 ×104
? w NaHSO3 = = 9.14g
2000
Also Meq. of I– formed in I step using valence factor 6 = 175.76
In II step valence factor of I– is 1 and valence factor IO3− is 5
175.76
Thus, Meq. of I– formed using valence factor 1 =
6
175.76
Also Meq. of NaIO3 used in step II =
6
175.76
? N?V=
6
5.8 175.76
or ×V = ? VNaIO3 = 200 mL
198 / 5 6

Problem 9: For estimating ozone in the air, a certain volume of air is passed through an
acidified or neutral KI solution when oxygen is evolved and iodide is
oxidized to give iodine. When such a solution is acidified, free iodine is
evolved which can be titrated with standard Na2S2O3 solution. In an
experiment, 10 litre of air at 1 atm and 27°C were passed through an
alkaline KI solution, at the end, the iodine entrapped in a solution on
titration as above required 1.5 mL of 0.01 N Na2S2O3 solution. Calculate
volume % of O3 in sample.
Solution: The reactions are
H2O + 2KI + O3?? 2KOH + I2 + O2
[Type text]

Also 2e– + I2?? 2I–


And 2S22+ ?? S4+5/2 + 2e–
? Meq. of I2 = Meq. of Na2S2O3 = 1.5 ? 0.01 = 1.5 ? 10–2
1.5 × 10 −2
or mM of I2 = = 7.5 ? 10–3
2
? mM of O3 = mM of I2 = 7.5 ? 10–3
(Q Mole ratio of O3 : I2 :: 1: 1)
7.5 ×10−6 × 0.0821× 300
? PO3 = = 184.725 ? 10–7 atm
10
? Volume % of O3 = 184.725 ? 10–7? 100 = 1.847 ? 10–3%

Problem 10: 1.5g of brass containing Cu and Zn reacts with 3M HNO3 solution, the
following reactions take place.
Cu + HNO3 ?? Cu2+ + NO2(g) + H2O
Zn + H+ + NO3–?? NH4+ + Zn2+ + H2O
The liberated NO2(g) was found to be 1.04 litre at 25°C and one atm.
a) Calculate the percentage composition of brass.
b) How many mL of 3M HNO3 will be required for completely reacting 1g
of brass?

Solution: (a) Cu0?? Cu2+ + 2e


N5+ + e ?? N4+
? Eq. of Cu = Eq. of NO2
w 1× 0.4  PV 
= Q mole of NO 2 = Eq. of NO 2 = 
63.6 / 2 0.0821× 298  RT 
? wCu = 1.35g
? wZn = 1.50 – 1.35 = 0.15g
1.35
? % of Cu = ×100 = 90%
1.5
0.15
and % of Zn = ×100 = 10%
1.5
(b) Thus, 1 g brass contains 0.9g Cu and 0.1g Zn
? Meq. of HNO3 = Meq. of and Meq. of HNO3 = Meq. of Cu
0.9
Zn or 3 × V2 = × 1000
0.1 636 / 2
or 3 × 8 × V1 = × 1000
65 / 2 ? V2 = 9.43 mL
[Type text]

? V1 = 0.128mL
(Q N +5 + 8e 
→ N −3 )
? Total volume of HNO3 used = 0.128 + 9.43 = 9.558 mL

Problem 11: 1.249 g of a sample of pure BaCO3 and impure CaCO3 containing some
CaO was treated with dil.HCl and it evolved 168 ml of CO2 at NTP. From
this solution, BaCrO4 was precipitated, filtered and washed. The precipitate
was dissolved in dilute sulphuric acid and diluted to 100 ml. 10 ml of this
solution, when treated with KI solution, liberated iodine which required
exactly 20 ml of 0.05N Na2S2O3. Calculate the percentage of CaO in the
sample.

168
Solution: nCaCO3 + nBaCO3 = nCO 2 = = 7.5 ? 10–3 ----- (1)
22400
+
2BaCO3??? 2BaCrO4 H
 BaCr2O7 KI
→ → I2 + Na2S2O3
20 × 10 −3 × 0.05 × 100
eq. of Na2S2O3 = eq. of I2 = eq of BaCr2O7 =
10
= 1 ? 10–12
1
Moles of BaCr2O7 = ? 10–2
6
2
Moles of BaCrO4 = (1? 10–2)
6
1
Moles of BaCO3 = ? 10–2 = 3.33 ? 10–3 ---- (2)
3
Weight of BaCO3 = 0.650 gm
From equation (1) and (2) we get
nCaCO3 = 4.17 ? 10–3
weight of CaCO3 = 100 × 4.17 × 10 −3 = 0.417 g
weight of CaO = 1.249 - 0.656 – 0.417 = 0.176
0.176
% of CaO = × 100 = 14.09%
1.249

Problem 12: In a quality control analysis for sulphur impurity 5.6g steel sample was
burnt in a stream of oxygen and sulphur was converted into SO2 gas. The
SO2 was then oxidized to sulphate by using H2O2 solution to which had been
added 30 mL of 0.04M NaOH. The equation for reaction is:
SO2(g) + H2O2(aq) + 2OH–(aq)?? SO4–2(aq) + 2H2O(l)
22.48 mL of 0.024M HCl was required to neutralize the base remaining
after oxidation reaction. Calculate % of sulphur in given sample.

Solution: Meq. of alkali added = 30 ? 0.04 = 1.2


[Type text]

Meq. of alkali left = 22.48 ? 0.024 = 0.54


? Meq. of alkali for SO2 and H2O2 = 1.2 – 0.54 = 0.66
0.66 × 40
? Weight of alkali used = = 0.0264
1000
Q 80g NaOH reacts with 64g SO2
64 × 0.0264
? 0.0264g NaOH reacts = = 0.021g SO2
80
Now Q 64g SO2 required = 32g S
32 × 0.021
? 0.021g SO2 required = = 0.0105g
64
0.0105
? % of S = ×100 = 0.1875%
5.6

Problem 13: A granulated sample of aircraft alloy (Al, Mg, Cu) weighing 8.72g was first
treated with alkali and then with very dilute HCl, leaving a residue. The
residue after alkali boiling weighed 2.10g and the acid insoluble residue
weighed 0.69g. What is the composition of the alloy?

Solution: Let Al, Mg and Cu be a, b and c g respectively.


2Al + 2NaOH  
2H O
2
→ 2NaAlO2 + 3H2
Mg + 2HCl ?? MgCl2 + H2
Cu + HCl ?? No reaction
i.e., only Al reacts with NaOH and then only Mg reacts with HCl
? a + b + c = 8.72
b + c = 2.10 (Residue left after alkali treatment)
c = 0.69 (Residue left after acid treatment)
? b = 6.62g
6.62
? % of Al = × 100 = 75.9
8.72
1.41
% of Mg = ×100 = 16.2
8.72
0.69
% of Cu = × 100 = 7.9
8.72

Problem 14: 25 ml from a stock solution containing NaHCO3 and Na2CO3 was diluted to
250 ml with CO2 free distilled water. 25 ml of the diluted solution when
titrated with 0.12 M HCl required 8 ml when phenolphthalein was used as
an indicator. When 20 ml of diluted solution was titrated with same acid
required 18 ml when methyl orange was used as an indicator. Calculate
concentration of NaHCO3 in the stock solution in gm/litre and in mole/litre.
To 100 ml of the stock solution how much NaOH should be added so that all
bicarbonate will be converted into carbonate?
[Type text]

Solution: When phenolphthalein used, NaHCO3 remains unaffected and Na2CO3 will be
converted into NaHCO3
Na2CO3 + HCl ?? NaHCO3 + NaCl
1
eq of Na2CO3 = eq. of HCl
2
1 w
= 0.12 ? 8 ? 10–3? 10
2 53
w = 1.017 gm in 25 ml = 40.7 gm / litre = 0.38 mole / litre
When methyl orange is used NaHCO3 and Na2CO3 both will be converted
into CO2
Eq. (Na2CO3) + eq(NaHCO3) = eq (HCl)
 W –3 250
1.92 × 10 +  = 18 ? 0.12 ? 10 ?
−3

 84  20
W
1.92 ? 10–2 + = 2.7 ? 10–2
84
For NaHCO3 W = 0.6552 gm in 250 ml = 26.2 gm /litre = 0.312 M
eq. of bicarbonate = eq of NaOH
100 W
0.78 ? 10–2? =
25 40
W = 1.248 gm

Problem 15: One litre of a mixture of O2 and O3 at NTP was allowed to react with an
excess of acidified solution of KI. The iodine liberated required 40 mL of
M/10 sodium thiosulphate solution for titration. What is the weight per cent
of ozone in the mixture?
Ultraviolet radiation of wavelength 300 nm can decompose ozone.
Assuming that one photon can decompose one ozone molecule, how many
photons would have been required for the complete decomposition of ozone
in the original mixture?

Solution: O 3 + 2KI + H 2 O 
→ 2KOH + I 2 + O 2
I 2 + 2 Na 2 S 2 O 3 
→ Na 2 S 4 O 6 + 2 NaI
1
∴ Milli mole of O3 = milli mole of I2 = × mM of Na 2S2 O3 (mM = M × Vin ml )
2
1 1
= × 40 × = 2mM = 0.002 mole
2 10
Total milli mole of O2 and O3 in mixture are calculated from PV = nRT
1 × 1 = n × 0.0821× 273
∴ n = 0.044 mole
∴ Mole of O2 = 0.044 – 0.002 = 0.042
Now weight of O2 = 0.042 × 32 g = 1.344 g
Weight of O3 = 0.002 × 48 g = 0.096 g
[Type text]

0.096
∴ % of O3 = × 100 = 6.7%
1.344
0.096 × 6.023 × 10 23
No. of photon or molecules of ozone = = 1.2 × 10 21
48
Problem 16: 2.480g of KClO3 are dissolved in conc. HCl and the solution was boiled.
Chlorine gas evolved in the reaction was then passed through a solution of KI
and liberated iodine was titrated with 100 mL of hypo. 12.3 mL of same hypo
solution required 24.6 mL of 0.5 N iodine for complete neutralization.
Calculate % purity of KClO3 sample.
Solution: 2KClO3 + 12HCl ?? 2KCl + 6H2O + 6Cl2
Cl2 + 2KI ?? 2KCl + I2
Also Meq. of I2 = Meq. of Hypo = 100 × 1
Q N Hypo12.3 = 24.6 × 0.5; ∴ N Hypo = 1
100
Also mM of Cl2 = mM of I2 = = 50
2
2 × mM of Cl2 2 × 50 50
Also mM of KClO3 = = =
6 6 3
w 50  Wt. ×1000  w
∴ ×1000 =  milli-mole (mM)= ∴ KClO 3 = 2.042
122.5 3  mol.wt. 
2.042
% of KClO3 = × 100 = 82.32%
2.48

Problem 17: P and Q are two elements which forms P2Q3 and PQ2. If 0.15 mole of P2Q3
weights 15.9g and 0.15 mole of PQ2 weights 9.3g, what are atomic weights of
P and Q?

Solution: Let atomic weight of P and Q are a and b respectively


\ Molecular weight of P2Q3 = 2a + 3b
and Molecular weight of PQ2 = a + 2b
Now given that 0.15 mole of P2Q3 weigh 15.9g
15.9  wt. 
(2a + 3b) = ∴ = mole 
0.15  mol. wt. 
9.3
Similarly, (a + 2b) =
0.15
Solving these two equations
b = 18
a = 26

Problem 18: 25 mL of a solution containing Fe2+ and Fe3+ sulphate acidified with H2SO4 is
reduced by 3g of metallic zinc. The solution required 34.25 mL of N/10
[Type text]

solution of K2Cr2O7 for oxidation. Before reduction with zinc, 25 mL of the


same solution required 22.45 mL of same K2Cr2O7 solution. Calculate the
strength of FeSO4 and Fe2(SO4)3 in g/litre of solution.
Solution: Redox changes are
Case I: Fe 2+  2
  → Fe 2+
Zn dust+H SO
4
(i.e. no change)
3+ 2+
2e + Fe 2 
Zn dust + H 2SO 4
→ 2Fe
Q Zn dust is used as reducing agent and thus,
Zn  → Zn 2+ + 2e
Let a meq. of Fe2+ and b Meq. of Fe3+ be present in 25 mL solution. In case I,
after reduction with Zn.
Meq. of Fe2+ + Meq. of Fe2+ from Fe3+ = a + b
Now these are oxidized by K2Cr2O7
∴ Total meq. of Fe2+ = Meq. of K2Cr2O7
1
a + b = 34.25 ×
10
∴ a + b = 3.425 …(1)
Case II: If reduction is not made, the solution contains Fe2+ and Fe3+ of
which only Fe2+ are oxidized by K2Cr2O7.
∴ Meq. of Fe2+ = Meq. of K2Cr2O7
1
a = 22.45 ×
10
a = 2.245
∴ By equation (1) b = 3.425 – 2.245 = 1.18
∴ Meq. of FeSO4 = a = 2.245 Meq. of Fe2(SO4=)3 = b = 1.18
Q Meq. of FeSO 4 = a = 2.245 (in 25 mL)
(in 25 mL) w
∴ ×1000 = 1.18
w M/2
∴ × 1000 = 2.245 Q M. wt. of Fe2 (SO 4 )3 = 400
M /1
Q M. wt. of FeSO 4 = 152 ∴ Wt. of Fe2 (SO 4 ) in 25 mL=0.236g

∴ Wt. of FeSO4 in 25 mL = 0.341g ∴ Strength of Fe2 (SO 4 )3 = 9.45g / litre

∴ Strength of FeSO4 = 13.64g / litre

Problem 19: 50 mL solution of H2O2 was treated with excess KI (s) and the solution was
acidified with acetic acid. The liberated iodine required 40 mL 0.5 M Na2S2O3
solution for the end point using starch as indicator. Find the “molarity” and
“volume strength” of the H2O2 solution.

Solution: Milli equivalent of H2O2 = milli equivalent of I2 = milli equivalent of Na2S2O3


Milli equivalent of H2O2 = milli equivalent of Na2S2O3
50 × N = 40 × 0.5 ×1
[Type text]

20
N= = 0.4
50
N
M=
n − factor
0.4
M (molarity) = = 0.2
2
Volume strength = 5.6 × N
= 5.6 × 0.4 = 2.24

Problem 20: 2.0g sample of KMnO4 (MW = 158) containing some inert materials was
dissolved in water acidified with H2SO4 resulting solution was treated with 62
mL 0.5 M oxalic acid solution. The excess of oxalic acid was back titrated
with 20 mL 0.1 M K2Cr2O7. Calculate percent purity of KMnO4 sample.
+7 +2 +3× 2 +4
Solution: KMnO 4  → MnSO 4 , H 2 C 2 O 4 
→ 2 C O2
n-factor = 7 – 2 = 5 n-factor = 6 – 4 = 2
+6× 2 +3× 2
K 2 Cr2 O7 
→ Cr2 (SO 4 )3
n-factor = 2(6 – 3) = 6
So, 0.5 M oxalic aid = 2 × 0.5 i.e. 1 N oxalic acid
0.1 M K2Cr2O7 = 6 × 0.1 i.e. 0.6N K2Cr2O7
No. of milli equiv. of KMnO4 in the sample
= No. of milli equivalents of oxalic acid reacted with it
= (no. of milli equivalents of oxalic acid taken) –
(no. of milli equivalents of oxalic acid remained unreacted)
= (no. of milli equivalents of oxalic aid taken) –
(no. of milli equivalents of K2Cr2O7 consumed.
= 62 × 1 – 20 × 0.6
= 50
50
No. of milli equivalents KMnO4 = = 10
5
Weight of KMnO4 = 10 × 10–3 × 158 = 1.58g
1.58
Hence ×100 i.e. 79%
2

KVPY QUESTIONS
1. Massofaliquid isweighedcorrecttothreedecimalplaceanditsvolumeismeasuredcorrecttoone
decimalplace.Thedensityoftheliquidcalculatedfromtheabovedatawillbecorrectto (2007)
(A)threedecimalplace (B)twodecimalplace (C)onedecimalplace (D)fourdecimalplace

2. Copper in an alloy is estimated by dissolving in conc. nitric acid. In this process copper is converted
to cupric nitrate with the evolution of nitric oxide (NO). The misture when treated with potassium
[Type text]

iodide forms cupric iodide. Which is unstable and decomposes to cuprous iodide and iodine. The
amount of copper in the alloy is estimated by litrating the libereated iodine with sodium thiosulphate.
The reactions are (2010)
a Cu + b HNO3 → c Cu(NO3 )2 + d NO + eH2O
f Cul2 → gCu2I2 + hI2
i Na S O + ˆjI → k Na S O + INal
2 2 3 2 2 4 6
(Fill up the blanks)
(A)The coefficients are : a− .b − ,c − ,d − and e−
(B)The coefficients are : f − ,g− and h−
(C)The coefficients are : i − ,j − ,k − and I−
(D)If 2.54 g of I2 is evolved from a 2.0 g sample of the alloy, what is the percentage of copper in the
alloy?
(atomic, weight of iodine and copper are 127 and 63.5, respectively)

3. The molar mass of CaCO3 is 100 g. The maximum amount of carbon dioxide that can be liberated on
heating 25 g of CaCO3 is (2010)
(A) 11 g (B) 55 g (C) 22 g (D) 2.2 g

4. The weight of calcium oxide formed by burning 20 g of calcium in excess oxygen is (2012)
(A) 36 g (B) 56 g (C) 28 g (D) 72 g

5. The weight of calcium oxide formed by burning 20 g of calcium in excess oxygen is (2012)
(A) 36 g (B) 56 g (C) 28 g (D) 72 g

6. Upon fully dissolving 2.0 g of a metal in sulfuric acid, 6.8 g of the metal sulfate is formed. The
equivalent weight of the metal is (2012)
(A) 13.6 g (B) 20.0 g (C) 4.0 g (D) 10. 0 g

7. The amount of metallic Zn (Atomic weight = 65.4) required to react with aqueous sodium hydroxide to
produce 1 g of H2, is (2013)
(A) 32.7 g (B) 98.1 g (C) 65.4 g (D) 16.3 g
12 13
8. Natural abundances of C and C isotopes of carbon are 99% and 1%, respectively. Assuming they
only contributes to the mol. wt. of C2F4, the percentage of C2F4 having a molecular mass of 101 is
(2013)
(A) 1.98 (B) 98 (C) 0.198 (D) 99

9. The volume of oxygen at STP required to burn 2.4 g of carbon completely is (2014)
(A) 1.12 L (B) 8.96 L (C) 2.24 L (D) 4.48 L

10. 10 moles of a mixture of hydogen and oxygen gases at a pressure of 1 atm at constant volume and
temperature, react to form 3.6 g of liquid water. The pressure of the resulting mixture will be closest to
(A) 1.07 atm (B) 0.97 atm (C) 1.02 atm (D) 0.92 atm

11. Complete reaction of 2.0 g of calcium (at wt. = 40) with excess HCL produces 1.125L of H2 gas.
Complete reaction of the same quantity of another metal “M” with excess HCL produces 1.85 L of H2
gas under indentical conditions. The equivalent weight of “M” is closest to
(A) 23 (B) 9 (C) 7 (D) 12

12. Mass of a liquid is weighed correct to three decimal place and its volume is measured correct to one
decimal place. The density of the liquid calculated from the above data will be correct to (2007)
[Type text]

(A) three decimal place (B) two decimal place (C) one decimal place (D) four decimal
place

13. In balancing the reaction. (2008)


xH2S + 2NaNO3 + 2HCl = y S + zNO + kNaCl + 4H2O
one would get x, y, z and k, respectively, as
(A) 3,3,2 and 2 (B) 2,2,3 and 3 (C) 3,3,4 and 4 (D) 4,4,3 and 3

14. Upon mixing equal volumes of aqueous solutions of 0.1 M HCl and 0.2 M H2SO4, the concentration of
+
H in the resulting solution is: (2012)
(A) 0.30 mol/L (B) 0.25 mol/L (C) 0.15 mol/L (D) 0.10 mol/L

15. The number of moles of Br2 produced when two moles of potassium permanganate are treated with
excess potassium bromide in aqueous acid medium is: (2012)
(A) 1 (B) 5 (C) 2 (D) 4

Answers??

3: A 4:C 5:C 6:B


7:A 8:A 9:D 10:B
11:D 12:C 13:A 14:B
15:B

Assignments
SECTION – I Single Choice Questions
1. The number of moles of CaCl2 needed to react with excess of AgNO3 to produce 4.31
gram of AgCl.
(a) 0.030 (b) 0.015
(c) 0.045 (d) 0.060

2. How many moles of electron is needed for the reduction of each mole of Cr in the
reaction,.
CrO 5 + H 2SO 4 
→ Cr2 (SO 4 ) 3 + H 2 O + O 2
(a) 4 (b) 3
(c) 5 (d) 7

3. 1.60g of a metal were dissolved in HNO3 to prepare its nitrate. The nitrate on strong
heating gives 2g oxide. The equivalent weight of metal is
(a) 16 (b) 32
(c) 48 (d) 12

4. A metal oxide has 40% oxygen. The equivalent weight of the metal is
(a) 12 (b) 16
[Type text]

(c) 24 (d) 48

5. The density of 1 M solution of NaCl is 1.0585 g/ml. The molality of the solution is:
(a) 1.0585 (b) 1.00
(c) 0.10 (d) 0.0585

6. Which of the following gives the molarity of a 17% solution of NaOAc in H2O? Given
the density is 1.09 g/ml
(a) 2.26 ? 10–6M (b) 0.207M
(c) 2.07M (d) 2.26M

7. A 15 volume sample of an H2O2 solution is equivalent to


(a) 5.30 N (b) 1.77N
(c) 2.68N (d) 7.50 N

8. 100 mL of N/5 NaOH will neutralize


(a) 0.0618g of H3BO3 (b) 0.1855g of H3BO3
(c) 1.2368g of H3BO3 (d) 0.03092g of H3BO3

9. The relation between molarity (M) and molality (m) is given by (r = density of solution,
M1 = molecular weight of solute)
1000M 1000M
(a) m = (b) m =
1000ρ − M1 1000ρ − MM1
1000MM1 1000M
(c) m = (d) m =
1000ρ − MM1 1000ρ − MM1

10. 0.2 mol of HCl and 0.1 mol of barium chloride were dissolved in water to produce a 500
mL solution. The molarity of the Cl– is
(a) 0.06M (b) 0.09M
(c) 0.12M (d) 0.80M

11. To prepare a 0.5 M KCl solution from 100 mL of 0.40M KCl we need to add
(a) 0.75g of KCl (b) 20 mL of water
(b) 0.1 mol of KCl (d) 0.2 mol of KCl

12. What would be the normality of a 0.1 M K2Cr2O7 solution used as a precipitating agent
for Pb2+?
(a) 0.1 N (b) 0.6N
(c) 0.4N (d) 0.2N

13. 0.44g of a colourless oxide of nitrogen occupies 224 mL at STP. The compound is:
(a) N2O (b) NO
(c) N2O5 (d) NO2
[Type text]

14. H3PO4 is a tribasic acid and one of its salt is NaH2PO4. What volume of 1 M NaOH
would be added to 12 g NaH2PO4 (molecular weight of 120) to exactly convert it into
Na3PO4.
(a) 100 mL (b) 300 mL
(c) 200 mL (d) 80 mL

15. 74.4g of a metallic chloride contains 35.5g of chlorine. The equivalent weight of the
metal is
(a) 19.5 (b) 35.5
(c) 38.9 (d) 78.0

16. It takes 2.56 ? 10–3 equivalent of KOH to neutralize 0.1254g H2XO4. The number of
neutrons in is
(a) 16 (b) 8
(c) 7 (d) 32

17. How many grams of sodium bicarbonate are required to neutralize 10.0 ml of 0.902 M
vinegar?
(a) 8.4g (b) 1.5g
(c) 0.75g (d) 1.07g
18. A sample of hard water contains 244 ppm of HCO 3− ions. What is the minimum mass of
CaO required to remove HCO 3− ions completely from 1 kg of such water sample
(a) 56 mg (b) 112 mg
(c) 168 mg (d) 244 mg

19. 100 ml of each of 0.5 N NaOH, N/5 HCl and N/10 H2SO4 are mixed together. The
resulting solution will be
(a) Acidic (b) Neutral
(c) Alkaline (d) None

20. The chloride of a metal (M) contains 65.5% of chlorine. 100 ml of the vapour of the
chloride of the metal at STP weight 0.72g. the molecular formula of the metal chloride is
(a) MCl3 (b) MCl
(c) MCl2 (d) MCl4

21. If 0.5 mole of BaCl2 is mixed with 0.2 mole of Na3PO4 the maximum number of moles of
Ba3(PO4)2 that can be formed is
(a) 0.7 (b) 0.5
(c) 0.3 (d) 0.1

22. The percent loss in weight after heating a pure sample of potassium chlorate (Molecular
weight = 122.5) will be
(a) 12.25 (b) 24.50
(c) 39.18 (d) 49
[Type text]

23. For the reaction


MnO−4 + C 2 O 24− + H + 
→ Mn 2+ + CO 2 + H 2 O
the correct coefficients of the reactants for the balanced reaction are
MnO −4 C 2 O 24− H+
(a) 2 5 16
(b) 16 5 2
(c) 5 16 2
(d) 2 16 5
24. In an experiment, 50 ml of 0.1 M solution of a salt reacted with 25 ml of 0.1 M solution
of sodium sulphite. The half equation for the oxidation of sulphite ion:
) + H 2 O 
→ SO 24(aq ) + 2H (aq )
− − +
SO 22(aq
If the oxidation number of the metal in the salt was 3, what would be the new oxidation
number of the metal?
(a) 0 (b) 1
(c) 2 (d) 4
25. The chloride of a metal contains 71% chlorine by weight and the vapour density of it is
50. The atomic weight of the metal will be
(a) 29 (b) 58
(c) 35.5 (d) 71

SECTION – II May be more than one choice


1. The number of moles of Cr2 O 72− needed to oxidize 0.136 equivalent of N 2 H 5+ by the
reaction.
N 2 H5+ + Cr2O72− 
→ N 2 + Cr 3+ + H 2O is
(a) 0.136 (b) 0.272
(c) 0.816 (d) 0.0227

2. The hydrated salt Na2SO4?nH2O undergoes 55.9% loss in weight on heating and
becomes anhydrous. The value of n will be
(a) 5 (b) 3
(c) 7 (d) 10
3. 22.4 litres of H2S and 22.4 litres of SO2 both at STP are mixed together. The amount of
sulphur precipitated as a result of chemical reaction is:
(a) 16g (b) 23g
(c) 48g (d) 96g
4. What volume of 0.1 M H2SO4 will be required to produce 17.0g of H2S by the reaction
5H2SO4 + 8NaI ?? 4Na2SO4 + 4I2 + H2S + 4H2O?
(a) 70.0 L (b) 50.0L
(c) 25.0 L (d) 5.0 L
[Type text]

5. 125 mL of 10% NaOH (w/V) is added to 125 mL of 10% HCl (w/V). The resultant
solution becomes
(a) alkaline (b) strongly alkaline
(c) acidic (d) neutral

6. How many grams of copper will be replaced in 2 L of a 1.50 M CuSO4 solution if the
later is made to react with 27.0g of aluminium? (Cu = 63.5, Al = 27.0)
(a) 190.50g (b) 95.25g
(c) 31.75g (d) 10.65g

7. A sample of H2SO4 (density 1.8g mL–1) is 90% by weight. What is the volume of the acid
that has to be used to make 1 L of 0.2 M H2SO4?
(a) 16 mL (b) 10 mL
(c) 12 mL (d) 18 mL
(d) 100 mL
8. An aqueous solution of urea containing 18g urea in 1500cm3 of solution has a density of
1.502 g/cm3. If the molecular weight of urea is 60, then the molality of solution is
(a) 0.2 (b) 0.192
(c) 0.064 (d) 1.2
9. For preparing M/10 solution of H2SO4 in one litre we need H2SO4
(a) 9.8g (b) 49.0g
(c) 4.8g (d) 0.09g
10. The normality of 1% (weight/volume) H3PO4 is nearly
(a) 0.02 (b) 0.2
(c) 0.1 (d) 1
11. Number of molecules of oxalic acid in 100 mL of 0.02 N oxalic acid solution are
(a) 10–3 × 6.023 × 1023 (b) 2 × 6.023 × 1023
23
(c) 3 × 6.023 × 10 (d) 4 × 6.023 × 1023
12. 3.0 molal NaOH solution has a density of 1.110 g/ml. The molarity of the solution is
(a) 2.97 (b) 3.05
(c) 3.64 (d) 3.050
13. A mixture of magnesium chloride and magnesium sulphate is known to contain 0.6 moles
of chloride ions and 0.2 moles of sulphate ions. The number of moles of magnesium ions
present is
(a) 0.4 (b) 0.5
(c) 0.8 (d) 1.0

14. The reaction between aluminium metal and dilute hydrochloric acid produces H2(g) and
Al3+ ions. The molar ratio of aluminium used to hydrogen produced is
(a) 1:2 (b) 2:1
(c) 2:3 (d) 3:2
[Type text]

15. Element X reacts with oxygen to produce a pure sample of X2O3. In an experiment it is
found that 1.00g of X produces 1.16g of X2O3. Calculate the atomic weight of X.
Given: atomic weight of oxygen, 16.0 g mol–1.
(a) 67 (b) 100.2
(c) 125 (d) 150

16. In an aqueous solution of barium nitrate, the [NO3–] is 0.80M. This solution is labelled as
(a) 0.080 N Ba(NO2)2 (b) 0.160 M Ba(NO3)2
(b) 0.040 M Ba(NO3)2 (d) 0.080 MNO3–

17. 0.01 mole of H3PO2 and 0.01 moles of H3PO3 are titrated with V ml of 0.1 M NaOH
solution in presence of phenolphthalein indicator, V will be
(a) 300 ml (b) 200 ml
(c) 400 ml (d) 500 ml

18. 10 ml of a solution of HOOC?COONa.H2O is completely neutralized by 10 ml. of


decinormal solution of caustic soda. In a separate titration, 10ml of 0.2M kMnO4 solution
in acidic medium be required for complete oxidation of x ml of above
HOOC?COONa.H2O solution. What is the value of x?
(a) 100 ml (b) 50 ml
(c) 10 ml (d) 20 ml
19. 36 mL 0.5 M Br2 solution upon being made alkaline undergoes complete
disproportionation into Br–and BrO3− . The resulting solution requires 45 mL As(III)
solution to reduce BrO3− to Br–. Given that As(III) is oxidised to As(V), what is the
molarity of As(III) solution?
(a) 0.2 (b) 0.1
(c) 0.4 (d) 0.5

20. A sample of hard water contains 192 ppm of SO 24 − ions and 305 ppm of HCO3− ions with
Ca2+ with as the only cations. The concentration in ppm of Ca2+ ions (Atomic mass = 40)
in the hard water is:
(a) 100 (b) 180
(c) 140 (d) 80

21. Hydroxyl amine reduces iron (III) according to following equation


NH 2 OH + Fe2 (SO 4 )3 
→ N 2 (g) + H 2 O + FeSO 4 + H 2 SO 4
which statement is correct
(a) n-factor for Hydroxyl amines is 1
(b) equivalent weight of Fe2(SO4)3 is M/2
(c) 6 meq of Fe2(SO4)3 is contained in 3 millimoles of ferric sulphate
(d) all of these.
22. 3.16 g of KMnO4 is dissolved in water and the solution is made upto 1 litre. An unknown
salt containing 6.88g Fe2+ ion was dissolved in water and solution was made upto 100 ml.
[Type text]

It was found that 20 ml of salt solution decolourised 27.25 ml of the above permanganate
solution. Which of the following statements are correct?
(a) Normality of ferrous ion = 0.15 N (b) Actual strength of ferrous ion 68.8 g liter-1
(c) Percentage of ferrous ion is 10.58 (d) Normality of ferrous ion is 0.13 N

23. 1 mol of H2SO4 will exactly neutralize


(a) 2 mol of ammonia (b) 1 mol of Ca(OH)2
(c) 0.5 mol of Ba(OH)2 (d) 2 mol of NaOH

24. During the titration of a mixture of NaOH, Na2CO3 and inert substances against HCl.
(a) Phenolpthalein is used to detect the end point when half equivalent of Na2CO3 and
full equivalent NaOH is consumed
(b) Phenolpthalein is used to detect the second end point
(c) Methyl orange is used to detect the final end point
(d) Methyl orange is used to detect the first end point

25. 1.66 g KI reacts with excess of KIO3 to produce I2, which converts Na2S2O3 into SO 24− . If
the hypo solution was decimolar, then the volume required to reach equivalent point will
be
(a) 0.015 litre (b) 15 ml
(c) 0.02 litre (d) 20 ml

SECTION – III Comprehension Type Questions

@ Write-up I
Double Titration
1. Methyl orange as an indicator
The volume of 0.1 NHCl used in the titration will correspond the neutralization as
directed.
NaHCO3 + HCl ?? NaCl + CO2 + H2O (colour change)
Na2CO3 + 2HCl ?? NaCl + CO2 + H2O (colour change)
NaOH + HCl ?? NaCl + H2O (colour change)
The volume of 0.1M NaOH in the titration will neutralize as directed
HA (mono basic acid) + NaOH ?? NaA (colour change)
H2A (di basic acid) + NaOH ?? NaHA (colour change)
H3A (tribasic acid) + NaOH ?? NaH2A (colour change)

2. Phenolphthelien as an indicator
The volume of 0.1 NHCl used in the titration will correspond the neutralization as
directed.
NaHCO3 + HCl ?? (no colour change)
[Type text]

Na2CO3 + HCl ?? NaHCO3 (colour change)


NaOH + HCl ?? NaCl (colour change)
The volume of 0.1M NaOH in the titration will neutralize as directed
HA (mono basic acid) + NaOH ?? NaA (colour change)
H2A (di basic acid) + 2NaOH ?? Na2A (colour change)
H3A (tribasic acid) + 2NaOH ?? Na2HA (colour change)

1. 0.01 mole of H3PO4 and 0.01 mole H3PO2 reacts with V ml of standard NaOH in
presence of phenolpthelien indicator volume of NaOH used is
(a) 500 ml (b) 400 ml
(c) 300 ml (d) 200 ml

2. 25 ml of Na2CO3 solution requires 100 ml of 0.1 N HCl to reach end point with
Phenolphthalein as indicator. Molarity of resulting solution with respect to HCO 3− ion
(a) 0.008 M (b) 0.004 M
(c) 0.16 M (d) 0.08 M

3. 20 ml of x M HCl neutralizes completely 10 ml of 0.1 M NaHCO3 solution and a further


5 ml of 0.2 M Na 2CO3 solution to methyl orange end point. The value of x is:
(a) 0.167 M (b) 0.133M
(c) 0.15 M (d) 0.2 M

4. Certain moles of CO2 is dissolved in excess of NaOH. The resulting solution is divided
into two equal parts. One part needs 30 ml of 2.5 N HCl and other part needs 40 ml of 2.5
N HCl using phenolphthalein and methyl orange indicator respectively. The mass of CO2
dissolved is:
(a) 2.5 × 10 −2 (b) 5 × 10 −2
(c) 10 × 10 −2 (d) 1.5 × 10 −2

@ Write-up II
Iodine Titration
All such titration which involves the direct titration of Iodine with a reducing agent are
grouped under Iodimetry. Iodimetry is employed to determine the strength of reducing
agent such as sodium those sulphate
I2 + Na2S2O3?? I– + S4 O 6−−
If iodine is liberated as a result of chemical reaction involving oxidation of an idodide
ion by a strong oxidizing agent in neutral or acidic medium the liberated iodine is then
titrated with reducing agent. Iodometry is used to estimate the strength of oxidizing
agent.
For example the estimation of Cu++ with thiosulphate.
[Type text]

Cu++ + I–?? Cu2I2 + I2


I2 + S2 O3−− 
→ S4 O 6−− + I −
Starch used as indicator near the end point which form blue colour complex with I3− . The
blue colour disappears when there is no more of free I2.
5. In Iodine titration Iodine remains in solution in the form of
(a) I3− (b) I2
(c) I3+ (d) I–

6. In the reaction, 2CuSO4 + 4KI ?? Cu2I2 + 2K2SO4 + I2 the ratio of equivalent weight of
CuSO4 to its molecular weight is:
(a) 1/8 (b) ¼
(c) ½ (d) 1

7. When 159.50g of CuSO 4 in a solution is reacted with KI, then the liberated iodine
required 100 ml 1 M Na 2S2O3 for complete reaction, then what is the percentage purity
of CuSO 4 sample used in making the solution.
(a) 10% (b) 20%
(c) 5% (d) None of these

8. 100 ml of 0.1 N hypo decolourised iodine by the addition of x g of crystalline blue vitriol
to excess of KI. The value of x is
(a) 5g (b) 2.5g
(c) 10g (d) 1.25g

@ Write-up III
Like acid base titration, in redox titration also, the equivalence point is reached when the
reducing agent is completely oxidized by the oxidizing agent. But contrary to the acid-
base titrations, oxidizing agents can themselves be used as internal indicator in redox
titration e.g. Cr2O72– (orange yellow), Cr3+ (green), MnO4– (purple), Mn2+ (light pink),
where strength of the solution may be expressed as molarity i.e. number of moles of
solute per litre of solution.
9. In a titration experiment, a student finds that 23.48 ml of a NaOH solution are needed to
neutralize 0.5468g of KHP (molecular formula KHC8H4O4). What is the concentration in
molarity of NaOH solution?
(a) 0.114 M (b) 0.228M
(c) 0.057M (d) 0.028 M
10. A 16.42 ml volume of 0.1327M KMnO4 solution is needed to oxidize 25.00 ml of a
FeSO4 solution in an acidic medium. What is the number of moles of FeSO4 being
oxidized for the reaction
5Fe 2+ + MnO −4 + 8H +  → Mn 2+ + 5Fe3+ + 4H 2 O
[Type text]

(a) 2.18 ? 10–2 ml (b) 1.09 ? 10–2 ml


(c) 0.545 ? 10–2 ml (d) 0.272 ? 10–2 ml

11. A purple coloured solution is added from a burette to FeSO4 solution kept in the flask.
After sometime, the purple colour changes to light pink. The ion formed from that
solution is
(a) MnO −4 (b) Fe2+
(c) Fe3+ (d) Mn2+
12. Concentrated aqueous sulphuric acid is 98% H2SO4 by mass and has a density of 1.84
g/ml. What volume of the concentrated acid is required to make 5 litre of 0.50M H2SO4
solution.
(a) 271.7 ml (b) 13.5 ml
(c) 135.85 ml (d) 27.1 ml

@ Write-up IV
KIO3 is an oxidant which is reduced to I2 or ICl depending on the medium whether it is
dil. HCl or 6 M HCl. KI on the other hand is a reductant, which is oxidized to I2 or ICl
depending on the condition as mentioned above. In IIT-JEE – 1992, a question based on
this concept was asked and the same in slightly modified form but basic spirit remaining
the same, reads as follows:
1.0g sample of AgNO3 containing some background inert impurities was dissolved in
water and solution was treated with 20 ml KI solution. The precipitated AgI was filtered
off. The filtrate containing unreacted KI was back titrated with 7.5 mL 0.1 M KIO3 in 6 M
HCl. In another titration with 10 mL fresh stock solution of KI exactly required 1.0 mL
0.1 M KIO3 solution for complete oxidation in 6 M HCl solution. Find percent purity of
AgNO3 in the sample.
A JEE aspirant solved the problem in the following manner.
KIO3 vs. KI titration:
+5 +1
K I O3 
→ I − Cl
− −
n-factor = 4
? 0.1 M KIO3 is 0.4N
Strength of stock solution of KI
V1N1 = V2N2
10 ? N1 = 10 ? 0.4
N1 = 0.4 N
No. of m. equiv. of AgNO3 in the sample
= No. of milli equiv. of KI added – No. of milli equiv. of KIO3 conserved in the back
titration
= 20 ?0.4 – 7.5 ? 0.4 = 5
n-factor of AgNO3 in its reaction with KI (reaction being double displacement) is 1 (=
charge carried by cation or anion).
[Type text]

5
So, No. of m. mole of AgNO3 in the sample = =5
1
% Mass of AgNO3 in the sample = 5 × 10 − 3 × 100 = 85

13. Correct statement among the following is?


(a) The solution is correct
(b) The solution is wrong as KIO3 under the conditions given in the figure must be
reduced to I2
(c) The solution is wrong as KI must have oxidized to I2 under the condition as laid down
in the question.
(d) The solution is wrong as there is fallacy in the calculation of no. of m. equiv. of
AgNO3
14. The correct final answer according to your calculation should be
(a) the same as calculated above (b) 42.5%
(c) 50% (d) 37.5%
15. The volume (mL) of 0.25 M KIO3 needed to oxidize completely 40.0 mL 0.1 M KI in dil.
HCl medium is:
(a) 8.0 (b) 10.4
(c) 3.2 (d) 4.8
16. An aqueous solution containing 0.10g KIO3 (Formula weight = 214) and an excess of KI
was acidified with HCl. The liberated I2 consumed 45.0 ml of thiosulphate. The molarity
of sodium thiosulphate solution is
IO 3− + I − + H + 
→ I2 + H 2O
(a) 0.063M (b) 0.0313M
(c) 0.126M (d) 0.252M

? Write-up V
Titration is a method of finding out the strength of an unknown solution by reacting with
a known solution using a suitable indicator. The choice of the indicator depends on the
nature of the reaction. The method is carried out by taking a definite volume of solution
of unknown strength in the beaker, indicator is added and then from the burette the
known solution is added till the indicator changes the colour which indicates the reaction
is complete. At this point the number of equivalence of the solution in the beaker is equal
to the number of equivalence of the solution added from the burette.

17. 10cc 2M phosphorous acid is titrated with 0.5M NaOH. The volume of NaOH needed to
reach equivalence point to completely react with phosphorus acid is
(a) 10cc (b) 80cc
(c) 40cc (d) 20cc

18. If in the above titration NaOH is replaced by 0.2M Fe2(Cr2O7)3 then the volume of
Fe2(Cr2O7)3 required to completely react with phosphorous acid is
(a) 8cc (b) 11.1cc
(c) 10cc (d) 10.5cc
[Type text]

19. A solution containing x m mole of H2SO4, y millimole of H2C2O4 requires A ml 1M


Ca(OH)2 for complete neutralisation. B ml of 0.1M Mg(MnO4)2 is required for complete
oxidation of the same solution. x and y can be expressed as
2B − A A 2A − B B
(a) , (b) ,
2 2 2 2
A−B A A − 2B B
(c) , (d) ,
2 2 2 2

20. The volume of 2M H4P2O7 needed to neutralise 2cc 2M Ca(OH)2 is


(a) 2cc (b) 1cc
(c) 4cc (d) 1.5cc

Answers to Assignments
SECTION – I

1. (b) 2. (b) 3. (b) 4. (a)


5. (b) 6. (d) 7. (c) 8. (c)
9. (d) 10. (d) 11. (a) 12. (d)
13. (a) 14. (c) 15. (c) 16. (a)
17. (c) 18. (b) 19. (c) 20. (a)
21. (d) 22. (c) 23. (a) 24. (c)
25. (a)

SECTION – II

1. (d) 2. (d) 3. (c) 4. (c)


5. (c) 6. (b) 7. (c) 8. (b)
9. (a) 10. (b) 11. (a) 12. (a)
13. (b) 14. (c) 15. (d) 16. (c)
17. (a) 18. (b) 19. (c) 20. (b)
21. (d) 22. (b), (c), (d) 23. (a), (b), (d) 24. (a), (c)
25. (a), (b)

SECTION – III

1. (c) 2. (d) 3. (c) 4. (b)


5. (a) 6. (d) 7. (a) 8. (b)
9. (a) 10. (b) 11. (d) 12. (c)
13. (d) 14. (b) 15. (c) 16. (a)
17. (b) 18. (c) 19. (b) 20. (b)
[Type text]

ATOMIC STRUCTURE

Dalton’s Theory of Atom


John Dalton developed his atomic theory. According to this theory the Atom is considered to be
hard, dense and smallest particle of matter, which is indivisible, the atoms belonging to a
particular element, is unique. The properties of elements differ because of the uniqueness of the
atoms belonging to particular elements. This theory provides a satisfactory basis for the laws of
chemical combination. The atom can neither be created nor destroyed i.e., it is indestructible.
Drawbacks: It fails to explain why atoms of different kinds should differ in mass and
valency etc.

The discovery of isotopes and isobars showed that atoms of same elements may have different
atomic masses (isotopes) and atoms of different kinds may have same atomic masses (isobars).

Sub-Atomic Particles

The electrically neutral charged particle, neutron was discovered by James Chadwick by
bombarding boron or beryllium with α -particles.
[Type text]

9
4 Be + 24 He ++ 
→ 12
6 C + 0n
1

Characteristics of the three fundamental particles are:


Electron Proton Neutron
Symbol e or e –1 P n
Approximate relative mass 1/1836 1 1
Approximate relative charge –1 +1 No charge
Mass in kg 9.109 × 10 –31
1.673 × 10 –27
1.675 × 10 –27
Mass in amu 5.485 × 10 –4 1.007 1.008
Actual charge (coulomb) 1.602 × 10 –19 1.602 × 10 –19 0
Actual charge (e.s.u.) 4.8 × 10 –10
4.8 × 10 –10
0

The atomic mass unit (amu) is 1/12 of the mass of an individual atom of 6 C12 , i.e.,
1.66 ? 10–27 kg.

The neutron and proton have approximately equal masses of 1 amu and the electron is about
1836 times lighter, its mass can sometimes be neglected as an approximation.

The electron and proton have equal, but opposite, electric charge while the neutron is electrically
neutral.

Models of Atom
Thomson's Model: Putting together all the facts known at that time, Thomson assumed that
an atom is a sphere of positive charges uniformly distributed, with the electrons scattered as
points throughout the sphere. This was known as plum-pudding model at that time. However this
idea was dropped due to the success of ?-particle scattering experiments studied by Rutherford
and Mardson.

Rutherford’s Model: α -particle emitted by radioactive substance were shown to be


dipositive Helium ions (He ++ ) having a mass of 4 units and 2 units of positive charge.

Rutherford allowed a narrow beam of α -particles to fall on


a very thin gold foil of thickness of the order of 4 × 10 –4 cm
and determined the subsequent path of these particles with
the help of a zinc sulphide fluorescent screen. The zinc
sulphide screen gives off a visible flash of light when
struck by an α − particle, as ZnS has the remarkable
property of converting kinetic energy of particle into
visible light. [For this experiment, Rutherford specifically
used α − particles because they are relatively heavy
[Type text]

resulting in high momentum].

Observation

i) Majority of the α -particles passed straight through the gold strip with little or no
deflection.

ii) Some -particles are deflected from their path and diverge.

iii) Very few -particles are deflected backwards through angles greater than 90°.

iv) Some were even scattered in the opposite direction at an angle of 180° [Rutherford was
very much surprised by it and remarked that "It was as incredible as if you fired a 15 inch
shell at a piece of tissue paper and it came back and hit you"].

Atomic model: On the basis of the above observation, and having realized that the
rebounding α -particles had met something even more massive than themselves inside the gold
atom, Rutherford proposed an atomic model as follows.

i) All the protons (+ve charge) and the neutrons (neutral charge) i.e. nearly the total mass of
an atom is present in a very small region at the centre of the atom. The atom's central core
is called nucleus.

ii) The size of the nucleus is very small in comparison to the size of the atom. Diameter of
the nucleus is about 10 –13 cm while the atom has a diameter of the order 10–8 cm. So, the
size of atom is 105 times more than that of nucleus. (volume of Atom = 1015 Nuclear
volume)

iii) Most of the space outside the nucleus is empty.

iv) The electrons, equal in number to the net nuclear positive charge, revolve around the
nucleus with high speed in various circular orbits.

v) The centrifugal force arising due to the high speed of an electron balances the columbic
force of attraction of the nucleus and the electron remains stable in its path. Thus
according to him atom consists of two parts (a) nucleus and (b) extra nuclear part.

Defects of Rutherford's atomic model

1. Position of electrons:The exact positions of the electrons from the nucleus are not
mentioned.

2. Stability of the atom: Neils Bohr pointed out that Rutherford's atom should be
highly unstable. According to the law of electro-dynamics, the electron should therefore,
continuously emit radiation and lose energy. As a result of this a moving electron will
[Type text]

come closer and closer to the nucleus and after passing through a spiral path, it should
ultimately fall into the nucleus.

It was calculated that the electron should fall into the nucleus in less than 10–8 sec. But it
is known that electrons keep moving outsided the nucleus.

Atomic Spectrum
If the atom gains energy the electron passes from a lower energy level to a higher energy level,
energy is absorbed that means a specific wave length is absorbed. Consequently, a dark line will
appear in the spectrum. This dark line constitutes the absorption spectrum.

Hydrogen spectrum:
1 1 1
=ν=R 2 – 2
λ  n1 n 2 
ν = wave number
λ = wave length
R = Rydberg constant (109678 cm –1 ) (R = 1.097 ? 107 m–1)
n1 and n 2 have integral values as follows
Series n1 n2 Main spectral lines
Lyman 1 2, 3, 4, etc Ultra-violet
Balmer 2 3, 4, 5 etc Visible
Paschen 3 4, 5, 6 etc Infra-red
Brackett 4 5, 6, 7 etc Infra-red
Pfund 5 6, 7, 8, etc Infra-red
Note: All lines in the visible region are of Balmer series but reverse is not true, i.e., all Balmer lines will not fall
in visible region

The pattern of lines in atomic spectrum is characteristic of hydrogen.

Types of emission spectra

i) Continuous spectra:When white light from any source such as sun or bulb is
analysed by passing through a prism, it splits up into seven different wide bands of colour
from violet to red (like rainbow). These colour also continuous that each of them merges
into the next. Hence the spectrum is called as continuous spectrum.

ii) Line spectra:When an electric discharge is passed through a gas at low pressure light
is emitted. If this light is resolved by a spectroscope, it is found that some isolated
coloured lines are obtained on a photographic plate separated from each other by dark
spaces. This spectrum is called line spectrum. Each line in the spectrum corresponds to a
particular wavelength. Each element gives its own characteristic spectrum.
[Type text]

Planck’s Quantum Theory


When a black body is heated, it emits thermal radiations of different wavelengths or frequency.
To explain these radiations, Max Planck put forward a theory known as Planck’s quantum
theory. The main points of quantum theory are:

i) Substances radiate or absorb energy discontinuously in the form of small packets or


bundles of energy.

ii) The smallest packet of energy is called quantum. In case of light the quantum is known as
photon.

iii) The energy of a quantum is directly proportional to the frequency of the radiation. E ??
(or E = h? where v is the frequency of radiation and h is Planck’s constant having the
value 6.626 × 10 –27 erg-sec or 6.626 × 10 –34 J-sec.

iv) A body can radiate or absorb energy in whole number multiples of a quantum hv, 2hv,
3hv …. nh? where n is the positive integer.

Bohr’s Atomic Model


Bohr developed a model for hydrogen and hydrogen like one-electron species (hydrogenic
species). He applied quantum theory in considering the energy of an electron bond to the
nucleus.

Important postulates: An atom consists of a dense nucleus situated at the center with the
electron revolving around it in circular orbits without emitting any energy. The force of
attraction between the nucleus and an electron is equal to the centrifugal force of the moving
electron.

Of the finite number of circular orbits possible around the nucleus, and electron can revolve only
in those orbits whose angular momentum (mvr) is an integral multiple of factor h/ 2π .

nh
mvr =

where, m = mass of the electron
v = velocity of the electron
n = orbit number in which electron is present
r = radius of the orbit

As long as an electron is revolving in an orbit it neither loses nor gains energy. Hence these
orbits are called stationary states. Each stationary state is associated with a definite amount of
[Type text]

energy and it is also known as energy levels. The greater the distance of the energy level from
the nucleus, the more is the energy associated with it. The different energy levels are numbered
as 1, 2, 3, 4, (from nucleus onwards) or K, L, M,N etc.

Ordinarily an electron continues to move in a particular stationary state without losing energy.
Such a stable state of the atom is called as ground state or normal state.

If energy is supplied to an electron, it may jump (excite) instantaneously from lower energy (say
1) to higher energy level (say 2, 3, 4, etc) by absorbing one quantum of energy. This new state of
electron is called as excited state. The quantum of energy absorbed is equal to the difference in
energies of the two concerned levels.

Since the excited state is less stable, atom will lose it’s energy and come back to the ground state.
Energy absorbed or released in an electron jump, ( ∆ E) is given by

∆E = E 2 – E1 = hv

Where E 2 and E1 are the energies of the electron in the first and second energy levels, and v is
the frequency of radiation absorbed or emitted.

Note: If the energy supplied to hydrogen atom is less than 13.6 eV, it will accept or absorb only those quanta
which can take it to a certain higher energy level i.e., all those photons having energy less than or more
than a particular energy level will not be absorbed by hydrogen atom. But if energy supplied to hydrogen
atom is more than 13.6 eV then all photons are absorbed and excess energy appear as kinetic energy of
emitted photo electron.

Limitations of Bohr’s Theory


i) It does not explain the spectra of atoms or ions having more than one electron.

ii) Bohr’s atomic model failed to account for the effect of magnetic field (Zeeman effect) or
electric field (Stark effect) on the spectra of atoms or ions. It was observed that when the
source of a spectrum is placed in a strong magnetic or electric field, each spectral line
further splits into a number of lines. This observation could not be explained on the basis
of Bohr’s model.

iii) de-Broglie suggested that electrons like light have dual character. It has particle and wave
character. Bohr treated the electron only as particle.

iv) Another objection to Bohr’s theory came from Heisenberg’s Uncertainty Principle.
According to this principle “it is impossible to determine simultaneously the exact
position and momentum of a small moving particle like an electron”. The postulate of
Bohr, that electrons revolve in well defined orbits around the nucleus with well defined
velocities is thus not attainable.
[Type text]

By Bohr’s theory

i) Radius of hydrogen atom


n2 n2
rn = r0 = 0.529 × Å
z z

(ii) Energy level of Hydrogen atom: The total energy, E of the electron is the sum of
kinetic energy and potential energy. Kinetic energy of the electron = 1 2 mv 2
–KZe 2
Potential energy =
r
KZe 2
Total energy = 1 2 mv 2 – … (4)
r
KZe2 KZe2 KZe2
Total energy (e) = – =–
2r r 2r
1
KE = – PE, KE = –TE
2
Substituting for r, gives us
2π2 mZ 2 e 4 K 2
E= where n = 1, 2, 3, …
n 2h 2

Z2
E = –21.8 ×10–12 × erg per atom.
n2
Z2 Z2
= – 21.8 × 10 –19 × 2 J per atom = – 13.6 × 2 eV per atom
n n
z2
E n = –13.6 2 eV per atom
n
(1eV = 3.83 ×10–23 Kcal)
(1eV = 1.602 × 10 –12 erg)
(1eV = 1.602 × 10 –19 J)
Z2
E = –313.6 × kcal/mole (1 cal = 4.18 J)
n2

iii) Velocity of electron

Z
v = 2.18 × 108 cm / sec.
n
[Type text]

Further application of Bohr’s work was made, to other one electron species (Hydrogenic
ion) such as He+ and Li 2+ . In each case of this kind, Bohr’s prediction of the spectrum
was correct.

iv) Explanation for hydrogen spectrum by Bohr’s theory: According to the


Bohr’s theory electron neither emits nor absorbs energy as long as it stays in a particular
orbit. However, when an atom is subjected to electric discharge or high temperature, and
electron in the atom may jump from the normal energy level, i.e., ground state to some
higher energy level i.e., exited state. Since the life time of the electron in excited state is
short, it returns to the ground state in one or more jumps.

During each jumps, energy is emitted in the form of a photon of light of definite
wavelength or frequency. The frequency of the photon of light thus emitted depends upon
the energy difference of the two energy levels concerned ( n1 , n 2 ) and is given by

–2π2 mZ2e4 K 2 1 1
hv = E 2 – E1 =  2 – 2
h2  n1 n 2 
2π2 mZ2e4 K 2 1 1
v=  2 – 2
h3  n1 n 2 

The frequencies of the spectral lines calculated with the help of above equation are found
to be in good agreement with the experimental values. Thus, Bohr’s theory elegantly
explains the line spectrum of hydrogen and hydrogenic species.

Bohr had calculated Rydberg constant from the above equation.

C 2π2 mZ2 e4 K 2 1 1
ν= =  2 – 2
λ h3  n1 n 2 
1 2π2 mZ2 e4 K 2  1 1
=ν=  2 – 2
λ 3
hc  n1 n 2 
2π2 me4 K 2
where = 1.097 ×10 –7 m –1 or 109678 cm –1
h 3c
1 1 1
i.e. Rydberg constant (R) ∴ ν = = RZ2  2 – 2 
λ  n1 n 2 
ν = wave number.
[Type text]

Electromagnetic Energy
Newton was first person to comment on the nature of light in terms of Corpuscular. Theory of
Light. According to this theory light is a stream of particles commonly known as corpuscles of
light. He was able to explain reflection and refraction, the most common phenomenon of light.
But the other phenomenon like diffraction and interference could not be explained on the basis of
this theory.

Maxwell, in 1956 proposed that radiant energy (light) has wave characteristics. Light according
to him is Electromagnetic Wave arising due to the disturbance created by electric and magnetic
fields oscillating perpendicular to each other in space. Like all other mechanical waves, it is
characterised by velocity c, frequency ν , wavelength λ which are related as :
c = νλ . The value of c is constant and equal to 3 × 10 8 m / s.

Electromagnetic Spectrum
Electromagnetic wave or radiation is not a single wavelength radiation, but a mixture of various
wavelength or frequencies. All the frequencies have same speed.

If all the components of Electromagnetic Radiation (EMR) are arranged in order of decreasing or
increasing wavelengths or frequencies, the pattern obtained is known as Electromagnetic
Spectrum. The following table shows all the components of light.
S.No. Name Wavelength Frequency(Hz) Source
1. Radio wave 3 × 10 – 3 × 10
14 7
1 × 10 – 1 × 10
5 9
Alternating current
of high frequency
2. Microwave 3 × 10 7 – 6 × 10 6 1 × 109 – 5 × 1011 Klystron tube
3. Infrared (IR) 6 × 10 6 – 7600 5 × 1011 – 3.95 × 1016 Incandescent
objects
4. Visible 7600–3800 3.95 × 1016 – 7.9 × 1014 Electric bulbs, sun
rays
5. Ultraviolet(UV) 3800–150 7.9 × 1014 – 2 × 1016 Sun rays, arc
lamps with
mercury vapours
6. X-Rays 150–0.1 2 × 1016 – 3 × 1019 Cathode rays
striking metal plate
7. γ -Rays 0.1–0.01 3 × 1019 – 3 × 10 20 Secondary effect
of radioactive
decay
8. Cosmic Rays 0.01–zero 3 × 10 20 –Infinity Outer space
Continuous Spectrum: When sunlight (white light) is passed through a prism, it is
dispersed or resolved into a continuous spectra of colours. It extends from RED (7600 Å) at one
end to the VIOLET (3800Å) at other end. In this region, all the intermediate frequencies between
[Type text]

red and violet are present. The type of spectrum is known as Continuous Spectrum., Hence
continuous spectra is one which contains radiation of all the frequencies.

Discontinuous Spectrum: Light emitted from atoms heated in a flame or excited


electrically in gas discharge tube, does not contain a continuous spectrum of wavelengths (or
frequencies). It contains only certain well-defined wavelength (or frequencies). The spectrum
pattern appears as a series of bright lines (separated by gaps of darkness) and hence called as
Line-Spectrum.

One notable feature observed is, that each element emits a characteristic spectrum, suggesting
that there is discrete relation between the spectrum characteristics and the internal atomic
structure of an atom.

Photoelectric Effect
According to Einstein, when a quantum of light (photon) strikes a metal surface, it imparts its
energy to the electrons in the metal. In order for an electron to escape from the surface of the
metal, it must overcomes the attractive force of the positive ions in the metal. So a part of the
photon's energy is absorbed by the metal surface to release the electron, this is known as work
function of the surface and is denoted by φ . The remaining part of the energy of the photon goes
into the kinetic energy of the electron emitted. If E is the energy of the photon, KE is the kinetic
energy of the electron and φ be the work function of the metal then we have;
φ = hν 0 and E i = hν
⇒ KE = E i – φ ⇒ KE = hν – hν 0 = h(ν – ν 0 )
Also, if m be the mass and v be the velocity of the electron ejected then
KE = 1 2 mv 2 = h(ν – ν 0 ) .

Note: The electromagnetic Radiation (or wave) now emerges as an entity which shows dual nature i.e.,
sometimes as Wave and sometimes as Particle (quantum aspect).

ν 0 = 4 × 1015 Hz

Quantum Mechanical Model of Atom


The atomic model which is based on the particle and wave nature of the electron is known as
wave or quantum mechanical model of the atom. This was developed by Schrodinger in 1926.
This model describes the electron as a three dimensional wave in the electronic field of
positively charged nucleus. Schrodinger derived an equation which describes wave motion of
positively charged nucleus. Schrodinger derived an equation which describes wave motion of an
electron. The differential equation is
[Type text]

d 2ψ d 2ψ d 2ψ 8π2 m
+ + + 2 (E – V)ψ = 0
dx 2 dy 2 dz 2 h

where x, y, z are certain coordinates of the electron, m = mass of the electron, E = total energy of
the electron. V = potential energy of the electron; h = Planck's constant and ψ (psi)= wave
function of the electron.

Significance of ?: The wave function may be regarded as the amplitude function expressed
in terms of coordinates x, y and z. The wave function may have positive or negative values
depending upon the value of coordinates. The main aim of Schrodinger equation is to give
solution for probability approach. When the equation is solved, it is observed that for some
regions of space the value of ψ is negative. But the probability must be always positive and
cannot be negative, it is thus, proper to use ψ 2 in favour of ψ .

Significance of ?2:? ?2is a probability factor. It describes the probability of finding an


electron within a small space. The space in which there is maximum probability of finding an
electron is termed as orbital. The important point of the solution of the wave equation is that it
provides a set of numbers called quantum numbers which describe energies of the electron in
atoms, information about the shapes and orientations of the most probable distribution of
electrons around nucleus.

Quantum Numbers
To understand the concept of Quantum Numbers, we must know the meaning of some terms
clearly so as to avoid any confusion.

Energy Level: The non-radiating energy circular paths around the nucleus are called as
Energy Levels or Shells. These are specified by numbers having values 1, 2, 3, 4, ... or K, L, M,
N, ... in order of increasing energies. The energy of a particular energy level is fixed.

Sub-Energy Level: The phenomenon of splitting of spectral lines in electric and magnetic
fields reveals that there must be extra energy levels within a definite energy level. These were
called as Sub-Energy Levels or Sub-Shells. There are four types of sub-shells namely; s, p, d, f.

First energy level (K or ) has one sub-shell designated as 1s, the second energy level (L or 2) has
two sub-shell as 2s & 2p, the third energy level (M or 3) has three sub shell as 3s, 3p and 3d, and
the fourth energy level (N or 4) has four sub-shells as 4s, 4p, 4d and 4f. The energy of sub-shell
increases roughly in the order: s < p < d <f.
[Type text]

Orbital: Each sub-energy level (sub-shell) is composed of one or more orbitals. These orbitals
belonging to a particular sub-shell have equal energies and are called as degenerate orbitals. s-
sub-shell has one orbital, p has three orbitals, d have five orbitals and f has seven orbitals.

To describe or to characterize the electrons around the nucleus in an atom, a set of four numbers
is used, called as Quantum Numbers. These are specified such that the states available to the
electrons should follows the laws of quantum mechanics or wave mechanics.

Principal Quantum Number: (n): This quantum number represents the main energy
levels (principal energy levels) designated as n = 1, 2, 3, ... or the corresponding shells are named
as K, L, M, N, ... respectively. It gives an idea of position and energy of an electron. The energy
level n = 1 corresponds to minimum energy and subsequently n = 2, 3, 4, ..., are arranged in
order of increasing energy.

Higher is the value of n, greater is its distance from the nucleus, greater is its size and also
greater is its energy.

It also gives the total electrons that may be accommodated in each shell, the capacity of each
shell is given by the formula 2n 2 , where n : principal quantum number.

Azimuthal Quantum Number: (l): This number determines the energy associated with
the angular momentum of the electron about the nucleus. It is also called as the angular
momentum quantum number. It accounts for the appearance of groups of closely packed spectral
lines in electric field.

It can assume all integral values from 0 to n–1. The possible values of l are :

0, 1, 2, 3, ..., n–1.

Each value of l describes a particular sub-shell in the main energy level and determines the shape
of the electron cloud.

When n = 1, l = 0, i.e., its energy level contains one sub-shell which is called as a s-sub-shell. So
for l = 0, the corresponding sub-shell is a s-sub-shell. Similarly when l = 1, 2, 3, the sub-shells
are called p, d, f sub-shells respectively.

As you know for n = 1, l = 0, there is only one sub-shell. It is represented by 1s. Now for n = 2, l
can take two values (the total number of values taken by l is equal to the value of n in a particular
energy level). The possible values of l are 0, 1. The two sub-shell representing the IInd energy
level are 2s, 2p. In the same manner, for n = 3, three sub-shells are designated as 3s, 3p, 3d
corresponding to l = 0, 1, 2, and for n = 4, four sub-shells are designated as 4s, 4p, 4d, 4f
corresponding to l = 0, 1, 2, 3.
h
The orbital Angular momentum of electron = l(l + 1) .

Note that its value does not depend upon value of n.
[Type text]

Magnetic Quantum Number (m): An electron with angular momentum can be thought as
an electric current circulating in a loop. A magnetic field due to this current is observed. This
induced magnetism is determined by the magnetic quantum number. Under the influence of
magnetic field, the electrons in a given sub-energy level prefer to orient themselves in certain
specific regions in space around the nucleus. The number of possible orientations for a sub-
energy level is determined by possible values of m corresponds to the number of orbitals in a
given sub-energy level).

m can have any integral values between –l to +l including 0, i.e., m = –l, 0 +, l, …, 0, 1, 2, 3, 4,


. . ., l–1 + l. We can say that a total of (2l + 1) values of m are there for a given value of l– 2, –1,
0, 1, 2, 3.

In s sub-shell there is only one orbital [l = 0, ⇒ m = (2l +1) = 1].

In p sub-shell there are three orbitals corresponding to three values of m : –1, 0 +1. [l = 1
⇒ m = (2l +1) = 3]. These three orbitals are represented as p x , p y , p z along X, Y, Z axes
perpendicular to each other.

In d sub-shell, there are five orbitals corresponding to –2, –1, 0 +1, +2, [l = 2 ⇒
m = ( 2 × 2 + 1) = 5] . These five orbitals are represented as d xy , d zx , d x – y , dz 2 .
2 2

In f sub-shell there are seven orbitals corresponding to –3, –2, –1, 0, +1, +2, +3 [l = 3 ⇒
m = (2 × 3 + 1) = 7] .

Spin quantum Number (s): When an electron rotates around a nucleus it also spins about
its axis. If the spin is clockwise, its spin quantum number is +1/2 and is represented as ↑ . If the
spin is anti-clockwise, its value is –1/2 and is represented as ↓ . If the value of s is +1/2, then by
convention, we take that electron as the first electron in that orbital and if the value of s is –1/2, it
is taken as second electron.

Shapes of Atomic Orbitals


i) S-orbital: An electron is considered to be immersed out in the form of a cloud. The shape
of the cloud is the shape of the orbital. The cloud is not uniform but denser in the region
where the probability of finding the electron is maximum.

The orbital with the lowest energy is the 1s orbital. It is a sphere with its center of the
nucleus of the atom. The s-orbital is said to spherically symmetrical about the nucleus, so
that the electronic charge is not concentrated in any particular direction. 2s orbital is also
spherically symmetrical about the nucleus, but it is larger than (i.e., away from) the 1s
orbit.
[Type text]

1s 2s
nucleus x

Z radial node

ii) p-orbitals: There are three p-orbitals: p x , p y and p z . they are dumb-bell shaped, the two
levels being separated by; a nodal plane, i.e., a plane where there is no likely hood of
finding the electron. The p-orbitals have a marked direction character, depending as
whether p x , p y and p z orbital is being considered. The p-orbitals consist of two lobes with
the atomic nucleus lying between them. The axis of each p-orbital is perpendicular to the
other two. The p x , p y and p z orbitals are equivalent except for their directional property.
They have same energy; orbitals having the same energy are said to be degenerated.
z z
z

y y y

x x x

px py pz

iii) d-orbitals: There are five d-orbitals. The shapes of four d-orbitals resemble four leaf
cloves. The fifth d-orbital looks different. the shapes of these orbitals are given below.
y z z

x y x

dxy dyz dzx


z
y

d 2 2 d z2
x −y

Dual Character
In case of light some phenomenon like diffraction and interference can be explained on the basis
of its wave character. However, the certain other phenomenon such as black body radiation and
photoelectric effect can be explained only on the basis of its particles nature. Thus, light is said
to have a dual character. Such studies on light were made by Einstein in 1905.
[Type text]

Louis de-Broglie, in 1942 extended the ideal of photons to material particles such as electron and
he proposed that matter also has a dual character-as wave and as particle.
h
λ = h / mv (or) λ =
p
where mv = p is the momentum of the particle.

Heisenberg’s Uncertainty Principle


It is impossible to measure simultaneously the position and momentum of a small microscopic
moving particle with absolute accuracy or certainty i.e., if an attempt is made to measure any one
of these two quantities with higher accuracy, the other becomes less accurate.

h
∆x.∆p ≥ h / 4π or ∆E × ∆t ≥

Electronic Configuration of Elements: Quantum numbers can now characterize the


electrons in an atom. To describe the arrangements and distribution of electrons for different
elements, following rules an selective principles are used. The distributions of electrons in an
atom is known as the electronic configuration of that element.

Aufbau Principle: An atom in its lowest state of energy is said to be in ground state. The
ground state is the most stable in an atom. According to Aufbau principle.
“Electrons are added progressively to the various orbitals in their order of increasing energy
starting with the orbital of lowest energy”.
The order of increasing energy may be summed up as follows
1s, 2s, 2p, 3s, 4s, 3d, 4p, 5s, 4d, 5p, 6s, 4f, 5d, 6p, 7s, 5f, 6d, …
As a working rule, a new electron enters an empty orbital for which the value of (n + l) is
minimum. If the value (n + l) is same for two or more orbitals, the new electron enters an orbital
having lower value of n.

Exceptions to Aufbau Principle: In some cases it is seen that actual electronic


arrangement is slightly different from arrangement given by Aufbau principle. A simple reason
behind this is that half-filled and full-filled sub-shell have got extra stability.

Cr(24) 
→ 1s 2 , 2s 2 2p6 , 3s 2 3p6 3d 4 , 4s 2 (wrong)
[Type text]

→ 1s 23 , 2s 2 2p6 ,3s 2 3p6 3d 5 , 4s1 (right)


Cu(29)  → 1s 2 , 2s 2 2p 6 , 3s 2 3p6 3d 9 , 4s 2 (wrong)
→ 1s 2 , 2s 2 2p6 , 3s 2 3p6 3d10 , 4s1 (right)

Similarly the following elements have slightly different configurations than expected.

Nb → [Kr] 4d 4 5s1


Mo → [Kr] 4d 4 5s1
Ru → [Kr] 4d 7 5s1
Rh → [Kr] 4d8 5s1
Pd 
→ [Kr] 4d10 5s 0
Ag → [Kr] 4d10 5s1
Pt 
→ [Xe] 4f 14 5d 9 6s1
Au → [Xe] 4f 14 5d10 6s1

Pauli’s Exclusion Principle: According to this principle

No two electrons in an atom can have the same set of all the quantum numbers or one can say
that no two electrons can have the same quantised states.

Hund’s Rule of maximum Multiplicity

“Electrons never pair until no available empty degenerate orbitals are left to him.”

By doing this, the electrons stay as far away from each other as possible. This is highly
reasonable if we consider the electron-electron repulsion. Hence electrons obey Hund’s rule as it
results in lower energy state and hence more stability.

Node and Nodal Plane


Node is defined as a region where the probability of finding an electron is zero.
The planes passing through the angular nodal points are called nodal planes.
Nodes
No. of radial or spherical nodes = n – l – 1
No. of angular nodes or non spherical nodes = l, Total no. of nodes = n–1

Solved Problems
LEVEL I
[Type text]

Problem 1: Calculate the ratio of the radius of Li+2 ion in 3rd energy level to that of He+
ion in 2nd energy level.
n2
Solution: r∝
z
2
r1  n1   z 2 
=   
r2  n 2   z1 
n1 = 3
n2 = 2
z1 = 3 (for Li 2+ )
z2 = 2 (for He+ )
2
r1  3   2  3
=  ×  =
r2  2   3  2

Problem 2: How far from the nucleus is the electron in a hydrogen atom if it has energy
of –0.850 eV.

Z2
Solution: E n = – 2 × 21.69 × 10 –19
n
Z2
– 2
× 21.69 ×10 –19 = –0.85 × 1.6 × 10 –19
n
n = 4 (Z = 1)
n2 16
r = × 0.529 = × 0.529 = 8.46Å
Z 1

Problem 3: Find the wavelengths of the first line of He+ ion spectral series whose
1 1
interval between extreme lines is – = 2.7451×104 cm –1
λ1 λ2

Solution: Extreme lines means first and last.


1 1 1 1  1 1  RZ2
– = RZ2  2 – 2  – RZ
2
 2– 2
=
λ λ2  n1 ∞   n1 (n1 + 1)  (n1 + 1)
2

109677.76 × 22
2.7451×104 =
(n1 + 1)2
(n1 + 1) = 4
n1 = 3
Wavelength of first line,
[Type text]

1 1 1
= 109677.76 × 22 ×  2 – 2 
λ 3 4 
λ = 4689 ×10 –8 cm = 4689Å

Problem 4: In a photoelectric experiment, the collector plate is at 2.0 V with respect to


emitter plate made of copper (work function 4.5 eV). The emitter is
illuminated by a source of mono-chromatic light of wavelength 200 nm.
Find the minimum and maximum kinetic energy of photoelectrons reaching
the collector.
Solution: Since plate potentials 2 V, minimum K.E. will be 2 eV. For max. K.E. use the
following relation:
Absorbed energy = Threshold energy + K.E.
hc
= 4.5 × 1.6 × 10 –19 + K.E.
λ
6.626 ×10 –34 × 3 × 108
= 4.5 ×1.6 ×10 –19 + K.E.
200 ×10 –9

K.E. = 2.739 ×10–19 J = 1.7 eV


Max K.E. = 2eV + 1.7 eV = 3.7 eV.

Problem 5: Write down the values of quantum numbers of all the electrons present in
the outermost orbit of argon (At. No. 18)

Solution: The electronic configuration of argon is


1s 2 , 2s 2 2p 6 ,3s 2 3p 2x 3p 2y 3p z2
Values of quantum numbers are:
n ? m s
3s 2 3 0 0 +1/2, –1/2
3p 2x 3 1 ±1 +1/2, –1/2
3p 2y 3 1 ±1 +1/2, –1/2
3p 2z 3 1 0 +1/2, –1/2

Problem 6: a) An electron is in 5f-orbital. What possible values of quantum numbers


n, l, m and s can it have?
b) What designation is given to an orbital having (i) n = 2, l =1, and (ii) n =
3, l = 0?

Solution: a) For an electron in 5f-orbital, quantum number are:


n = 5, ? = 0, 2, 3, 4, ? = 0, m = 0, ? = 1, m = –1, 0, +1, ? = 2, m = –2, –1,
0, +1, +2, ? = 3, m = –3, –2, –1, 0, +1, +2, +3, ? = 4, m = –4, to +4
including 0
[Type text]

b) (i) 2p, (ii)3s

Problem 7: 13.6 eV is needed for ionization of a hydrogen atom. An electron in a


hydrogen atom in its ground state absorbs 1.50 times as much energy as the
minimum required for it to escape from the atom. What is the wavelength of
the emitted electron? ( me = 9.109 × 10–31 kg, e = 1.602 × 10–19 coulomb, h
= 6.63 × 10–34 J.s)

Solution: 1.5 times of 13.6 eV i.e., 20.4 eV is absorbed by the hydrogen atom out of
which 6.8 eV (20.4 –13.6) is converted to kinetic energy. KE = 6.8 eV = 6.8
(1.602 × 10 –19 coulomb) (1 volt) = 1.09 × 10 –18 J.

1
Now, KE = mv 2
2

Problem 8: If a 1 g body is traveling along the x-axis with an uncertainty in velocity of 1


cm/s, what is theoretical uncertainty in its position? (b) If an electron is
traveling with uncertainty in velocity of 1 m/s, what is the theoretical
uncertainty in its position?
Solution: a) According the uncertainty
h
∆x × m∆v =

h 6.628 ×10 –27 ergs sec.
∆x = = = 3 × 10 –28 cm = 3 ×10 –30 m
4πm∆v 4 × 3.14 ×1g ×1 cms –1
6.626 ×10 –34 Js
b) ∆X =
4 × 3.14 × 9.1×10031 kg ×1 ms1
3 ×10 –5 m = 30 µ m
LEVELII
Objective

Problem 1: Bohr’s atomic model can explain


(a) the spectrum of hydrogen atom only
(b) the spectrum of an atom or ion containing one electron only
(c) the spectrum of hydrogen molecule
(d) the solar spectrum

Solution: In Bohr’s theory while calculating the energy of electron, the potential energy
has been found out by considering only the attraction between the electron and
nucleus. If there is another electron in the orbit, the potential energy would
change due electron-electron repulsion. Therefore the Bohr’s model is meant
for all one-electron systems.
∴ (b)
[Type text]

Problem 2: The orbital angular momentum of an electron in 2s orbital is


(a) 4 (b) 1
h
(c) zero (d)

h
Solution: The orbital angular momentum of an electron is calculated as l(l + 1) ,

h
0(0 + 1) =0

∴ (c)

Problem 3: The ratio of the energies of photons of 2000Å to that of 4000Å is


(a) 2 (b) 4
1 1
(c) (d)
2 4

hc E λ 4000
Solution: E= ,∴ 1 = 2 = =2
λ E 2 λ1 2000
? (a)

Problem 4: Which of the following postulates does not belong to Bohr’s model of the
atom?
h
(a) Angular momentum is an integral multiple of

(b) The electron stationed in the orbit is stable
(c) The path of an electron is circular
(d) The change in the energy levels of electron is continuous

Solution: In Bohr’s model, the energy are discrete and not continuous
∴ (d)

Problem 5: The wave number of the first Balmer line of Li 2+ ion is 1,36,800 cm –1 the
wave number of the first line of Balmer series of hydrogen atom is
(in cm –1 )
(a) 68,400 (b) 15,200
(c) 76,000 (d) 30,800.

Solution: Atomic number of Li 2+ is 3


1 1 1 1 1
= R H Z2  2 – 2  , 1,36,800 = R H × 9  2 – 2 
λ  n1 n 2  2 3 
 1 1  1,36,800
RH  2 – 2  = = 15, 200
2 3  9
∴ (b)
[Type text]

Problem 6: If uncertainty in the position of an electron is zero the uncertainty in its


momentum will be
h h
(a) < (b) >
4π 4π
(c) zero (d) infinite

Solution: ∆ x × ∆ p ≥ h/4?
if ∆x = 0 , then ∆P will be infinite
∴ (d)

Problem 7: The principle quantum number represents


(a) shape of an orbital
(b) number of electrons in an orbit
(c) distance of an electron from the nucleus
(d) orientation of orbitals in space.
Solution: (c)
Problem 8: The energy of an electron of 2py orbital is
(a) greater than 2p orbital
(b) less than 2px orbital
(c) equal to 2s orbital
(d) same as that of 2px and 2pz orbitals.

Solution: All the 2p orbitals are degenerate


∴ (d)

Problem 9: If the following matter waves travel with equal velocity, the longest
wavelength is that of a/an.
(a) electron (b) proton
(c) neutron (d) α − particle

h
Solution: λ=
mv
λ will be large if m is small
∴ (a)

Problem 10: Number of nodal planes (planes of zero electron density) in the d xy
orbital is
(a) 1 (b) 2
(c) 0 (d) 4

Solution: xz and yz are planes with zero electron density for d xy orbital.
∴ (b)

Problem 11: If the speed of electron in the Bohr’s first orbit of hydrogen atom is x the
speed of the electron in the third Bohr’s orbit is
[Type text]

(a) x/9 (b) x/3


(c) 3x (d) 9x
Solution: According to Bohr’s model for hydrogen and hydrogen like atoms the velocity
2π k ze2
of an electron in an atom is quantized and is given by v =
nh
∴ (b)

Problem 11: Which set of quantum number is not consistent with the quantum
mechanical theory?
(a) n = 2, l = 1, m = 1, s = ½ (b) n = 4, l = 3, m = 2, s = –1/2
(c) n = 3, l = 2, m = 3, s = +1/2 (d) n = 4, l = 3, m = 3, s = +1/2.

Solution: The e is a well defined co-relation between n, l, m and s quantum numbers in


an atom. l = (n – 1), m will be ± ½, for n = 3 there cannot be m = 3. Highest
value m will be 2.
∴ (c)

Problem 13: If the radius of first Bohr orbit be a 0 , then the radius of third Bohr orbit
would be
(a) 3 ? a0 (b) 6 ? a0
(c) 9 ? a0 (d) 1/9? a0

Solution: The radius as per Bohr’s model r ∝ n 2


∴ (c)

Problem 14: The quantum numbers of most energetic electron in Ne atom when it is in
first excited state is
(a) 2, 1, 0, ± 1/2 (b) 3, 1, 1, ± ½
(c) 3, 0, 0, ± 1/2 (d) 3, 1, 0, ± 1/2.

Solution: The electronic configuration of Ne atom is 1s 2 2s 2 2p 6 . In its first excitation


state the electron from 2p will unpair and shift to 3s where its quantum
numbers will be 3, 0, 0, ± ½.
∴ (c)

Problem 15: Of the following transitions in hydrogen atom, the one which gives an
absorption line of lowest frequency is
(a) n = 1 to n = 2 (b) n = 3 to n = 8
(c) n = 2 to n = 1 (d) n = 8 to n = 3

Solution: Absorption line in the spectra arise when energy is absorbed i.e., electron
shifts from lower to higher orbit, out of a and b, b will have the lowest
frequency as this falls in the Paschen series.
∴ (b).
[Type text]

Subjective
Problem 1: Which is larger, an He+ ion with an electron in an orbit with n = 3 or Li2+
ion with an electron in an orbit with n = 5?

Solution: Radius of the nth Bohr’s orbit of the species of atomic number Z is given by
n 2a 0
rn =
Z
Where a 0 ( = 0.529Å) is called Bohr’s radius.
9a 25a 0
i) r3 (He + , Z = 2) = 0 = 4.5a 0 ii) r5 (Li 2+ , Z = 3) = = 8.33a 0
2 3
Problem 2: How many elements would be in the second period of the periodic table if
1 1
the spin quantum number ms could have the value – , 0, ?
2 2
Solution: For second period n = 2, hence,
1 m ms
1 1
0 0 + , 0, –
2 2
1 1
1 –1 + , 0, –
2 2
1 1
0 + , 0, –
2 2
1 1
+1 + , 0, –
2 2
Hence, total number of electrons = 12
(= total values of spin quantum number)

Problem 3: Calculate the wavelength of a soft ball of mass of 100 g traveling at a


velocity of 35 m s–1.

Solution: Using de-Broglie’s equation


h 6.626 × 10 –34 J s –1
λ= = = 1.893 × 10 –34 m
mv  100 
 kg × 35 m s
–1

 1000 

Problem 4: O2 undergoes photochemical dissociation into 1 normal oxygen atom (O)


and more energetic oxygen atom O*. If (O*) has 1.967 eV more energy (O)
and normal dissociation energy of O2 is 498 kJ mol–1, what is the maximum
wavelength effective for the photochemical dissociation of O2?

Solution: O 2 → O + O*
498 × 1000
Dissociation energy = 498 kJ mol –1 = J molecule –1
6.02 × 10 23
[Type text]

= 8.27 × 10–19 J molecule –1


Excitation energy to form O*
= 1.967 eV atom –1 = 1.967 × 1.6 × 10 –19 J atom –1
= 3.15 × 10 –19 J atom –1
Total energy = 11.42 × 10 –19 J
hc
E=
λ
hc
λ=
E
6.626 × 10 –34 × 3 × 108
= = 174 × 10–9 m = 174 nm
11.42 × 10 –19

Problem 5: The electron energy in hydrogen atom is given by E = (–21.7 × 10–12)/n2


ergs. Calculate the energy required to remove an electron completely from
the n = 2 orbit. What is the longest wavelength (in cm) of light that can be
used to cause this transition?

 21.7 × 10 –12 
Solution: ∆E = E ∞ – E 2 = [0] –  – 
 4 
21.7 × 10 –12
21.7 × 10 –12
Since E n = – ergs, =
n2 4
∆E = hv
hc
∆E =
λ
21.7 × 10 –12 3 × 1010
= 6.627 × 10 –27 ×
4 λ
whence λ = 3.67 × 10 cm .
–5

Problem 6: The wave number of the first line in the Balmer series of hydrogen is 15200
cm–1. What is the wave number of the first line in the Balmer series of Be3+?
1 1
Solution: ν H = R H  2 – 2  ; Z = 1 for H-atom
2 3 
1 1
ν Be3+ = R H × 16  2 – 2  ; Z = 4 for Be3+
2 3 
Thus, ν Be3+ = 16 ν H = 16 × 152000 cm –1
= 2.432 × 105 cm –1
= 4.16 × 10 –14 .

Problem 7: Two 1.0 g carbon disks 1.00 cm apart have equal and opposite charges. If
force of attraction between them is 1.00 × 10–5N, calculate the ratio of
[Type text]

excess electrons to total atoms on the negatively charged disk. (permittivity)


constant is 9.0 × 109 N m2 C–2).

Fr 2 1.00 ×10 –5 × (1.0 ×10 –2 )2


Solution: q1 = = = 3.33 ×10 –10 C on each disk,
k 9.0 ×10 9

Since charge on one electron = 1.6 × 10 –19 C


3.33 ×10 –10
hence, number of electron on the disk = = 2.08 ×109
1.6 ×10 –19

1
number of atoms in 1.0 g carbon = × 6.02 × 1023 = 5.0 × 1022 atoms
12
2.08 ×109
Hence, ratio of electron to atoms =
5.00 ×1022

Problem 8: When a certain metal was irradiated with light of frequency 1.6 × 1016 Hz,
the photoelectrons emitted had twice the kinetic energy as did
photoelectrons emitted when the same metal was irradiated with light of
frequency 1.0 × 1016 Hz. Calculate v0 (threshold frequency) for the metal.

Solution: KE1 = h(ν1 – ν 0 ) … (i)


KE 2 = h(ν 2 – ν 0 ) … (ii)
Dividing Eqs. (ii) by (i) we have
ν – ν0 1
∴ 2 =
ν1 – ν 0 2
1.0 ×1016 – ν 0 1
=
1.6 ×1016 – ν 0 2
2.0 × 1016 – 2ν 0 = 1.6 × 1016 – ν 0
ν 0 = 4 × 1015 Hz

Problem 9: One mole of He+ ion is excited. Spectral analysis showed the existence of
50% ions in 3rd level, 25% in 2nd level and remaining 25% in ground state.
ionization energy of He+ is 54.4 eV; calculate total energy evolved when all
the ions return to ground state.
N0  1 1  N
Solution: ( ∆E)3→1 = (54.4)  2
– 2  , for 0 ions falling to ground state
2 1 3  2
4 × N0
= 54.4 × eV
9
N0  1 1  N
and ( ∆E)3→1 = (54.4)  1
– 2  , for 0 ions falling to ground state
4 1 2  4
3 × N0
= 54.4 × eV
16
[Type text]

4 3  91
Hence, total energy = 54.4 × N 0  +  = 54.4 × 6.02 × 1023 × eV
 9 16  144
91
= 54.4 × 6.02 ×1023 × × 1.6 × 10 –19 J = 3.31×106 J
144

Problem 10: Consider the following two electronic transition possibilities in a hydrogen
atom as pictured below:
1) The electron drops from third Bohr’s orbit to Second Bohr’s orbit
followed with the next transition from second to first Bohr’s orbit.
2) The electron drops from third Bohr’s orbit to first Bohr’s orbit directly.
Show that:
a) The sum of the energies for the transitions n = 3 to n = 2 and n = 2
to n = 1 is equal to the energy of transition for n = 3 to n = 1.
b) Are wavelengths and frequencies of the emitted spectrum are also
additive in the same ways as their energies are ?

1 1
Solution: a) ∆E = R H  2 – 2 
 n1 n 2 
1 1
for 3 to 2 ∆E 3→ 2 = R H  2 – 2 
2 3 
1 1 
for 2 to 1 ∆E 2→1 = R H  1 – 2 
1 2 
1 1 
for 3 to 1 ∆E 3→1 = R H  2 – 2 
1 3 
It is evident from Eqs. (1), (2) and (3), that
∆E3→1 = ∆E3→2 + ∆E 2→1
b) Also E = hv; thus frequencies are also additive
hc
but E = and thus wavelengths are not additive.
λ
Problem 11: Ultraviolet light of wavelength 800Å and 700Å when allowed to fall on
hydrogen atoms in their ground state is found to liberate electrons with
kinetic energy 1.8 eV and 4.0 eV respectively. Find the value of Planck’s
constant.
c
Solution: KE1 = hν1 – W = h. – W … (i)
λ1
c
KE 2 = hν 2 – W = h. – W … (ii)
λ2
Substracting equation (i) from equation (ii),
 1 1  λ − λ2 
(KE 2 – KE1 ) = hc  –  = hc  1 
 λ 2 λ1   λ1λ 2 
[Type text]

(KE 2 – KE1 ) × λ1λ 2


or h =
c × (λ1 – λ 2 )
(4.0eV –1.8eV) × 800 ×10 –10 × 700 ×10 –10
=
3 ×108 × (800 – 700) ×10 –10
(2.2 ×1.6 ×10 –19 J) × (56 ×10 –16 × m 2 ) 2.2 × 1.6 × 56 ×10 –35
= = J–s
(3 ×108 ms –1 ) × (100 ×10 –10 m) 3
= 6.57 ×10 –34 J – s

Problem 12: The λ of H α line of Balmer series is 6500Å. What is the λ of Hβ line of
Balmer series?

Solution: For H α lines of Balmer series n1 = 2, n 2 = 3


For Hβ line of Balmer series n1 = 2, n 2 = 4
1 1 1
∴ = RH  2 – 2  … (1)
λ Hα 2 3 
1 1 1
and = RH  2 – 2  … (2)
λ Hβ 2 4 
By Eqs. (1) and (2)
1 1
λβ –
∴ = 4 9
λα 1 – 1
4 16
 80  80
∴ λβ = λ α ×   = 6500 × = 4814.8Å
108  108

Problem 13: The IP1 of H is 13.6 eV. It is exposed to electromagnetic waves of 1028Å
and gives out induced radiations. Find the wavelength of these induced
radiations.

Solution: E1 of H atom = –13.6 eV


6.625 ×10 –34 × 3.0 × 108
Energy given to H atom =
1028 ×10 –10
= 1.933 ×10 –18 J = 12.07eV
∴ Energy of H atom after excitation = –13.6 + 12.07
= –1.53 eV
E
∴ E n = 21
n
–13.6
∴ n2 = =9
–1.53
∴ n=3
[Type text]

Thus, electron in H atom is excited to 3rd shell


hc
∴ I induced λ1 =
E 3 – E1
∴ E1 = –13.6 eV; E3 = –1.53 eV
6.625 ×10 –34 × 3.0 ×108
∴ λ1 =
(–1.53 + 13.6) × 1.602) × 10 –19
= 1028 ×10 –10 m
∴ λ = 1028 Å
hc
∴ II induced λ 2 =
(E 2 – E1 )
13.6
∴ E1 = –13.6 eV; E 2 = – eV
4
6.625 ×10 –34 × 3.0 × 108
∴ λ2 =
 13.6 
– + 13.6  × 1.602 × 10 –19
 4 
= 1216 ×10 –10 m = 1216 Å
hc
∴ III induced λ 3 =
E3 – E 2
13.6 13.6
∴ E1 = –13.6eV; E 2 = – eV; E 3 = – eV
4 9
6.625 ×10 –34 × 3.0 ×108
∴ λ3 = = 6568 ×10 –10 m = 6568 Å
 13.6 13.6 
– +  ×1.602 ×10
–19

 9 4 

Problem 14: Calculate the wavelength in angstrom of the photon that is emitted when an
electron in Bohr orbit n = 2 returns to the orbit n = 1 in the hydrogen atom.
The ionization potential of the ground state of hydrogen atom is
2.17 × 10 –11 erg per atom.

Solution: In the ground state of H-atom the solitary electron occupies the first orbit and
hence, the ionization potential of the ground state of the hydrogen atom is the
energy of the electron of the first orbit, with sign reversed.
E1 = –2.17 ×10 –11 erg
E 2.17 ×1011
Thus, E 2 = = –
n2 22
∴ Energy of the radiation emitted,
∆E = E 2 – E1
2.17 ×10–11
=– 2
– (–2.17 ×10 –11 ) = 1.627 ×10 –11 erg
2
[Type text]

hc
We know ∆E = hv =
λ
hc
Thus = 1.627 × 10 –11
λ
6.62 ×10 –27 × 3 ×1010
λ= = 1.22 ×10 –5 cm
1.627 ×10 –11

= 1220 Å (1Å = 10 –8 cm)

Problem 15: Infrared lamps are used in restaurants and cafeterias to keep food warm.
The infrared radiation is strongly absorbed by water raising its temperature
and that of the food in which it is incorporated. How many photons per
second of infrared radiation are produced by an infrared lamp that
consumes energy at the rate of 100 watt (100 J s –1 ) and is 12% efficient in
converting this energy to infrared radiation? Assume that the radiation has
a wavelength of 1500 nm.

Nhc
Solution: E = Nh ν =
λ
N × 6.626 ×10 –34 × 3 ×108
=
1500 ×10 –9
100 × 12
E= watt (Js –1 )
100
12 N × 6.626 × 10 –34 × 3 ×108
∴ 100 × =
100 1500 × 10 –9
∴ N= 9.06 ×106 photons s –1
[Type text]

KVPY QUESTIONS
1. AccordingtoBohr’stheory,theangularmomentumoftheelectroninthegroundstateofthehydrogen
atomis – (2007)
(A)0 (B)1 (C)2 (D)3

2. The set of principal (n), azimuthal (l ) and magnetic (m1) quantum numbers that is not allowed for the
electron in H–atom is (2011)
(A) n = 3, l = 1, m1 = -1 (B) n = 3, l = 0, m1 = 0
(C) n = 2, l = 1, m1 = 0 (D) n = 2, l = 2, m1 = -1

3. A plot of the kinetic energy (1/2 mv ) of ejected electrons as a function of the frequency ( ν ) of
2

incident radiation for four alkali metals (M1, M2, M3, M4) is shown below: (2012)
Kinetic Energy

M1 M2 M3 M4

ν
The alkali metals M1, M2, M3 and M4 are respectively:
(A) Li, Na, K, and Rb (B) Rb, K, Na, and Li
(C) Na, K, Li, and Rb (D) Rb, Li, Na, and K

4. The radii of the first Bohr orbit of H (rH ), He + (rHe


+
) and Li2 + ( rLi2 + ) are in the order (2013)
+
(A) rHe > rH > rLi2 + +
(B) rH > rHe > rLi2 + +
(C) rH > rHe > rLi2 + (D)
+ 2+
r
He > rH > r
Li

+
5. If the radius of the hydrogen atom is 53 pm, the radius of the He ion is closest to (2014)
(A) 108 pm (B) 81 pm (C) 27 pm (D) 13 pm

Answer Key

1:B 2:D 3:B 4:C 5:C


[Type text]

LEVEL1
SECTION – I Single Choice Questions

1. Photoelectric emission is observed from a surface of frequencies ν1 and ν 2 of the


incident radiation (ν1 > ν 2 ) , if the maximum kinetic energies of the photoelectrons. In
two cases are in the ratio of 1 : K then the threshold frequency ν 0 is given by:
ν – ν2 Kν1 – ν 2
(a) 1 (b)
K –1 K –1
Kν 2 – ν 2 ν – ν2
(c) (d) 1 .
K –1 K

2. The radial distribution curve of the orbital with double dumbbell shape in the 4th principle
shell consists of `n’ nodes, n is
(a) 2 (b) 0
(c) 1 (d) 3

3. The probability of finding an electron residing in a Px orbital is zero in the


(a) yz plane (b) xy plane
(c) xz plane (d) y and z directions.

4. The first emission line in the atomic spectrum of hydrogen in the Balmer Series appears
at
9R H 7R H
(a) cm –1 (b) cm –1
400 144
3R H 5R
(c) cm –1 (d) H
cm –1 .
4 36
5. The hydrogen line spectrum provides evidence for the
(a) Heisenberg Uncertainty Principle
(b) wave like properties of light
(c) Diatomic nature of H2
(d) quantized nature of atomic energy states.

6. If λ1 and λ 2 denote the de-Broglie wavelength of two particles with same masses but
charges in the ratio of 1 : 2 after they are accelerated from rest through the same potential
difference, then
(a) λ1 = λ 2 (b) λ1 < λ 2
(c) λ1 > λ 2 (d) None of these.

7. The increasing order of the value of e/m (charge to mass ratio) for electron (e), proton
(p), neutron (n) and alpha particle (α ) is
[Type text]

(a) e < p < n < α (B) n < p < e < α


(c) n < p < α < e (d) n < α < p < e.
8. The energy of the second Bohr orbit in the hydrogen atom is –3.41eV. the energy of the
second Bohr orbit of He+ ion would be
(a) –0.85 eV (b) –13.64 eV
(c) –1.70 eV (d) –6.82 eV

9. The amount of energy required to remove electron from a Li +2 ion in its ground state is
how many times greater than the amount of energy needed to remove the electron from a
H atom in its ground state?
(a) 9 (b) 6
(c) 4 (d) 3.

10. The velocity of electron of H-atom in its ground state is 2.2 × 10 6 ms –1 . The de-Broglie
wavelength of this electron would be
(a) 0.33 nm (b) 23.3 nm
(c) 45.6 nm (d) 100 nm.

11. The M shell of an atom contains


(a) only s-sub-shell (b) both s and p sub-shell
(c) s, p and d sub-shell (d) s, p, d and f sub-shell.

12. Which of the following sets of the four quantum numbers, n, l, m and ms describes one
of the outermost electrons in a ground state magnesium atom?
1 1
(a) 3, 1, 1, (b) 3, 0, 0, –
2 2
1 1
(c) 3, 0, 1, (d) 3, 1, 0, .
2 2

13. Which of the following statements concerning px orbitals is false?


(a) the electron density has two regions of high magnitude pointed in the +x and –x
directions
(b) these two high probability regions are separated by a node passing through the
nucleus
(c) the electrons stay half the time in one of these region and the remaining time in the
other region
(d) none of these.

14. The wavelength associated with a golf ball weighing 200 g and moving at a speed of 5
m/h is of the order
(a) 10 –10 m (b) 10 –20 m
(c) 10 –30 m (d) 10 –40 m .

15. The quantum numbers of +1/2 and –1/2 for the electron spin represent
[Type text]

(a) Rotation of the electron in clockwise and anticlockwise direction respectively


(b) Rotation of the electron in anticlockwise and clockwise direction respectively
(c) Magnetic moment of the electron pointing up and down respectively
(d) Two quantum mechanical spin states which have no classical analogue.
16. What element has a H-like spectrum, whose lines have wavelengths four times shorter
than those of atomic hydrogen?
(a) He (b) He+
(c) H (d) Li+2
17. The atomic spectrum of Li+2 arises due to the transition of an electron from n2 to n1. If
(n1 + n2) = 4 and (n2 – n1) = 2 find the wavelength of the 3rd line of this series in Li+2 ion?
(a) 10.8 nm (b) 1.08 nm
(c) 108 nm (d) 100 nm
18. The second line of Lyman series of H coincides with the 6th line of Paschen series of an
Ionic species X. Find X assuming R to be same for both H and X?
(a) He+ (b) Li+2
(c) Li+ (d) H
19. The ratio of energy of the electron in ground state of hydrogen to the electron in first
excited state of Be+3 is
(a) 4 : 1 (b) 1 : 4
(c) 1 : 8 (d) 8 : 1
20. In an atom two electrons move around the nucleus in circular orbits of radii R and 4R.
The ratio of the time taken by them to complete one revolution is
(a) 1 : 4 (b) 4 : 1
(c) 1 : 8 (d) 8 : 1
21. The work function for a metal is 4 ev. To emit a photo electron of zero velocity from the
surface of the metal, the wavelength of incident light should be;
(a) 2700Å (b) 1700Å
(c) 5900Å (d) 3100Å
22. An electron in a H-like atom is in an excited state. It has a total energy of –3.4 ev,
calculate the de-Broglie’s wavelength?
(a) 66.5Å (b) 6.66Å
(c) 60.6Å (d) 6.06Å
23. When the frequency of light incident on a metallic plate is doubled, the KE of the emitted
photoelectron will be;
(a) doubled
(b) Halved
(c) Increased but more than doubled of the previous KE
(d) Remains unchanged
24. The ratio of the difference in energy of electron between the first and second Bohr’s orbit
to that between second and third Bohr’s orbit is;
1 27
(a) (b)
3 5
[Type text]

9 4
(c) (d)
4 9
25. The approximate quantum no. of a circular orbit of diameter 20.6 nm of the hydrogen
atom according to Bohr’s theory is
(a) 10 (b) 14
(c) 12 (d) 16
May be more than one
LEVEL-II
choice

1. The electrons, identified by quantum numbers n and 1, (i) n = 4, l = 1 (ii) n = 4, l = 0 (iii)


n = 3, l = 2 (iv) n = 3, l = 1 can be placed in order of increasing energy, from the lowest
to highest, as
(a) iv < ii < iii < I (b) ii < iv < i < iii
(c) I < iii < ii < iv (d) iii < I < iv < ii.

2. The energy of an electron in the first Bohr orbit of H atom is –13.6 eV. The possible
energy value(s) of the excited state(s) for electrons in Bohr orbits of hydrogen is (are)
(a) –3.4 eV (b) –4/2 eV
(c) –6.8 eV (d) + 6.8 eV.

3. Which of the following statement(s) is (are) incorrect?


(a) The electronic configuration of Cr is [Ar]3d 5 4s1 . (Atomic No. of Cr = 24).
(b) the magnetic quantum number may have a negative value.
(c) In silver atom, 23 electrons have a spin of one type and 24 of the opposite type.
(Atomic No. of Ag = 47).
(d) The oxidation state of nitrogen in NH3 is –3.

4. For a `d’-electron, the orbital angular momentum is


(a) 6h (b) 2 h
(c) h (d) 2h.

5. The first use of quantum theory to explain the structure of atom was made by
(a) Heisenberg (b) Bohr
(c) Planck (d) Einstein.

6. The orbital angular momentum of an electron in 2s orbital is:


1 h
(a) + . (b) zero
2 2π
h h
(c) (d) 2 .
2π 2π

7. The wavelength of a spectral line for an electronic transition is inversely related to


(a) The number of electrons undergoing the transition
[Type text]

(b) The nuclear charge of the atom


(c) The difference of the energy of the energy levels involved in the transition
(d) The velocity of the electron under going the transition.

8. Bohr model can explain


(a) the spectrum of hydrogen atom only
(b) spectrum of an atom or ion containing one electron only
(c) the spectrum of hydrogen molecule
(d) the solar spectrum

9. The radius of an atomic nucleus is of the order of


(a) 10 –10 cm (b) 10 –13 cm
(c) 10 –15 cm (d) 10 –8 cm .
10. When alpha particles are sent through a thin metal foil, most of them go straight through
the foil because:
(a) alpha particles are much heavier than electrons
(b) alpha particles are positively charged
(c) most part of the atom is empty space
(d) alpha particle move with high velocity
11. The increasing order (lowest first) for the value of c/m (charge/mass) for electron (e),
proton(p), neutron (n) and alpha particle (α) is:
(a) e, p, n, α (b) n, p, e, α
(c) n, p, a, α (d) n, α, p, e
12. Which electronic level would allow the hydrogen atom to absorb a photon but not to emit
a photon?
(a) 3s (b) 2p
(c) 2s (d) 1s
13. Any p-orbital can accommodate upto
(a) four electrons (b) six electrons
(c) two electrons with parallel spins (d) two electrons with opposite spins

14. The principal quantum number of an atom is related to the


(a) size of the orbital (b) spin angular momentum
(c) orbital angular momentum (d) orientation of the orbital in space

15. If uranium (mass number 238 and atomic number 92) emits an α-particle, the product has
mass no. and atomic no.
(a) 236 and 92 (b) 234 and 90
(c) 238 and 90 (d) 236 and 90

16. If the radius of first Bohr orbit is x, then the de-Broglie wavelength of electron in 3rd orbit
is nearly;
(a) 2?x (b) 6?x
[Type text]

x
(c) 9x (d)
3
17. The angular momentum of electron of H-atom is proportional to;
1
(a) r2 (b)
r
1
(c) r (d)
r
18. With what velocity must an electron travel so that its momentum is equal to that of a
photon of wavelength ? = 5200Å?
(a) 1400 ms–1 (b) 140 ms–1
(c) 14 ms–1 (d) 1.4 ms–1

19. The eyes of a certain member of reptile family pass a visual signal to the brain when the
visual receptors are struck by photon of wavelength 850 nm. If a total of energy of
3.15 ? 10–14 Joules is required to trip the signal, what is the minimum no. of photons that
must strike the receptor?
(a) 1.37 ? 105 photons (b) 13.7 ? 105 photons
(c) 4 ? 104 photons (d) 2 ? 103 photons

20. A photon of 300 nm is absorbed by a gas and then re-emitted two photons. One re-
emitted photon has wavelength 496 nm, the wavelength of second re-emitted photons is;
(a) 759 (b) 857
(c) 957 (d) 600

21. Which represents an possible arrangement


n l m s
(a) 3 2 –2 1/2
(b) 4 0 0 1/2
(c) 3 2 –3 1/2
(d) 5 3 0 1/2
22. Which set of quantum number is consistent with theory
(a) n = 2, l = 1, m = 0, s = –1/2 (b) n = 4, l = 3, m = 2, S = –1/2
(c) n = 3, l = 2, m = 3, S = +1/2 (d) n = 4, l = 3, m = 3, S = +1/2

23. Which of the following statements are false:


(a) The uncertainty in position and momentum in Heisenberg’s principle due to electron
wave.
(b) The energy level order 4s < 3d < 4p < 5s may not hold good for all elements
(c) The quantum nature of light radiation is manifested in photoemission of electrons
(d) According to Bohr’s theory the energy decreases as n increases.
24. Five valence electrons of 15P are labeled as
[Type text]

Pq x y z
3s 3P
1
If the spin quantum number of q and z is +
2
The group of electrons with three of the quantum number same are :
(a) Pq (b) (xyz), (pq)
(c) (pq), (xyz), (pz) (d) (pq), (xyz), (qy)
25. Which of the following is/are correct?
(a) the energy of an electron depends only on the principal quantum numbers not on the
other quantum numbers
(b) the energy of an electron depends only on the principal quantum number in case of
hydrogen and hydrogen like atoms.
(c) The difference in potential energies of any two energy level is always more than the
difference in kinetic energies of these two levels.
(d) An electron in an excited state can always emit a photon or two but can not absorb a
photon

LEVEL – III Comprehension Type Questions


? Write-up I
An orbital is designated by certain values of first three quantum numbers (n, ? and m)
and according to Pauli’s Exclusion Principle, no two electron in an atom can have all the
four quantum numbers equal. n, ? and m denote size, shape and orientation of the orbital.
The permissible values of n are 1, 2, 3 …..? while that of ? are all possible integral
values from 0 to n –1. Orbitals with same values of n and ? but different values of m
(where m can have any integral values from –? to +? including zero) are of equal energy
and are called degenerate orbitals. However degeneracy is destroyed in an
inhomogeneous external magnetic field due to different extent of interaction between the
applied field and internal electronic magnet of different orbitals differing in orientations.
In octahedral magnetic field the external magnetic field are oriented along axes while in
tetrahedral field the applied field acts more in between the axes than that on the axes
themselves. For ? = 0, 1, 2, 3…, the states (called sub-shells) are denoted by the symbol
s, p, d f …… respectively. After f, the subshells are denoted by the letters alphabetically. ?
determines orbital angular motion (L) of electron as
h
L = l ( l + 1)

On the otherhand, m determines Z – component of orbital angular momentum as
 h 
LZ = m  
 2π 
Hund’s rule states that in degenerate orbitals electrons do not pair up unless and until
each such orbital has got an electron with parallel spins.
Besides orbital motion, an electron also possess spin-motion. Spin may be clock-wise and
anti-clock wise. Both these spin motion are called two spin states of electron
[Type text]

1 1
characterised by spin Q.N.(s): s = + and s = − respectively. The sum of spin Q.N. of
2 2
all the electrons is called total spin (s) and (2s + 1) is called spin multiplicity of the
configuration as a whole. The spin angular momentum of an electron is written as:
h
Ls = s( s + 1 )

1. According to Hund’s rule, the distribution of electron within the various orbitals of a
given sub-shell is one which is associated with
(a) Minimum spin multiplicity (b) Maximum spin multiplicity
(c) Maximum energy (d) Minimum total spin

2. An orbital has n = 5 and its ? value is the maximum possible. The orbital angular
momentum of the electron in this orbital will be
h h
(a) 2 (b) 6
2π 2π
h  h 
(c) 12 (d) 20  
2π  2π 

3. ? introduces quantisation in which of the following?


(a) Energy of electron b) Shape of orbital
(c) Orbital angular momentum of orbital (d) All the three

4. What is the spin angular momentum of N-atom (Z = 7): EC = 1s22s22p3 (GS)


3 h 
(a) Zero (b)  
4  2π 
15  h  1 h 
(c)   (d)  
4  2π  2  2π 

? Write-up II
In H-spectrum, we get several spectral lines in different region like UV, visible and IR.
The wave lengths of different spectral lines in a particular series are different and can be
calculated by using Rydberg’s formula.
1 1 1
= ν = RZ 2  2 − 2 
λ  n1 n2 
where, R = 1.1 ? 107 m–1
In a particular series lines are very close to each other. In addition to this the energy gap
between two energy levels are going to decrease from lower energy level to higher
energy levels.
5. The significance of quantisation is going to be lost as
(a) The energy difference between consecutive energy levels are going to decrease.
[Type text]

(b) When we move from lower energy level to higher energy level, energy levels are
trying to converge into a single line
(c) Both (a) and (b)
(d) None

6. A high Rydberg atom is characterisecd by a transition of electron from n = 100 to n = X


level. The wavelength of emitted light is 4.49 cm. What is the value of X?
(a) X = 99 (b) X = 90
(c) X = 9 (d) X = 10

7. The atomic spectrum of Li2+ arises due to the transition of an electron from n1 to n2. If
(n1 + n2) = 4 and (n2 – n1) = 2, then find the wavelength of the 3rd line of this series in
Li2+ ion?
(a) 1.08 nm (b) 10.8 nm
(c) 108 nm (d) 1080 nm

? Write-up III

The single quantum number suggested by Neil’s Bohr and the appearance of several
spectral lines in a particular series of H-spectrum, suggest that there must be more than
one quantum number to explain all the quantized properties of an electron in a particular
energy level inside the atom.
In fact when the properties of the electron is transformed from Cartesian co-ordinate to
polar co-ordinates then it becomes the function of r, ?, and ? suggesting three
independent quantum numbers. In addition to this a fourth quantum number is required
to consider the spinning behaviour of an electron.
The principal quantum number n, suggest the orbit number in which electron revolves;
Azimuthal quantum number l suggest the shape of orbitals, magnetic quantum number
m, gives orientation of orbital in presence of external magnetic field, while spin quantum
number s, gives direction of rotation of an electron about it’s own axis.
n ? 0 values of “l” ranges in between 0 to (n –1), while m depends upon l. Total number
of values of m = 2l + l, and “m” ranges in between –l to +l including 0. For each
1
electron spin will be ± .
2

1 1
S =+ S =−
2 2
Clock wise spin Anti clock wise spin

8. The maximum number of electrons with n = 3, l = 3 is


(a) 14 (b) 10
(c) 6 (d) 0
[Type text]

9. If n = 3, l = 0, m = 0 then the possible atomic number may be


(a) 10, 11 (b) 11, 12
(c) 12, 13 (d) 13, 14

10. Two electrons A and B in an atom have the following set of quantum numbers; what is
true for A and B
For A; n = 3, l = 2, m = –2, s = +1/2
For B; n = 3, l = 0, m = 0,s = +1/2
(a) A and B have same energy (b) A has more energy than B
(c) B has more energy than A (d) A and B represent the same electron

? Write-up IV
Rutherford’s and Bohr’s atomic model became defective as in those atomic models only
particle nature of electron was considered. While according to wave mechanical concept
an electron behaves as a wave as well as a particle. This is true for all the microscopic
particles like nucleons, atoms, molecules and ions. de-Broglie established a relationship
between wavelength momentum and kinetic energy of the electron
h h
λ= =
p 2mKE
Where h = Planck’ constant, p = momentum, m = mass of electron & KE = Kinetic
energy
Later on, Heisen-berg (1927) told that the location of an electron at particular point from
nucleus is not possible. According to him “It is impossible to determine the momentum
and position of moving electron or any microscopic particle simultaneously and exactly.
If one will be certain then other will be uncertain.
According to Heisenberg
h
∆x.∆p ≥

Photoelectric effect, reflection of light suggest the particle nature of electron, while
deffraction, polarization, interference, scattering of light suggest the wave nature of
electron.

11. Determine de-Broglie wavelength of an electron having kinetic energy of 1.6 ? 10–6 erg,
me = 9.11 ? 10–28 g, h = 6.62 ? 10–27 erg sec.
(a) 1.22 ? 10–10 cm (b) 2.1 ? 10–3 cm
(c) 5 ? 10–5 cm (d) 6.0 cm

12. Calculate the accelerating potential that must be imparted to a proton beam to give it an
effective wave length of 0.005 nm.
(a) 30.85 volt (b) 32.85 volt
(c) 25.0 volt (d) 26.5 volts.
[Type text]

13. A green ball weighs 75 gm and comes travelling towards you at 400 cm/sec. A photon of
light emitted from green ball has a wavelength of 5 ? 10–5 cm. Assuming that error in
position of ball is the same as wave length of itself, calculate the error in momentum of
green ball.
(a) 1.055 ? 10–23 (b) 0.5 ? 10–23
(c) 2.3 ? 10–23 (d) 1.0

? Write-up V

It is tempting to think that all possible transitions are permissible, and that an atomic
spectrum arises from the transition of an electron from any initial orbital to any other
orbital. However, this is not so, because a photon has an intrinsic spin angular
h
momentum of 2 corresponding to S = 1 although it has no charge and no rest mass.

On the other hand, an electron has got two types of angular momentum: Orbital angular
h  h 
momentum, L == l(l + 1) and spin angular momentum, Ls  = s(s + 1)  arising
2π  2π 
from orbital motion and spin motion of electron respectively. The change in angular
momentum of the electron during any electronic transition must compensate for the
angular momentum carried away by the photon. To satisfy this condition the difference
between the azimuthal quantum numbers of the orbitals within which transition takes
place must differ by one. Thus, an electron in a d-orbital (? = 2) cannot make a transition
into an s-orbital (? = 0) because the photon cannot carry away enough angular
momentum. An electron as is well known, possess four quantum numbers n, ?, m and s.
Out of these four ? determines the magnitude of orbital angular momentum (mentioned
 h 
above) while m determines its Z-component as m   . The permissible values of only
 2π 
integers right from –? to +?. While those for ? are also integers starting from 0 to (n –1).
The values of ? denotes the sub-shell. For ? = 0, 1, 2, 3, 4….. the sub-shells are denoted
by the symbols s, p, d, f, g…. respectively.

14. To what orbitals may a 4d-electron make radioactive transitions?


(a) any of ns, np, nd and nf (b) any of np and nf orbitals
(c) only 3s and 4p (d) only 6f

15. Electronic transition from 4s to 3s orbital is forbidden meaning that it cannot occur
because
(a) There will be no change in the orbital angular momentum of electron although the
emitted photon has angular momentum.
(b) There will be change in the orbital angular momentum whereas the emitted photon
has no momentum.
[Type text]

(c) ?m? values between 4s and 3s is not zero which is an important selection rule for
allowed transition
(d) In 4s and 3s orbitals the wavelength of electron-wave are the same.

16. The maximum orbital angular momentum of an electron with n = 5 is


h h
(a) 6 (b) 12
2π 2π
h h
(c) 42 (d) 20
2π 2π

17. The orbital angular momentum of an electron in p-orbital makes an angle of 45° from Z-
axis. Hence Z-component of orbital angular momentum of electron is:
h  h 
(a) (b)  
π  2π 
h  h 
(c) − (d) −  
π  2π 

18. The spin-only magnetic moment of a free ion is 8 B.M. The spin angular momentum of
electron will be
h h
(a) 2 (b) 8
2π 2π
h 3 h
(c) 6 (d)
2π 4 2π

Answers to Assignments
SECTION - I

1. (b) 2. (d) 3. (a) 4. (d)


5. (d) 6. (c) 7. (d) 8. (b)
9. (a) 10. (a) 11. (c) 12. (b)
13. (c) 14. (c) 15. (d) 16. (b)
17. (a) 18. (b) 19. (b) 20. (c)
21. (d) 22. (b) 23. (c) 24. (b)
25. (b)

SECTION - II
[Type text]

1. (a) 2. (a) 3. (c) 4. (a)


5. (a) 6. (b) 7. (c) 8. (b)
9. (b) 10. (a, (c) 11. (d) 12. (d)
13. (d) 14. (a) 15. (b) 16. (b)
17. (c) 18. (a) 19. (a) 20. (a)
21. (a), (b), (d) 22. (a), (b), (d) 23. (b), (d) 24. (a), (b)
25. (b), (d)

SECTION - III

1. (b) 2. (d) 3. (d) 4. (c)


5. (c) 6. (a) 7. (b) 8. (d)
9. (b) 10. (b) 11. (a) 12. (b)
13. (a) 14. (b) 15. (a) 16. (d)
17. (b) 18. (a)

PERIODIC PROPERTIES
[Type text]

Graphs of periodic properties alkali metals


[Type text]

Lanthanides

Alkaline earth metals


[Type text]

Element of group IIIA


[Type text]

Carbon family

Nitrogen family (pnicogens)


[Type text]

Chalcogens

3-d series
[Type text]

Some Important Points


The periodic table proposed by – Rang.
Modified by – Werner

Extended by – Burry and his co_workers

(1) Triad rule – Dobereiner

(2) Octet rule – Newland

(3) Study of atomic volume – Lothar Mayer

(4) Inventor of atomic number – Moseley

(5) Godfather of periodic table – Mendeleef

(6) Maker of modern periodic table – Bohr

(7) Mg is bridge element, which joins metals of IIA and II B groups.

(8) Elements after atomic number 92 are transuranic elements.

(9) Artificial element is Tc43.

(10) Liquid non-metal – Br

(11) Liquid metal – Hg, Ga, Cs, Fr

(12) Solid volatile non-metal – Iodine

(13) Lightest metal – Li

(14) Heaviest metal – Ir


[Type text]

(15) Hardest metal – W

(16) Noble metals – Pc, Pt, Au, Ag

(17) Element most found on earth – Al

(18) Gaseous elements – 11 (He, Ne, Ar, Kr, Xe, Rn, H2, N2, O2, Cl2, F2)

(19) Liquid elements – 5(Br, Hg, Ga, Cs, Fr)

(20) Submetals – 5(B, Si, As, Te, At)

(21) Inert gases – 6

(22) Lowest electronegativity : Cs

(23) Highest electronegativity : F

(24) Highest ionisation potential : He

(25) Lowest ionisation potential : Cs

(26) Highest electron affinity :Chlorine (Cl)

(27) Least electropositive element : Fluorine (F)

(28) Most reactive solid element : Li

(29) Most reactive liquid element : Cs

(30) Most stable element :Te

(31) Largest atomic size :Cs

(32) Most electropositive element : Cs ; Fr


(in stable element) (In all element)
[Type text]

KVPY
1. Theenergyrequiredtoremoveanelectronfromanisolatedatominthegasphasefollowstheorder. (2007)
(A) C> B>Be> Li (B) C>Be> B> Li (C) B> C>Be> Li (D) Be> Li> B> C

2. The atomic radii of the elements across the second period of the periodic table (2010)
(A) Decrease due to increase in atomic number
(B) Decrease due to increase in effective nuclear charge
(C) Decrease due to increase in atomic weights
(D) Increase due to increase in effective nuclear charge
2 2 6 2 3
3. The number of valence electrons in an atom with electronic configuration 1s 2s 2p 3s 3p is
(2013)
(A) 2 (B) 3 (C) 5 (D) 11

4. The isoelectronic pair is (2014)


(A) CO, N2 (B) O2, NO (C) C2, HF (D) F2, HCl

5. The numbers of lone pairs and bond pairs in hydrazine are (2014)
(A) 2 and 4 (B) 2 and 6 (C) 2 and 5 (D) 1 and 5
-1
6. The first ionization enthalpies for three elements are 1314, 1680, and 2080 kJ mol , respectively.
The correct sequence of the elements is
(A) O, F and Ne (B) F, O and Ne (C) Ne, F and O (D) F, Ne and O

7. The energy required to remove an electron from an isolated atom in the gas phase follows the order.
(2007)
(A) C > B > Be > Li (B) C > Be > B > Li (C) B > C > Be > Li (D) Be > Li > B
>C

8. The atomic radii of the elements across the second period of the periodic table (2010)
(A) Decrease due to increase in atomic number
(B) Decrease due to increase in effective nuclear charge
(C) Decrease due to increase in atomic weights
(D) Increase due to increase in effective nuclear charge

2 2 6 2 3
9. The number of valence electrons in an atom with electronic configuration 1s 2s 2p 3s 3p is
(2013)
(A) 2 (B) 3 (C) 5 (D) 11

10. The isoelectronic pair is (2014)


(A) CO, N2 (B) O2, NO (C) C2, HF (D) F2, HCL

11. The numbers of lone pairs and bond pairs in hydrazine are, respectively (2014)
(A) 2 and 4 (B) 2 and 6 (C) 2 and 5 (D) 1 and 5

12. The first ionization inthalpies for three elements are 1314, 1680, and 2080 kJ mol-1,
respectively. The correct sequence of the elements is (2014)
(A) O, F and Ne (B) F, O and Ne (C) Ne, F and O (D) F, Ne and
O
[Type text]

13. The correct electronic configuration for the ground state of silicon (atomic number 14) is (2010)
(A) 1s2 2s2 2p6 3s2 3p2 (B) 1s2 2s2 2p4 3s2 3p4
2 2 6 2 4 2 2 6 1 2
(C) 1s 2s 2p 3s 3p (D) 1s 2s 2p 3s 3p

Answer Key:
7:B 8:B 9:C 10:A
11:C 12:A 13:A

LEVEL1
SECTION – I Single Choice Questions

1. The elements with atomic numbers 21 to 30 belong to


(a) halogens (b) boble gases (c) first transition element (d) alkali
metals

2. Which of the following oxides will be more acidic in nature ?


(a) Na2O (b) MgO (d) SiO2 (d) Cl2O7

3. Which of the following has the smallest size ?


(a) Na (b) Na+ (c) K (d) K+

4. Oxidising power of halogens decreases in the order


(a) Cl > Br > I (b) Br > I > Cl (c) I > Cl > Br (d) I > Br > Cl

5. Melting points of alkali metlas decrease in the order


(a) Li > Na > K (b) Na > K > Lin (c) K > Li > Na (d) K > Na >
Li

6. Which of the following elements is expected to have the most metallic character ?
(a) S (b) P4 (c) Cl2 (d) I2

7. In the periodic table, the ionization energy of elements


(a) decrease, (i) form left to right across a period and (ii) on descending a group
(b) decrease from left to right across a period and increases on descending a group
(c) increases, (i) from left to right across a period and (ii) on descending a group
(d) increase from left to right across a period and decreases on descending a group

8. In the periodic table, electro negativity of elements


(a) decreases, (i) from left to right across a period and (ii) on descending a group
(b) decrease from left to right across a period and increases on descending a group
(c) increases, (i) from left to right across a period and (ii) on descending a group
(d) increase from left to right across a period and decreases on descending a group
[Type text]

9. Which of the following elements has the least electron affinity ?


(a) F (b) Cl (c) Br (d) I

10. Which of the following orders regarding electron affinity is correct ?


(a) F > Cl > Br (b) F < Cl < Br (d) F < Cl > Br (d) F > Cl <
Br

11. Which of the following orders regarding the ionization energy is correct ?
(a) Be > B > C (b) Be < B < C (c) Be > B < C (d) Be < B > C

12. Which of the following orders regarding ionization energy is correct ?


(a) N > O > F (b) N < O < F (c) N > O < F (d) N < O > F

13. Which of the following orders regarding the melting point of alkali metal chlorides is true
?
(a) LiCl > NACl > KCl (b) LiCl < NaCl < K Cl
(c) LiCl < NaCl < KCl (d) LiCl > NaCl < KCl

14. Which of the following alkaline earth metal hydroxides is the most soluble in water ?
(a) Mg(OH)2 (b) Ca(OH)2 (c) Sr(OH)2 (d) Ba(OH)2

15. Which of the following alkaline earth metal sulphates is the least soluble in water ?
(a) MgSO4 (b) CaSO4 (c) SrSO4 (d) BaSO4

16. Which is the correct order of the basicity of alkaline earth hydroxides ?
(a) Be(OH)2> Mg(OH)2> Ca(OH)2 (b) Be(OH)2< Mg(OH)2< Ca(OH)2
(c) Be(OH)2> Mg(OH)2< Ca(OH)2 (d) Be(OH)2< Mg(OH)2> Ca(OH)2

17. Which of the following statements is not correct ?


(a) Boric acid is a very weak monobasic acid
(b) Boric acid contains planar BO3 units which are bonded together through hydrogen
bonds forming a layer structure
(c) Boric acid is used as food preservative
(d) Boric acid is a tribasic acid.

18. Which of the following is an electron deficient compound ?


(a) NH3 (b) NH2OH (c) BCl3 (d) H3N ?
BCl3

19. Orthoboric acid is


(a) monobasic (b) diabasic (d) tribasic (d) tetrabasic

20. In chain and cyclic structures of silicates


(a) no oxygen atom is shared amongst SiO4 tetrahedra
(b) one oxygen atom per tetrahedron is shared
[Type text]

(c) two oxygen atoms per tetrahedron are shared


(d) three oxygen atoms per tetrahedron are shared.

CHEMICAL BONDING

Ionic Bond or Electrovalent Bond


An ion is an atom or group of atoms which has acquired charge due to the loss or gain of one or
more electrons. When an atom gains an electron to form a negative ion (anion), it will increase in
size. On the other hand, when an atom loses an electron to give positive ion (cation), it will
contract. The electronlost or gained is always from the outermost shell.

When two atoms, one of which can lose one or more electrons to attain a noble gas configuration
and the other can receive these electrons and thereby acquire a noble gas configuration, they are
said to be bonded by an ionic bond. Since the loss and gain of electrons by atoms results in the
formation of ions, ionic bond is formed when two ions interact with each other and are thus held
together by electrostatic attraction. The formation of potassium chloride (KCl), is illustrated
below.
+
K (1s 2 2s 2 2p 6 3s 2 3p 6 4s1 ) loses
→ K (1s 2 2s 2 2p 6 3s 2 3p 6 )
1 electron

→ Cl − (1s 2 2s 2 2p6 3s 2 3p 6 )
Cl (1s 2 2s 2 2p 6 3s 2 3p5 ) 
gains
1 electron (Ar configuration)

From the above illustrations, it is clear that the formation of an ionic compound is obviously
related to the ease of formation of the cations and anions from the neutral atoms which depends
on two main factors:
i) Ionization energy:Lower the value of ionization energy of an atom, greater will be the
ease of formation of the cation from it.
ii) Electron affinity:Higher the electron affinity of an atom, greater the ease of formation
of the anion from it.
Lattice Energy
When one mole of an ionic solid is formed from its constituent gaseous ions, the energy released
is called the lattice energy.
[Type text]

Energetics of Formation of Ionic Substances: The energy included in the formation


of an ionic compound from its constituent elements may be considered as shown by the Born-
Haber Cycle for the formation of one mole of sodium chloride from sodium and chlorine.

Na ( s ) Sublimatio → Na (+g ) + e −
 n → Na ( g ) 
+S +I

1
Cl 2 ( g ) Dissociati
 on
→ Cl ( g ) Addition
 → Cl (−g )
of e -

2 +1 / 2 D* +EA

Na (+g ) + Cl (−g ) Crystal


 formation
  → NaCl ( S)
+U

Where S = heat of sublimation of sodium metal


I = ionization energy of sodium
D = heat of dissociation of molecular chlorine
Ea = electron affinity of chlorine, and
U = lattice energy of sodium chloride
The amount of heat liberated in the overall reaction is the heat of formation of sodium chloride.
From the above

1
?H? = S + I +
D – Ea – U
2
The most important of these energy terms are I, Ea and U, since these are considerably greater
than the remaining terms S and D.

More the negative value of the heat of formation, greater would be the stability of the ionic
compound produced. Thus on the basis of the above equation, formation of an ionic compound is
favoured by

a) low ionization energy (I) of the metal.


b) high electron affinity (EA) of the other element.
c) higher lattice energy (U) of the resulting compound.

Formation of Ions with Higher Charges:Formation of a cation with unit positive


charge is easy if the first ionization energy is low as in the case of alkali metals. Alkaline earth
metals ionizes in two successive steps.

Mg  → Mg+ + e–
Mg+  → Mg2+ + e–

But energy needed to ionize alkaline earth metals are higher than alkali metals.

However, bipositive ions like Mg2+, Ca2+, Sr2+ and Ba2+ are quite common. Formation of a
tripositive ion like Al3+ requires much more energy (= 5138 kJ) which is not available ordinarily.
Successive ionization energies of aluminium are:

Al →
E1
Al + + e − E1 = 577kJ
+ 2+ −
A l  → A l
E2
+e E2 = 1816kJ
A l 2 +  E→
3
A l3+ + e − E3 = 2745kJ
[Type text]

It is on this account that most of aluminium compounds are covalent. In solution, however,
aluminium is known to give hydrated ions [Al.6H2O]3+. This is possible because of the high heat
of hydration of Al3+. The energy liberated during hydration of ions is sufficient for ionization.

Similarly, anions with unit negative charge (e.g. Cl–1, Br–, I–) are very common. This is because
the electron affinity of these atoms is positive and quite high. Formation of anions carrying two
units of negative charge (e.g. S2–, O2–) is not so easy as their electron affinities are negative i.e.,
energy is needed to add second electron. Formation of anions carrying three units of negative
charge (e.g. N3–, P3–) is almost rare.

Characteristics of Electrovalent Compounds


Melting and Boiling Point:Due to the powerful electrostatic force between the ions in a
crystal of an electrovalent compound considerable energy is needed to overcome these forces
and break down the crystal lattice. Hence such compounds possess high melting and boiling
points.

Electrical Conductivity:When an electrovalent compound is molten or dissolved in a


solvent of high dielectric constant e.g., water, the binding forces in the crystal lattice disappear
and the component ions become mobile. Under the influence of applied electrical field, the ions
get charged and thus act as charge carrier of the current. Hence their melts or solutions conduct
electricity.

Solubility:Ionic compounds are soluble in polar solvents like water because of molecules of
the polar solvent interact strongly with the ions of the crystal and the solvation energy is
sufficient to overcome the attraction between the ions in the crystal lattice. Dissolution is also
favoured by the high dielectric constant of the solvents such as water, since this weakens the
interionic attractions in the resulting solutions.

Non-polar solvents like benzene and carbon tetrachloride do not solvate the ions as their
dielectric constants are low. Ionic compounds are, therefore insoluble in non-polar solvents.

Ionic compounds like sulphates and phosphates of barium and strontium are insoluble in water
(because lattice energy is greater than hydration energy). This can be attributed to the high lattice
energies of these compounds due to polyvalent nature of both the cation and the anion. In these
cases, hydration of ions fails to liberate sufficient energy to offset the lattice energy.

Covalent Bond (By Mutual Sharing of Electrons)


The covalent bond is formed when two atoms achieve stability by the sharing of an electron
pair, each contributing one electron to the electron pair.

The arrangement of electrons in a covalent molecule is often shown by a Lewis structure in


which only valency shells (outer shells) are depicted. For sake of clarity, the electrons on
different atoms are denoted by dots and crosses.
[Type text]

Polarity of Bonds: A covalent bond is set up by sharing of electrons between two atoms. It
is further classified as polar or non-polar depending upon the fact whether the electron pair is
shared unequally between the atoms or shared equally. For example, the covalent bonds in H2
and Cl2 are called non-polar as the electron pair is equally shared between the two atoms.
H H Cl Cl +δ −δ
Hydrogen molecule Chlorine molecule
(Both formed by equal sharing of H F
electrons between the atoms, i.e., by
non-polar bonds)
d

In the case of hydrogen fluoride the bond is polar as the electron pair is unequally shared.
Fluorine has a greater attraction for electrons or has higher electronegativity than hydrogen and
the shared pair of electrons is nearer to the fluorine atom than hydrogen atom. The hydrogen end
of the molecule, therefore, appears positive with respect to fluorine.

Bond polarities affect both physical and chemical properties of compounds containing polar
bond. The polarity of a bond determines the kind of reaction that can take place at that bond and
even affects the reactivity at nearby bonds. The polarity of bonds can lead to polarity of
molecules and affect melting point, boiling point and solubility.

Dipole Moment: It is vector quantity and is defined as the product of the magnitude of
charge on any of the atom and the distance between the atoms. It is represented by µ .

Magnitude of dipole moment | µ |= q × r


( charge in esu) ( Dis tan ce in Α )

The unit = 10–18 (esu) cm (D) is used in practice. In SI units charge q is measured in coulombs
(C) and the distance, r in metre, m

1C = 2.998 × 109 esu and 1 m = 102 cm


? 1 Cm = 2.998 × 109 × 102 = 2.998 × 1011 (esu) cm

Therefore in SI system, the unit of dipole moment is coulomb metre


2.998 × 1011
? 1 Cm = −18
= 2.998 × 10 29 D
10
1
or 1D = = 3.336 × 10 −30 Cm
2,998 × 10 29
Dipole moment is a vector quantity and is often indicated by an arrow parallel to the line joining
the point of charge and pointing towards the negative end e.g., H − F .
% Ionic character of a covalent bond = Experimental dipole moment
× 100
Theoretical dipole moment assuming 100% ionic character

Illustration 1: The dipole moment of KCl is 3.336 × 10–29 Cm. The interatomic distance
K+ and Cl– ion in KCl is 260 pm. Calculate the dipole moments of KCl, if
[Type text]

there were opposite charges of the fundamental unit located at each


nucleus. Calculate the percentage ionic character of KCl.

Solution: From the given data


q = 1.602 × 10–19C
r = 260 pm = 260 × 10–12 m = 2.6 × 10–10 m
Magnitude of dipole moment for 100% ionic character
|?| = qr = (1.602 × 10–19) (2.6 × 10–10) = 4.165 × 10–29 Cm
Actual dipole moment = 3.336 × 10–29 Cm
3.336 × 10 −29
? % of ionic bond = × 100 = 80.1%
4.165 × 10 −29
The bond is 80.1% ionic.

In general a polar bond is established between two atoms of different radii and different
electronegativities while positive centres (nuclei) of different magnitudes combine to share an
electron pair. Greater the values of the dipole moment, greater is the polarity of the bond.

The following points may be borne in mind regarding dipole moments:


i) In case a molecule contains two or more polar bonds, its dipole moment is obtained by
the vectorial addition of the dipole moments of the constituent bonds.

ii) A symmetrical molecule is non-polar even though it contains polar bonds.

For example, carbon dioxide, methane and carbon tetrachloride, being symmetrical molecules,
have zero dipole moments.

Dipole moment of methyl chloride is a vectorial addition of dipole moments of three C – H


bonds and one C – Cl bond.
H Cl Cl

H F C C C

H H Cl Cl H
µ = 1.75d H

H Cl H
µ = 0D µ = 0D µ = 1.86D

Hydrogen Methane Carbon Methyl


fluoride tetrachloride chloride
Dipole moments of some molecules

Dipole moment gives valuable information about the structure of molecules. For example,
carbon dioxide is assigned a linear structure since its dipole moment is zero.

We have seen that in a polar covalent bond between two atoms (say A and B), there is a partial
separation of charge. This bond is, therefore, said to have a partial ionic character. Greater the
difference of electronegativity between A and B, greater is the degree of ionic character (or
polarity measured by dipole moment of AB) of the bond. Pauling gave a fairly accurate rule by
[Type text]

which the nature of the bond can be predicted. According to this rule, “If the difference on the
electronegativity scale between the two atoms is 1.9, the bond is 50% ionic in character. When
the difference is greater than 1.9, the bond is correspondingly more ionic”. For example, when
the electro negativity difference is 0.8, 1.2, 2.2 and 2.6, the corresponding partial ionic character
is 12%, 25%, 61% and 74% respectively.

Characteristics of Covalent Compounds


Melting Point and Boiling Point:In covalent compounds, except those consisting of
giant molecules, the molecules are less powerfully attracted to each other, as a result of which
their melting points and boiling points are relatively low compared to ionic compounds, e.g.,

SiCl 4 (b.p. = 33K) and NaCl (b.p. = 1713K)


(Covalent compound) (ionic compound)

Conductivity:Covalent substances (whether of the “molecular lattice” or “giant molecule”


type) do not conduct electricity in the fused state since there are no free electrons or ions to carry
the current. However, substances like graphite which consists of separate layers conduct
electricity because the electrons have a passage in between the two flat layers.

Solubility:The characteristic solubility of covalent compounds in non-polar solvents such as


benzene and carbon tetrachloride can be described to the similar covalent nature of the molecules
of solute and solvent (i.e., like dissolves like). Covalent compounds in solution react more slowly
as compared with the ionic compounds which react instantaneously in solution. The solubility of
covalent compounds is, however, very much dependent upon the size of the molecule. Thus
covalent substances having giant molecules are insoluble in virtually all solvents due to the big
size of the molecule unit.

Fajan’s Rules
When two oppositely charged ions approach each other closely, the positively charged cation
attracts the outermost electrons of the anion and repel its positively charged nucleus. This results
in the distortion or polarization of the anion followed by some sharing of electrons between the
two ions, i.e., the bond becomes partly covalent in character.

i) Charge on Either of the ions:As the charge on the cation increases, its tendency
to polarize the anion increases. This brings more and more covalent nature in the
electrovalent compound. Whereas with the increasing charge of anion, its ability to get
polarized, by the cation, also increases.

For example, in the case of NaCl, MgCl2 and AlCl3 the polarization increases, thereby
covalent character becomes more and more as the charge on the cation increases.

Similarly, lead forms two chlorides PbCl2 and PbCl4 having charges +2 and +4
respectively. PbCl4 shows covalent nature. Similarly among NaCl, Na2S, Na3P, the
charge of the anions are increasing, therefore the increasing order of covalent character.
[Type text]

NaCl < Na2S < Na3P

ii) Size of the cation:Polarisation of the anion increases as the size of the cation
decreases i.e., the electrovalent compounds having smaller cations show more of the
covalent nature. For example, in the case of halides of alkaline earth metals, the covalent
character decreases as we move down the group. Hence melting point increases in the
order of
BeCl2< MgCl2< CaCl2< SrCl2< BaCl2

iii) Size of anion:The larger the size of the anion, more easily it will be polarized by the
cation i.e., as the size of the anion increases for a given cation, the covalent character
increases. For example, in the case of halides of calcium, the covalent character increases
CaF < CaCl < CaBr < CaI
from F– anion to I– anion i.e.  
2

2

2

2 →
increasing covalent character
Similarly, in case of trihalides of aluminium, the covalent character increases with
increase in size of halide anion i.e.

AlF3 AlCl3 AlBr 3 AlI 3


Covalent character increases as the
size of the halide ion increases

iv) Nature of the cation: Cations with 18 electrons (s2p6d10) in outermost shell polarize
an anion more strongly than cations of 8 electrons (s2p6) type. The d electrons of the 18
electron shell screen the nuclear charge of the cation less effectively than the s and p
electrons of the 18-electron shell. Hence the 18-electron cations behave as if they had a
greater charge. Copper (I) and Silver (I) halides are more covalent in nature compared
with the corresponding sodium and potassium halides although charge on the ions is the
same and the sizes of the corresponding ions are similar. This illustrates the effect of 18-
electron configuration of Cu+ (3s2, p6, d10) and Ag+ (4s2, p6, d10) ions.

Illustration 4: The decomposition temperature of Li2CO3 is less than that of Na2CO3.


Explain.

Solution: As Li+ ion is smaller than Na+ ion, thus small cation (Li+) will favour more
covalent character in Li2CO3 and hence it has lower decomposition
temperature than that of Na2CO3.
BRAIN TEASER 1:
SnCl2 is solid whereas SnCl4 is liquid, why?

Hydrogen Bonding
In 1920, Latimer and Rodebush introduced the idea of “hydrogen bond” to explain the nature of
association in liquid state of substance like water, hydrogen fluoride, ammonia, formic acid etc.
In a hydrogen compound, when hydrogen is bonded to highly electronegative atom (such as F,
O, N) by a covalent bond, the electron pair is attracted towards electronegative atom so strongly
[Type text]

that a dipole results i.e., one end carries a positive charge (H-end) and other end carries a
negative charge (X-end).

δ− δ+ δ− δ+
X H or X H

Electro-negative atom

If a number of such molecules are brought nearer to each other, the positive end of one molecule
and negative end of the other molecule will attract each other and weak electrostatic force will
develop. Thus, these molecules will associate together to form a cluster of molecules.
δ− δ+ δ− δ+ δ− δ+ δ− δ+ δ− δ+
X− HL X− H L X− HL X− HL X− H

The attractive force that binds hydrogen atom of one molecule with electronegative atom of the
other molecule of the same or different substance is known as hydrogen bond.

Hydrogen bonding is of two types:

a) Intermolecular hydrogen bonding:This type of bonding results between the


positive and negative ends of different molecules of the same or different substances.

Example

i) Ammonia

δ+ δ+ δ+ δ+ δ+
H H H H H
+
δ− δ− δ− δ− δ− δ
N H N H N H N H N H

H H H H H
δ+ δ+ δ+ δ+ δ+

ii) Water
δ+ δ+ δ+ δ+
H H H H
δ− δ− δ− δ−
O H O H O H O H

iii) Acetic acid


O H O
H3C CH3
O H O
This type of hydrogen bonding increases the boiling point of the compound and also its
solubility in water. The increase in boiling point is due to association of several
molecules of the compound.
[Type text]

b) Intramolecular hydrogen bonding:This type of bonding results between


hydrogen and an electronegative element both present in the same molecule. This type of
bonding is generally present in organic compounds. Examples are o-nitro-phenol, o-
hydroxy benzoic acid, etc.
O - O -
H H
O O
N

O O
o-Nitrophenol o-Hydroxy benzoic acid

This type of bonding decreases the boiling point of the compound. The solubility of the
compound also decreases. Hence compound becomes more volatile.

Properties Explained by Hydrogen Bonding

a) Strength of certain acids and bases can be explained on the basis of hydrogen bonding.

b) Solubility: An organic substance is said to be insoluble in water if it does not form


hydrogen bonding with water. The organic compound like alkanes, alkenes, ethers, etc.,
are insoluble in water as they do not form hydrogen bonding with water, while alcohols
and acids are soluble because they readily form hydrogen bonds with water.

i) Melting and boiling points of hydrides of N, O and F.If the melting points
and boiling points of the hydrides of the elements of IVA, VA, VIA and VIIA groups
are plotted against the molecular weights of these hydrides, we shall get the plots as
shown in figure (a) and (b).

From these plots it may be seen that although in case of SbH3, AsH3, PH3 (VA group
elements hydrides), H2Te, H2Se, H2S (VI A group elements hydrides) and HI, HBr,
HCl (VIII group elements hydrides) there is a progressive decrease in their mp’s and
b.p’s with the decrease in their molecular weights, the mp’s and b.p’s of NH3, H2O
and HF hydrides suddenly increase with a further decrease of their molecular weights.
The sudden increase in mp’s and bp’s in these hydrides is due to the inter-molecular
H-bonding in between H and F in case of HF, in between H and O in case of H2O and
in between H and N in case of NH3 respectively. The existence of H-bonding in these
molecules gives polymerized molecules (NH3)n. Thus mp’s and bp’s of these
molecules are suddenly raised.

Having no power to form H-bonds, the simple carbon family hydrides (SnH4, GeH4,
SiH4 and CH4) show a decrease in their bp’s and mp’s with the decrease in their
molecular weights.
[Type text]

100 100
H2 O

HF
Melting points (°C) increasing

H 2 Se

Boiling points (°C) increasing


H2 O
VIA
0 0
H 2 Se H 2 Se SbH3
VIA NH 3 H2 S VA
H 2 Se VIIIA VIIIA
HI
H2 S HI
NH 3 VA HCl
HBr SbH3 HBr
HCl IVA
-100 HF -100 SnH4
PH3 GeH4
AsH3
PH3 SnH4 SnH4
IVA
GeH4
CH 4 SnH4 CH 4

-200 -200
Molecular weight increasing Molecular weight increasing
(a) (b)

ii) Ice has less density than water.The explanation of this fact is as follows: In
the crystal structure of ice, the O-atom is surrounded by four H-atoms. Two H-atoms
are linked to O-atom by covalent bonds as shown (by normal covalent bond) and the
remaining two H-atoms are linked to O-atom by two H-bonds shown by dotted lines.
Thus in ice every water molecule is associated with four other water molecules by H-
bonding in a tetrahedral fashion. Ice has an open cage like structure with a large
empty space due to the existence of H-bonds. As ice melts at 0°C, a number of H-
bonds are broken down and the space between water molecules decreases so that
water molecules move closer together. The density of water increases, from 0° to
4°C, and at 4°C it is maximum. Above 4°C the increase in kinetic energy of the
molecules is sufficient to cause the molecules to begin to disperse and the result is
that the density decrease with increasing temperature.

H H

2.76 Å 1.80Å

-
H Open cage-like tetrahedral crystal structure
of ice. Circles indicate oxygen atoms.
0.96Å
0.96Å
Bonds represented by solid line are normal
covalent bonds while those represented by
water molecule dotted lines are hydrogen bonds.
H
H H

H H
H

BRAIN TEASER 2:
Although HF forms stronger hydrogen bond than H2O, ?Hv of H2O is greater than that
of HF why?
[Type text]

Coordinate Bond
It is a special type of covalent bond in which both the shared electrons are contributed by one
atom only. It may be defined as “a covalent bond in which both electrons of the shared pair
are contributed by one of the two atoms”. Such a bond is also called as dative bond. A
coordinate or a dative bond is established between two such atoms, one of which has a complete
octet and possesses a pair of valence electrons while the other is short of a pair of electrons.

xx xx
A + B xx A B x or A B
xx xx x

This bond is represented by an arrow (?) pointing towards acceptor atom.

The atom which contributes electron pair is called the donor while the atom which accepts it is
called acceptor.

The compound consisting of the coordinate bond is termed coordinate compound. Some
examples of coordinate bond formation are given below:

i) Formation of ammonium ion:Hydrogen ion (H+) has no electrons and thus


accepts a lone pair donated by nitrogen.
H H
+
H N + H H N H

H H

ii) Formation of CO:Carbon has four valency electrons and oxygen has six. They
combine to form two double bond and a coordinate bond as to achieve their octet
completed.
xx
C + xx Oxx C Oxx
Acceptor Donor

Characteristics of Coordinate Compounds: The properties of coordinate compounds


are intermediate between the properties of electrovalent compounds and covalent compounds.
The main properties are described below:
i) Melting and Boiling Points:Their melting and boiling points are higher than purely
covalent compounds and lower than ionic compounds.
ii) Solubility:These are sparingly soluble in polar solvents like water but readily soluble in
non-polar (organic) solvents.
iii) Conductivity:Like covalent compounds, these are also bad conductors of electricity.
The solutions or fused mass do not allow the passage of electricity.
[Type text]

Valence Shell Electron Pair Repulsion (VSEPR) Theory


In 1957 Gillespie and Nyhom gave this theory to predict and explain molecular shapes and bond
angles more exactly. The theory was developed extensively by Gillespie as the Valence Shell
Electron Pair Repulsion (VSEPR) theory. This may be summarized as:
1. The shape of the molecule is determined by repulsions between all of the electron pairs
present in the valence shell.
2. A lone pair of electrons takes up more space round the central atom than a bond pair,
since the lone pair is attracted to one nucleus whilst the bond pair is shared by two nuclei.
It follows that repulsion between two lone pairs is greater than repulsion between a lone
pair and a bond pair, which in turn is greater than the repulsion between two bond pairs.
Thus the presence of lone pairs on the central atom causes slight distortion of the bond
angles from the ideal shape. If the angle between a lone pair, the central atom and a bond
pair is increased, it follows that the actual bond angles between the atoms must be
decreased. The order of repulsion between lone pairs and bond pairs of electrons follows
the order as:
Lone pair - lone pair repulsion > lone pair – bond pair repulsion > bond pair – bond pair
repulsion.
3. The magnitude of repulsions between bonding pairs of electrons depends on the
electronegativity difference between the central atom and the other atoms.
4. Double bonds cause more repulsion than single bonds, and triple bonds cause more
repulsion than a double bond.

Effect of Lone Pairs: Molecules with four electron pairs in their outer shell are based on a
tetrahedron. In CH4 there are four bonding pairs of electrons in the outer shell of the C atom, and
the structure is a regular tetrahedron with bond angle H – C – H of 109°28’. In NH3 and N atom
has four electron pairs in the outer shell, made up of three bond pairs and one lone pair. Because
of the lone pair, the bond angle H – N – H is reduced from the theoretical tetrahedral angle of
109°28’ to 107°28’. In H2O the O atom has four electron pairs in the outer shell. The shape of
the H2O molecule is based on a tetrahedron with two corners occupied by bond pairs and the
other two corners occupied by lone pairs. The presence of two lone pairs reduces the bond angle
further to 104°27’.

In a similar way, SF6 has six bond pairs in the outer shell and is a regular octahedron with bond
angles of exactly 90°. In BrF5, the Br also has six outer pairs of electrons, made up of five bond
pairs and one lone pair. The lone pair reduces the bond angles to 84°30’. Whilst it might be
expected that two lone pairs would distort the bond angles in an octahedral as in XeF4 but it is
not so. Actual bond angle is 90°, reason being that the lone pairs are trans to each other in the
octahedron, and hence the atoms have a regular square planar arrangement.

Molecules with five pairs of electrons are all based on a trigonal bipyramid. Lone pairs distort
the structures as before. The lone pairs always occupy the equatorial positions (in an triangle),
rather than the axial positions (up and down).Thus in I3− ion, the central I atom has five electron
pairs in the outer shell, made of two bond pairs and three lone pairs. The lone pairs occupy all
[Type text]

three equatorial positions and the three atoms occupy the top, middle, and bottom positions in the
trigonal bipyramid, thus giving a linear arrangement with a bond angle of exactly 180°.

Effect of Electronegativity: NF3 and NH3 both have structures based on a tetrahedron
with one corner occupied by a lone pair. The high electronegativity of F push the bonding
electrons further away from N than in NH3. Hence the lone pair in NF3 causes a greater distortion
from tetrahedral and gives a F – N – F bond angle of 102°30’, compared with 107°48’ in NH3.
The same effect is found in H2O (bond angle 104°27’) and F2O (bond angle 102°).

The effects of bonding and lone pairs on bond angles


Orbitals on central Number of Number of lone
Shape Bond angle
atom bond pairs pairs
BeCl2 2 Linear 2 0 180°
BF3 3 Plane triangle 3 0 120°
CH4 4 Tetrahedral 4 0 109°28?
NH3 4 Pyramidal 3 1
107°48?
NF3 4 Pyramidal 3 1
H2 O 4 Bent (V-shape) 2 2 102°30?
F2 O 4 Bent (V-shape) 2 2
104°27?
102°
PCl5 5 Trigonal bipyramid 5 0 120° and 90°
SF4 5 Trigonal bipyramid 4 1 101°36? and 86°33?
ClF3 5 T-shape 3 2
87°40?
XeF2 5 Linear 2 3
180°
SF6 6 Octahedral 6 0 90°
BrF5 6 Square pyramidal 5 1 84°30?
XeF4 6 Square planar 4 2
90°

Some examples using the VSEPR Theory

Phosphorus pentachloride PCl5:Gaseous PCl5 is Cl


covalent. (The electronic structure P is 1s22s22p63s23p3). All five
outer electrons are used to form bonds to the five Cl atoms. In the
PCl5 molecule the valence shell of the P atom contains five
electron pairs: hence the structure is a trigonal bipyramid. There Cl Cl
are no lone pairs, so the structure is not distorted. However, a
trigonal bipyramid is not a completely regular structure, since P
some bond angels are 90° and others 120°. Symmetrical structures
are usually more stable than asymmetrical ones. Cl

Note: Thus PCl5 is highly reactive, and in the solid state it splits
into [PCl4]+ and [PCl6]– ions, which have tetrahedral and
octahedral structures respectively.
Cl
Structure of PCl5 molecule
[Type text]

Chlorine trifluoride ClF3:The chlorine atom is at the centre of the molecule and determines
its shape. The electronic configuration of Cl is 1s22s22p63s23p5. Three electrons form bonds to F,
and four electrons do not take part in bonding. Thus in ClF3, the Cl atom has five electron pairs
in the outer shell, hence the structure is a trigonal bipyramid. There are three bond pairs and two
lone pairs.

It was noted previously that a trigonal bipyramid is not a regular shape since the bond angles are
not all the same. It therefore follows that all the corners are not equivalent. Lone pair occupy two
of the corners, and F atoms occupy the other three corners. Three different arrangements are
theoretically possible, as shown in figure below.

The most stable structure will be the one of lowest energy, that is the one with the minimum
repulsion between the five orbitals. The great repulsion occurs between two lone pairs. Lone pair
bond pair repulsions are next strongest, and bond pair-bond pair repulsions are weakest. Groups
at 90° repel each other strongly, whilst groups 120° apart repel each other much less.
F F

F F F F
Cl Cl Cl

F F

F
I II III
Chlorine trifluoride molecule

Structure I is the most symmetrical, but has six 90° repulsions between lone pairs and atoms.
Structure II has one 90° repulsion between two lone pairs, plus three 90° repulsions between lone
pairs and atoms. These factors indicate that structure III is the most probable. The observed bond
angles are 80°40?, which is close to the theoretical 90°. This confirms that the correct structure
is III, and the slight distortion from 90° is caused by the presence of the two lone pairs.

As a general rule, if lone pairs occur in a trigonal bipyramid they will be located in the equatorial
position (round the middle) rather than the axial positions (top and bottom), since this
arrangement minimizes repulsive forces.

Sulphur hexafluoride SF6:The electronic structure of S F

is 1s22s22p63s23p6. All six of the outer electrons are used to form F F


bonds with the F atoms. Thus in SF6, the S has six electron pairs
S
in the outer shell: hence the structure is octahedral. There are no
lone pairs, so the structure is completely regular with bond angles F F
of 90°.
F

Valence Bond Theory


This theory was proposed by Linus Pauling, who was awarded the Noble Prize for
Chemistry 1954.
[Type text]

Atoms with unpaired electrons tend to combine with other atoms which also have unpaired
electrons. In this way the unpaired electrons are paired up, and the atoms involved, all attain a
stable electronic arrangement. This is usually a full shell of electrons(i.e. a noble gas
configuration). Two electrons shared between two atoms constitute a bond. The number of bonds
formed by an atom is usually the same as the number of unpaired electrons in the ground state,
i.e. the lowest energy state. However, in some cases the atom may form more bonds than this.
This occurs by excitation of the atom (i.e. providing it with energy) when electrons which were
paired in the ground state are unpaired and promoted into suitable empty orbitals. This increases
the number of unpaired electrons, and hence it increases number of bond which can be formed.

A covalent bond results from the pairing of electrons (one from each atom). The spins of the two
electrons must be opposite (antiparallel) because of the Pauli exclusion principle that no two
electrons in one atom can have all four quantum numbers the same.

1. In HF, H has a singly occupied s-orbital that overlaps with a singly filled 2p orbital on F.

2. In H2O, the O atom has two singly filled 2p orbitals, each of which overlaps with a single
occupied s-orbital from two H atoms.

3. In NH3, there are three singly occupied p orbitals on N which overlap with s orbitals from
three H atoms.

4. In CH4,the C atom in its ground state has the electronic configuration 1s2, 2s2, 2p1x , 2p1y
and only has two unpaired electrons, and so can form only two bonds. If the C atom is
excited, then the 2s electrons may be unpaired, giving 1s2, 2s1, 2p1x , 2p1y , 2p1x . There are
now four unpaired electrons which overlap with singly occupied s orbitals on four H
atoms.
2p
2s 2px 2py 2pz
Electronic structure of
carbon atom - groun state

1s 2s 2p
Carbon atom - excited
state

Carbon atom having


gained four electrons
from H atoms in CH4
molecule. sp3 hybridisation

CH4 molecule uses its three p-orbitals px, py and pz, which are mutually at right angles to
each other, and the s orbital is spherically symmetrical. Hence they form tetrahedral
structure.

CH4 H – C – H = 109°28?

? and ? Bonds)
Sigma and Pi Bonds (?
[Type text]

A covalent bond is formed by the overlapping of atomic orbitals. Covalent bonds formed are of
two types depending upon the way the orbitals overlap each other.

1. ? bond):The bond formed by the overlapping of two half filled atomic


Sigma bond (?
orbitals along their axis is known as sigma bond. ? bond is a strong bond because
overlapping in it takes place to large extent. The hybrid orbitals always from ? bond.

a) s – s overlapping

Molecular axis

b) s – p overlapping

c) p – p overlapping
head on
overlap
pz pz p-p overlap σ M.O.

2. Pi bond (? ? bond):The bond formed by the lateral overlapping of half filled atomic
orbitals is known as pi bond. The sidewise overlapping takes place to less extent.
Therefore, ? bond formed is a weak bond. ? bond overlapping takes place only at the
sides of two lobes. A ? bond is formed when a ? bond already exists between the
combining atoms.

p p p-p overlapping π M.O.

Example:

In A – B molecule the bond formed is ? bond.


σ
In A B, molecule there are one σ and one π bonds
π
σ
In A B, molecule there are one σ and two π bonds
π
Thus, all the single bonds are ? bonds. Double bond has one ? and one ? bond. Triple bond has
one ? and two ? bonds.
[Type text]

Hybridisation
It is the mathematically fabricated concept that is introduced to explain the geometry/shapes of
the covalent molecules of polyatomic ions containing covalent bonds.

It is a process of intermixing of atomic orbitals with small difference in energy and belonging to
the same atom, at the time of bonding so as to give another set of orbitals with equivalent shapes
and energies.

sp3 Hybridisation:In ground state, the electronic configuration of carbon is 1s2, 2s2, 2p2. It is
proposed that from 2s orbital, being quite near in energy to 2p orbitals, one electron may be
promoted to the vacant 2pz orbital thus obtaining the excited atom. At this stage the carbon atom
undoubtedly has four half-filled orbitals and can form four bonds. In the excited atom, all the
four valence shell orbitals may mix up to give four identical sp3 hybrid orbitals. Each of these
four sp3 orbital possesses one electron and overlaps with 1s orbitals of four H atoms thus forming
four equivalent bonds in methane molecule. Due to the tetrahedral disposition of sp3 hybrid
orbitals, the orbital are inclined at an angle of 109° 28’. Thus all the H– C– H angles are equal to
109° 28’

2p 2p
Promotion sp3
Energy

of an electron Hybridisation

2s 2s
Ground State Excited State Hybridised State

H
H

109.5°
C
H H
H H
H H
Shape and formation of methane molecule

sp2 Hybridisation:When three out of the four valence obritals of carbon atom in excited
state hybridize, we have three sp2 hybrid orbitals lying in a plane and inclined at an angle of
120°. If 2s and 2p, orbitals of the excited carbon atom are hybridized, the new orbitals lie in the
xy plane while the fourth pure 2pz orbitals lies at right angles to the hybridized orbitals with its
two lobes disposed above and below the plane of hybrid orbitals. Two such carbon atoms are
involved in the formation of alkenes (compounds having double bonds). In the formation of
ethene two carbon atoms (in sp2 hybridization state) form one sigma bond by `head-on’ overlap
of two sp2 orbitals contributed one each by the two atoms. The remaining two sp2 orbitals of each
carbon form ? bonds with H atoms. The unhybridized 2p, orbitals of the two carbon atoms
[Type text]

undergo a side-wise overlap forming a ? bond. Thus the carbon to carbon double bond in ethene
is made of one ? bond and one ? bond. Since the energy of a ? bond is less than that of a ? bond,
the two bonds constituting the ethene molecule are not identical in strength. The molecule is a
planar one.
Pure p-orbital

2p 2p
Promotion sp2
Energy

of an electron Hybridisation
sp2 hybrid
orbitals

2s 2s
Ground State Excited State

pz pz

π π
H H H
H
σ σ σ
sp2 C C sp2 2
sp2 sp
σ H sp2 sp2 σ H H
π H

Orbital model of ethane molecule


Different types of hybridization depend upon the type of atomic orbitals which are used for
intermixing.

Types of hybridization and spatial orientation of hybrid orbitals: The


geometry and shapes of various species on the basis of VSEPR theory along with hybrid state of
central atom is given below in tabular form.
Orientation
Types of atomic
Hybridisation of hybrid Examples
orbitals used
orbitals
1. one s + one p-orbital sp Linear BeF 2 , BeCl 2 , C 2 H 2 , HgCl 2
Trigonal
2. one s + two p-orbitals sp 2 BF3 , C 2 H 4 , NO 3− , CO 32 −
planar
3. one s + three p-orbitals sp 3 Tetrahedral CH 4 , CCl 4 , SiF4 , NH +4 , SO 24− , ClO −4
Trigonal
4. one s + three p + d sp 3 d PF5 , PCl 5
bipyramidal
5. one s + three p + two d sp 3 d 2 Octahedral SF5 , [CrF6 ] 3− , IF5
Pentagonal
6. one s + three p+three d sp 3 d 3 IF7
Bipyramidal
Square Only in complexes like
7. One d + one s + two p dsp 2
planar [ Ni (CN ) 4 ) 2 − , [ PtCl 4 ] 2− etc.
Note: i) Orbitals participating in hybridization must have only small difference in their energies.
ii) Both half-filled and completely filled orbitals can get involved in hybridization.
[Type text]

iii) The number of hybrid orbitals is equal to the number of orbitals participating in hybridization.
iv) Hybrid orbital form more stronger bonds than pure atomic orbitals.
v) Same atom can assume different hybrid states under different situations.
vi) Hybrid orbitals form sigma bonds.

Method of predicting the Hybrid state of the central atom in covalent


molecules of polyatomic ions:

The hybrid state of the central atom in similar covalent molecule or polyatomic ion can be
predicted by using the generalized formula as described below :

Simple Molecule Polyatomic Anion Poyatomic Cation


1 1 1
X = [V + G] X = [V + G + a ] X = [ V + G − c]
2 2 2
In the above formulae,
V = Number of monovalent atoms or groups attached to the central atom
G = Number of outer shell electrons in ground state of the central atom
a = Magnitude of charge on anion
c = Magnitude of charge on cation
Calculate the value of X and decide the hybrid state of central atom as follows :
X 2 3 4 5 6 7
Hybrid state sp sp 2
sp 3
sp d 3 3 2
sp d sp 3 d 3
PF5 COCl 2 NH +4 ClO −4
X =12 [5 + 5] X =12 [ 2 + 4] X =12 [ 4 + 5 − 1] X =12 [0 + 7 + 1]
=5 =3 =4 =4
Hybrid state of P is Hybrid state of C is Hybrid state of N is Hybrid state of Cl is
sp 3 d sp 2 sp 3 sp 3
NO 3− IF5 CO 2 XeF4

X =12 [0 + 5 + 1] X =12 [5 + 7] X =12 [0 + 4] X =12 [ 4 + 8]


=3 =6 =2 =6
2 3 2 sp
sp sp d sp 3 d 2
PCl 6− PH3 SF3+ SF4
X =12 [6 + 5 + 1] X =12 [3 + 5] X =12 [3 + 6 − 1] X =12 [ 4 + 6]
=6 =4 =4 =5
Hybrid state Hybrid state Hybrid state Hybrid state
sp 3 d 2 sp 3 sp 3 sp 3 d

Molecular Orbital Theory


[Type text]

Why He2 molecule does not exist and why O2 is paramagnetic? These questions cannot be
explained by valence bond theory. In 1932 F. Hund and R.S. Mulliken put
forward a theory known as Molecular Orbital Theory to explain above
questions and many others. According to this theory, as the electrons of an
atom are present in various atomic orbitals, electrons of a molecule are
present in various molecular orbitals. Molecular orbitals are formed by the
combination of atomic orbitals of comparable energy and proportional
symmetry. While an electron in atomic orbital is influenced by one
nucleus, in a molecular orbital, it is influenced by two or more nuclei
depending upon the number of atoms in the molecule. Thus, an atomic
orbital is monocentric while a molecular orbital is polycentric. The
number of molecular orbitals formed is equal to the number of combining
atomic orbitals. When two atomic orbitals combine, two molecular
orbitals are formed. One is known as bonding molecular orbital (BMO)
whereas other is anti-bonding molecular orbital (ABMO). BMO has lower
energy and hence greater stability than the corresponding ABMO. First
BMO are filled, then ABMO starts filling because BMO has lower energy
than that of ABMO.

Molecular orbitals like the atomic orbitals are filled in accordance with the Aufbau Principle
obeying the Pauli’s Principle and the Hund’s rule.

Order of energy of various molecular orbitals is as follows:


For O2 and higher molecules

?1s, ?*1s, ?2s, ?*2s, ?2px, [?2py = ?2pz], [?*2py = ?*2pz], ?*2px
For N2 and lower molecules
?1s, ?*1s, ?2s, ?*2s, [?2py = ?2pz], ?2px, [?*2py = ?*2pz], ?*2px

Bond order:It may be defined as the half the difference between the number of electrons
present in the bonding orbitals and the anti-bonding orbitals i.e.

No. of electrons in BMO - No. of electrons in ABMO


Bond order (B.O.) =
2
A positive bonding order suggest a stable molecule while a negative bond order or zero bond
order suggest an unstable molecule.

Magnetic Behaviour:If all the molecular orbitals in species are spin paired, the substance is
diamagnetic. However, if one or more molecular orbitals are singly occupied it is paramagnetic.

Illustration 2 Arrange the species O2, O2–,O22– and O2+ in the decreasing order of bond
order and stability and also indicate their magnetic properties.
Solution: The molecular orbital configuration of O2, O2–,O22– and O22+ are as follows:
[Type text]

O2 = ?1s2, ?*1s2, ?2s2, ?*2s2, ?2px2, ?2py2, ?2pz2, ?*2py1 = ?*2pz1


10 - 6
Bond order = = 2 , No. of unpaired electrons = 2
2
? paramagnetic
O2– = ?1s2, ?*1s2, ?2s2, ?*2s2, ?2px2, ?2py2, ?2pz2, ?*2py2 = ?*2pz1
10 - 5
Bond order = = 2.5 , No. of unpaired electrons = 1
2
? paramagnetic

O22– = ?1s2, ?*1s2, ?2s2, ?*2s2, ?2px2, ?2py2, ?2pz2, ?*2py2 = ?*2pz2
10 - 8
Bond order = = 1 , No. of unpaired electrons = 0
2
? diamagnetic
O2+ = ?1s2, ?*1s2, ?2s2, ?*2s2, ?2px2, ?2py2, ?2pz2, ?*2py1 = ?*2pz0
10 - 5
Bond order = = 2.5 , No. of unpaired electrons = 1
2
? paramagnetic

Now as the bond order decreases in the order O2+? O2? O2–? O22–

So, same will be the stability order of the above species because stability is
directionally proportional to bond order.

Note: Bond length is inversely proportional to bond order.


BRAIN TEASER 3:
Arrange N2, N2+, N2– in the increasing order of stability

Solved Problems
SUBJECTIVE

Problem 1: The bond angle of H2O is 104° while that of F2O is 102°.
[Type text]

Solution: Both H2O and F2O have a lone pair of electrons. But fluorine being highly
electronegative, the bond pair electrons are drawn more towards F in F2O. The
bond pairs being displaced away from the central atom has very little tendency
to open up the angle. But in H2O, this opening up is more as bond pair
electrons are closer to each other. So bond angle of F2O is less than H2O.

H F
repulsion repulsion less
H F

Problem 2: Explain why bond angle of NH3 is greater than NF3 while bond angle of
PH3 is less than that of PF3.

Solution: Both NH3 and NF3 are pyramidal in shape with one lone pair on N. However
F has higher electronegativity than H, the electron pair is attracted more
towards F in NF3 i.e. the bond pairs of electrons are away from N or in other
words distance between bond pairs ion is more. Hence repulsion between
bond pairs in NF3 is less than NH3. Hence the lone pair repels the bond pairs
of NF3 more than it does in NH3. As a result, the bond angle decreases to
102.4°. Whereas in NH3 it decreases to 107.3° only.

N N
H H F F
H F
107.3° 102.4°

PH3 and PF3 are also pyramidal in shape with one lone pair on P. But PF3 has
greater bond angle than PH3 (opposite to NH3 and NF3). This is due to
resonance in PF3, leading to partial double bond character as shown below
P +
F F
F

As result repulsions between P – F bonds are large and hence the bond angle
is large. There is no possibility of formation of double bonds in PH3.

Problem 3: Though Cs is most electropositive element in periodic table, Li has highest


oxidation potential why?

Solution: A metal ionizes in following way in gaseous state


[Type text]

M ?? M+(g) + 1e– ?H = I.E. …(1)


But in water cation undergoes hydration
M+(g) + nH2O ?? [M(H2O)n]+ ?H = hydration energy …(2)

I.E. has positive value but hydration energy has –ve value. For Cs, I.E. is less
than Li but for Li, hydration energy is more than Cs as Li+ has higher charge
density. The resultant of these two values is more –ve for Li rather than Cs.
Therefore in aqueous solution Li ionizes more than Cs.

Problem 4: A diatomic molecule has a dipole moment of 1.2D. If its bond distance is
1.0Å. What fraction of an electronic charge exist on each atom?

Solution: Dipole moment for a diatomic molecule (?) = e ? d


µ 1.2 × 10 −18 esu.cm
? electronic charge = = −8
= 1.2 ? 10–10 esu
d 1.0 × 10 cm
1.2 × 10 −10
? fraction of electronic charge = = 0.25
4.8 × 10 −10

Problem 5: In trimethylamine, the nitrogen has a pyramidal geometry whereas in


trisilylamine N(SiH3)3, it has a planar geometry. What is the reason behind
this ?

Solution: In N (CH 3 ) 3 , there is sp 3 hybridization at nitrogen but due to lone pair-bond


pair repulsion, shape become pyramidal.

N
Me Me
Me
In trisilyl amine, there is vacant d-orbital at silicon, hence formation of
p π − pπ back bonding takes place and geometry becomes planar.
SiH3 SiH3 SiH3

N N N
H3Si SiH3 H3Si SiH3 H3Si SiH3

Problem 6: Draw the molecular structures of XeF2 and XeF4, indicating the location of
lone pair(s) of electrons.

Solution: Structure of XeF2


Hybridization – sp3d
Shape – linear
F

Xe

F
[Type text]

Structure of XeF4
Hybridization sp 3 d 2
Shape – square planar
F F
Xe
F F

Problem 7: Anhydrous AlCl3 is covalent. From the data given below, predict
whether it would remain covalent or become ionic in aqueous solution
[Ionisation energy for Al = -5137 kJ mol–1, ΔH Hydration for Al 3+ = -4665
kJ mol–1, ΔH Hydration for Cl– = –381 kJ mol–1]
Solution: Total energy released during hydration of ions ( Al 3+ and 3 Cl − ) of
−1
AlCl 3 = −4665 + ( −3 × 381) = -5808 kJ mole . Since the energy released during
hydration is more (5808 kJ mole −1 ) than the ionization energy for Al (5137 kJ
mole −1 ), AlCl 3 would be ionic in aqueous solution.

Problem 8: Dipole moment of HX is 2.59 ? 10–30 coulomb-metre. Bond length of


HX is 1.39Å. Calculate percentage ionic character of molecule.
Solution: dipole moment = q ? d
?cal = q ? 1.39 ? 10–10 coulomb – metre
?cal = 1.6? 10–19? 1.39 ? 10–10
µ exp 2.59 × 10 −30
% age ionic character= × 100 = × 100 = 11.65%
µ cal 1.6 × 10 −19 × 1.36 × 10 −10
Problem 9: Calculate lattice energy of NaCl, from following data using Born-Haber’s
cycle.
Na ( s ) ∆
H sub = +108.4 kJ mol
  → Na ( g )
-1
I.E. = +495.8 kJ mol-1
E.A. = 348 kJ mol-1

1
Cl2 ( g ) ∆
H D = 242kJ mol-1
 → Cl(g )
2

∆H f = −411.2kJ mol -1

Cl− ( g ) Na + (g )
U
NaCl(s)

1
Solution: ?Hf = ?Hsub + ∆H D + IE + EA + U (using Hess' Law)
2
1
– 411.2 = 108.5 + ? 242 + 495.8 – 348 + U
2
U = –788.4 kJ mol–1
Problem 10: Why is that Lithium salts have a greater degree of covalent character than
other halides of the alkali metal.
[Type text]

Solution: The small size of Li+ gives it a large polarizing power, hence covalent nature
of its compound increases. It can be viewed in terms of Fajan’s rule, smaller
the cation, larger the anion, greater the charge density at the surface causes
greater polarizing power of Li+. Hence, greater the covalent nature.
Assignments (New Pattern)
SECTION – I Single Choice Questions

1. Which of the following is required for the formation of an ionic bond?


(a) An electron from the more electronegative element should be transferred to the less
electronegative element.
(b) The total energy of the resulting molecule should be less than the total energy of the
reactants.
(c) The lattice energy of the resultant molecule should be as low as possible.
(d) The ionic potentials of the reactants should be identical.

2. Which of the following statements is incorrect ?


(a) NH3 is more basic than PH3
(b) NH3 has a higher boiling point than that of HF
(c) N2 is less reactive than P4
(d) The dipole moment of NH3 is less than that of SO2.

3. AlCl3 is covalent while AlF3 is ionic. This can be justified on the basis of
(a) the valence-bond theory (b) Fajans’ rules
(c) the molecular-orbital theory (d) hydration energy

4. Which of the following pairs have nearly identical values of bond energy ?
(a) O2 and H2 (b) N2 and CO
(c) F2 and I2 (d) O2 and Cl2

5. Which of the following is the most ionic ?


(a) P4O10 (b) MnO
(c) CrO3 (d) Mn2O7

6. Among LiCl, BeCl2, BeCl3 and CCl4, the covalent bond character varies as
(a) LiCl < BeCl2> BCl3> CCl4 (b) LiCl > BeCl2< BCl3< CCl4
(c) LiCl < BeCl2< BCl3< CCl4 (d) LiCl > BeCl2> BCl3> CCl4

7. In a metallic crystal the


(a) valence electrons remain within the fields of influence of their own kernels
(b) valence electrons constitute a sea of mobile electrons
(c) valence electrons are localized between the two kernels
(d) kernels as well as the electrons move rapidly
[Type text]

8. Polarization involves the distortion of the shape of an anion by an adjacently placed


cation. In this context, which of the following statements is correct ?
(a) Maximum polarization is brought about by a cation of high charge.
(b) Minimum polarization is brought about by a cation of low radius
(c) A large cation is likely to bring about a high degree of polarization
(d) The polarizing power of a cation is less than that of an anion.
9. Which of the following oxyacids of phosphorus are monoprotic (monobasic) ?
(a) H3PO4 (b) H3PO3
(c) H3PO2 (d) H4P2O7

10. Which of the following has greater bond length ?


(a) P—O (b) S—O
(c) Cl—O (d) O=O

11. Which of the following has been arranged in order of increasing covalent character?
(a) KCl < CaCl2< AlCl3< SnCl4 (b) SnCl4< AlCl3< CaCl2< KCl
(c) AlCl3< CaCl2< KCl < SnCl4 (d) CaCl2< SnCl4< KCl < AlCl3

12. Orthonitrophenol is steam volatile but paranitrophenol is not because


(a) orthonitrophenol has intramolecular hydrogen bonding while paranitrophenol has
intermolecular hydrogen bonding.
(b) both ortho and paranitrophenol have intramolecular hydrogen bonding.
(c) orthonitrophenol has intermolecular hydrogen bonding and paranitrophenol has
intramolecular hydrogen bonding.
(d) Van der Waals forces are dominant in orthonitrophenol.

13. During the formation of a molecular orbital from atomic orbitals, the electron density is
(a) minimum in the nodal plane (b) maximum in the nodal plane
(c) zero in the nodal plane (d) zero on the surface of the lobe

14. Which of the following have been arranged in increasing order of bond order as well as bond
dissociation energy ?
(a) O 2−2 < O 2− < O +2 < O 2 (b) O −22 < O −2 < O 2 < O +2
(c) O 2 < O 2+ < O 22− < O −2 (d) O 2+ < O 22− < O 2− < O 2

15. How many sigma and pi bonds are present in tetracyanoethylene ?


(a) Nine σ and nine π (b) Five π and nine σ
(c) Nine σ and seven π (d) Eight σ and eight π

16. Which has maximum ionic mobility


(a) Li+ (b) Na+
(c) K+ (d) Cs+
[Type text]

17. In PO −43 , P—O bond order is


(a) 1.25 (b) 2
(c) –0.75 (d) –3

18. Which of the following has least covalent P—H bond?


(a) PH3 (b) P2H6
(c) P2H5 (d) PH6+

19. Which of the following diatomic molecules would be stabilized by the removal of an
electron?
(a) O2 (b) CN
(c) N2 (d) C2

20. In which of the following species the bonds are non-directional?


(a) NCl3 (b) RbCl
(c) BeCl2 (d) BCl3

21. Which contains both polar and non-polar bonds?


(a) NH4Cl (b) HCN
(c) H2O2 (d) CH4

22. The bond angle between two hybrid orbitals is 105°. The percentage s-character of hybrid
orbital is between
(a) 50 – 55% (b) 9 – 12%
(c) 22 – 23 % (d) 11 – 12%

23. The first I.E. of lithium is 5.41 eV and first electron affinity of Cl is 3.61 eV. Calculate
?H in kJ mol–1 for the reaction:
Li(g) + Cl(g)?? Li(g)
+ −
+ Cl(g)
(a) 133.6 kJ (b) 347.4 kJ
(c) 173.7 kJ (d) None of these

24. which type of bond is not present in HNO2 molecule?


(a) Covalent (b) Co-ordinate
(c) Ionic (d) Ionic as well as co-ordinate

25. KF combines with HF to form KHF2. The compound contains the species
(a) K+, F– and H+ (b) K+, F– and HF
+ –
(c) K and [HF2] (d) [KHF]+ and F2

SECTION – II May be more than one choice


[Type text]

1. There is no S—S bond in:


(a) S2 O 24− (b) S2 O 52−
(c) S2 O 32− (d) S2 O 72−

2. Among KO2, Al O −2 , BaO2 and N O +2 , unpaired electron is present in


(a) N O +2 and BaO2 (b) KO2 and Al O −2
(c) KO2 only (d) BaO2 only

3. Angle between two hybridized orbital is 105o and hence the percentage of s-character in
the hybridized orbital would be in the range
(a) 23–24% (b) 20–21%
(c) 19–20% (d) 21–22%
+
4. There are four species CO2, N 3− , N O2 and I 3− . Which of the following statement is
correct about these species?
(a) All are linear and having sp hybridization of central atoms
(b) All are linear but only CO2 and N 3− and I 3− have sp hybridization on their central
atom.
+
(c) All are linear but only CO2, N 3− and N O2 have sp hybridization on their central atom.
+
(d) CO2, N 3− , N O2 are linear but I 3− is not

5. Ifone assumes linear structure instead of bent structure for water, then which one of the
following properties cannot be explained
(a) The formation of intermolecular hydrogen bond in water.
(b) The high boiling point of water.
(c) Solubility of polar compounds in water.
(d) Ability of water to form co-ordinate covalent bond.

6. The shapes of PCl 4+ , PCl −4 and AsCl5 are respectively


(a) square planar, tetrahedral and see-saw.
(b) tetrahedral, see-saw and trigonal bipyramidal.
(c) tetrahedral, square planar and pentagonal bipyramidal.
(d) trigonal bipyramidal, tetrahedral and square pyramidal.

7. The I 3− ion has


(a) five equatorial lone pair on the central I atom and two axial bonding pairs in a
trigonal bipyramidal arrangement.
(b) five equatorial lone pair on the central I atom and two axial bonding pairs in a
pentagonal bipyramidal arrangement.
(c) three equatorial lone pair on the central I atom and two axial bonding pairs in a
trigonal bipyramidal arrangement.
[Type text]

(d) two equatorial lone pairs on the central I atom and three axial bonding pairs in a
trigonal bipyramidal arrangement.

8. Dipole moment of H2O is 1.84 D. If the bond angle is 105° and O—H bond length is
0.94Ao, what is magnitude of charge on the oxygen atom in water molecule.
(a) 2 ? 10–10 esu (b) 3.28 ? 10–10 esu
(c) 3.22 ? 10–10 esu (d) 1.602 ? 10–19 coulomb

+ −
9.  PCl 4 + PCl6 , the change in hybridization is from
In the reaction 2PCl5  
(a) sp3d to sp3 and sp3d2 (b) sp3d to sp2 and sp3
(c) sp3d to sp3d2 and sp3d3 (d) sp3d2 to sp3 and sp3d

10. A diatomic molecule has a dipole moment of 1.2 D. If its bond distance is 1.0A°, what
fraction of an electronic charge, e, exists on each atom?
(a) 11% (b) 20%
(c) 25% (d) none of these
11. Specify the coordination geometry around and hybridisation of N and B atoms in a 1 : 1
complex of BF3 and NH3
(a) A : tetrahedral, sp3; B : tetrahedral, sp3
(b) N : pyramidal, sp3; B : pyramidal, sp3
(c) N : pyramidal, sp3; B : planar, sp2
(d) N : pyramidal, sp3; B : tetrahedral, sp3

12. Select correct orders for corresponding property as indicated in bracket for the following.
(a) NH3 ? BiH3? SbH3? AsH3? PH3 (Boiling point)
(b) H2O ? H2Te ? H2Se ? H2S (Boiling point)
(c) NH3? PH3? AsH3? SbH3 (Basic character)
(d) H2O ? H2S ? H2Se ? H2Te (Acidic character)

13. Which one or more among the following involve(s) p? - d? bonding?


(a) (SiH 3 ) 3 N (b) (CH 3 ) 3 N

(c) CCl3 (d) CF3

14. Choose the correct choice(s):


(a) [Fe(H2O)5NO]SO4 is paramagnetic with a moment of about 6.0 B.M.
(b) [Fe(H2O)5NO]SO4 is paramagnetic with a moment of about 4.0 B.M.
(c) [Fe(H2O)5NO]SO4 is paramagnetic with a moment of about 1.7 B.M.
(d) [Fe(H2O)5NO]SO4 is an octahedral complex formed by sp3d2 hybridisation.

15. Paramagnetic pair(s) among the following is (are):


[Type text]

(a) [BaO2, NO2] (b) [KO2, NO]


(c) [H2O2, NO] (d) [K3(CN)6, CuCl2]

16. Which of the following have tetrahedral structure?


(a) CrO42– (b) [Ni(CN)4]2–
(c) [Ni(CO)6] (d) [NiCl4]2–

17. Which of the following orders are correct for property indicated in brackets
(a) NH3? NF3? BF3 (dipole moment)
(b) Cl ? S ? O ? N (electron affinity)
(c) Si ? Mg ? Al ? Na (first ionization enthalpy)
(d) HClO4? HBrO4? HIO4 (pKa values)

18. Which among the following are diamagnetic?


(a) [Fe(CN)6]3– (b) [Fe(CN)6]4–
(c) [Ni(CN)4]2– (d) [Ni(CO)4

19. Which among the following are having sp3d hybridization of the central atom.
(a) XeF4 (b) XeO2F2
(c) ClO3? (d) BrF3

20. Which of the following statements are correct:


(a) The bond angle of NCl3 is greater than that of NH3.
(b) The bond angle in PH3 is greater than that of PF3.
(c) ClO3− and SO32− are isostructural
(d) It is not necessary that in TBP structure the lone pairs always would occupy the
equatorial positions.

21. Bond angle in PH3 is


(a) much less than NH3 (b) Much less than PF3
(c) slightly more than NH3 (d) much more than PF3

22. Which among the following are isostructural


(a) CO2, I3− (b) XeO2F2, SF4
(c) SO 32 − , CO32 − (d) ClF3, XeF2

23. Stability of ions of Ge, Sn and Pb will be in the order


(a) Ge2+? Sn2+? Pb2+ (b) Ge 4 + > Sn 4 + > Pb 4 +
(c) Sn4+? Sn2+ (d) Pb2+? Pb4+

24. Select the correct statements:


[Type text]

(a) the heat of hydration of the dipositive alkaline earth metals ions decrease with an
increase in their ionic size.
(b) hydration of alkali metal ions is less than that of IIA
(c) alkaline earth metal ions, because of their much larger charge to size ratio exert a
much stronger electrostatic attraction on the oxygen of water molecule surrounding
them.
(d) melting point of sodium halides follow order
NaF ? NaCl ? NaBr ? NaI

25. The first element of groups 13-16 differ from rest of the elements. This is due to
(a) small size and high electronegativity
(b) inabilitiy to expand to the octet
(c) ability to form strong p? - p? multiple bonds
(d) Due to greater abundance

SECTION – III Comprehension Type Questions


? Write-up I
The shape of a molecule is determined by electron-pair repulsions in the valence shell. A
lone pair occupies larger space than a bond pair because it is not shared by two nuclei.
Thus the lone pair-lone pair repulsion is greater than the lone pair-bond pair repulsion,
which in trun is greater than the bond pair-bond pair repulsion. The presence of lone
pairs causes distortion of bond angles and hence a deviation from an ideal shape. The
extent of distortion depends upon the orientation of the lone pairs around the central
atom. In a trigonal bipyramid, the lone pairs occupy equatorial positions than the apical
ones.
In ABn type molecules, as the electrongativity of A increases, the bond pairs come closer
and the repulsion between them increases. On the other hand, as electronegativity of B
increases, the bond pairs get farther and repulsion decreases.

1. Which of the following statements is true?


(a) F-N-F angle in NF3 is greater than H-N-H angle in NH3
(b) F-N-F angle in NF3 is smaller than H-N-H angle in NH3
(c) H-O-H angle in H2O is greater than H-N-H angle in NH3
(d) F-O-F angle in F2O is greater than H-O-H angle in H2O

2. Bond angle in which of the following molecules is the largest?


(a) PF3 (b) PCl3
(c) PBr3 (d) PI3

3. The shape of which of the following molecules will not be distorted?


(a) BrF3 (b) CiF3
(c) XeF4 (d) XeF6

4. Which of the following species will have the lone pair effects cancelled?
(a) ICl −2 (b) ClF3
(c) PCl3 (d) BrF5
[Type text]

? Write-up II
The molecular orbital energy diagrams for homonuclear diatomic molecules are given
below. For systems containing up to 14 electrons:
For systems containing more than 14 electrons:
Electrons taken from both the atoms are filled from lower to higher energy molecular
orbitals (MO’s) following Hund’s rules.
The energy diagram of a heteronuculear diatomic molecule is similar. However, the
energies of the atomic orbitals (AO’s) of the atom having higher atomic number being
lower, the diagram will be unsymmetrical, but that will not make a difference in the
electron count.
Bond order is given by half the difference in the number of electrons of the bonding (?
and ?) and anti-bonding (?* and ?*) molecular orbitals. For a bond to have been formed,
the bond order should be greater than zero. The greater the bond order, the shorter is the
bond distance and the greater is the bond dissociation energy. But if the bond order is the
same in two cases, the bond distance will be greater and the bond dissociation energy
smaller in the case which has more populated anti-bonding orbitals. The presence of
unpaired electron(s) in a molecular orbital will make the system paramagnetic.

5. Which of the following species is not expected to exist?


(a) He+2 (b) H +2
(c) Be2 (d) Be+2

6. Which among the following will have a triple bond order?


(a) CO (b) CN–
(c) NO+ (d) All of these

7. Which of the following orders is correct in respect of bond dissociation energy?


(a) N +2 > N 2− (b) O +2 > O −2
(c) NO+ > NO (d) All of these

8. Which of the following species is expected to be paramagnetic?


(a) NO– (b) O −2
(c) O +2 (d) All of these

? Write-up III

Hydrogen bond is a feeble bond formed between hydrogen atoms and highly
elecgronegative elements. It is of two types – intermolecular, intramolecular. It is weaker
than ionic, covalent and metallic bonds.

9. Number of hydrogen bonds in H 9 O +4 species


(a) 2 (b) 3
(c) 4 (d) 1
[Type text]

10. Which is correct statement


(a) Para nitrophenol is steam volatile but not orthnitrophenol
(b) Ethyl alcohol is more viscous than glycerol
(c) if a dry paper is torn sound is heard is is due to breaking of hydrogen bonds one after
another in a rhythmatic manner.
(d) In Fermic salt bonds present are covalent, metallic and hydrogen bond

11. Which is correct statement?


(a) Keto form of acetoacetic ester involves in hydrogen bonding
(b) In water vapour hydrogen bonding exists
(c) For first ionization maleic acid is stronger acid than fumaric acid. This can be
explained on basis of concept of hydrogen bonding
(d) Boiling point of HCl is higher than that of HF
12. In a suitable solvent like benzene, benzoic acid associates and exists as
(a) dimer (b) trimer
(c) tetramer (d) hexamer

? Write-up IV

In the ionic bond, a cation tend to polarize the electron cloud of the anion by pulling
electron density towards itself. This causes development of covalent character in ionic
bond because the electron density gets localized in between the nuclei. The tendency of
cation to bring about the polarization of anion is expressed as its polarizing power. The
ability of ion to undergo polarization is called its polarisability. The polarizing power of
cation and polarisibality of anion are decided on the basis of Fajan rules as given below:
i) The smaller the cation, the higher is its polarizing power.
ii) Cations with pseudo noble gas configuration (ns2np6nd10) having relatively high
polarizing power than those with noble gas configuration (ns2np6).
iii) The larger the size of the anion, the higher is its polarisability.
13. Arrange the following species in increasing order of polarizing power?
Ag+, Tl+, Na+
(a) Tl+? Ag+? Na+ (b) Tl+? Na+? Ag+
(c) Ag+? Tl+? Na+ (d) Na+? Tl+? Ag+

14. Among the following LiCl, BeCl2, BCl3, CCl4 which will have the lowest melting point
and the highest solubility
(a) CCl4, LiCl (b) LiCl, CCl4
(c) BeCl3, BCl3 (d) BCl3, BeCl2

15. The correct order of polarisibility is


I–, Br–, Cl–, F–
[Type text]

(a) I–? Br–? Cl–? F– (b) I–? Br– = Cl–? F–

(c) I– = Br– = Cl–? F– (d) I– = Br–? Cl– = F–

16. The ionic conductance of which of the following is the highest?


(a) Li+ (aq) (b) Na+ (aq)
(c) K+ (aq) (d) Cs+ (aq)

? Write-up V
To explain the nature of bonding beautifully in transition metal complexes, out of lot of
theories, valence bond theory (VBT) has great importance which is based on the theory
of covalent bonding. It deals with electronic structures of central metal ion in its ground
state, kinds of geometry and magnetic properties of the complex.
According to VBT
i) The central metal ion or atom makes available a number of s, p & d atomic orbitals
equal to its co-ordination number. First of all, these atomic orbital hybridise
together to form hybrid orbital which are same in number as the atomic orbital
hybridizing together. These are vacant, equivalent in energy and have definite
geometry.
ii) The complexes having unpaired electron is known as para-magnetic complex &
having no unpaired electron is known as diamagnetic complex.
Magnetic moment due to spin only = µspin only = n(n + 2) B.M .
Where, n = no. of unpaired electron.
iii) Nature of ligands, also influences the nature of the complex compounds. Stronger
ligands make pairing of electron against Hund’s rule, and generally produces inner
orbitals complex are complexes in which ligands use the inner d-orbital of central
metal ion.
Weaker ligands does not make pairing of electron and obey’s Hund’s rule and generally
produces outer d-orbital complexes.
Keep in mind that the chelated complex is more stable than that of the non-chelated
complex.

17. Which of the following is not the square planar complex?


(a) [Ni(CN) 4 ]−2 (b) [Cu(NH3)4]2+
(c) [Pt(NH3)4]2+ (d) [Ni(CO)4]

18. In square planar complex, which of the folloiwng d-orbitals are used up?
(a) d x 2 − y2 (b) d z2
(c) dxy (d) dxy

19. What is the magnetic moment (spin only) and hybridisation of the brown ring complex
[Type text]

[Fe(H2O)5NO]SO4?
(a) 3 B.M. , sp3d2 (b) 3 B.M. , d2sp3
(c) 15 B.M. , sp3d2 (d) 15 B.M. , d2sp3

20. Which of the following complex is more stable?


(a) [Cr(en)3]2+ (b) [Cr(CN)6]3–
(c) [Cr(H2O)6]3+ (d) [Cr(EDTA)]–

Answer to Assignments
SECTION – I

LEVEL – I
1. (b) 2. (b) 3. (b) 4.
(d)

5. (b) 6. (c) 7. (b) 8.


(a)

9. (c) 10. (a) 11. (a) 12.


(a)

13. (c) 14. (b) 15. (a) 16.


(d)

17. (a) 18. (d) 19. (a) 20.


(b)

21. (c) 22. (c) 23. (c) 24.


(d)

25. (c)

SECTION – II
1. (d) 2. (a) 3. (a) 4. (c)

5. (c) 6. (b) 7. (c) 8. (c)

9. (a) 10. (c) 11. (a) 12. (b), (c),


(d)
[Type text]

13. (a), (c) 14. (b), (d) 15. (b), (d) 16. (a), (c),
(d) 17. (a), (b), (c) 18. (b), (c), (d) 19. (b), (d) 20.
(a), (c)
21. (a), (b) 22. (a), (b) 23. (a), (c), (d) 24. (a), (b), (c), (d)
25. (a), (b), (c)

SECTION – III

1. (b) 2. (d) 3. (c) 4. (a)


5. (c) 6. (d) 7. (d) 8. (d)
9. H H

H O O H
H H
O

O
H H

?(b)
10. Orthonitrophenol (intramolecular hydrogen bonding) more volatile than para nitrophenol
(intermolecular hydrogen bonding)
? (c)
11. Enol form of acetoacetic ester has hydrogen bond. In water vapour no hydrogen bond
exists. Boiling point of HF ? Boiling point of HCl (due to molecular association in HF
due to hydrogen bonding). The ion formed after removal of first proton from maleic acid
is stabilized by hydrogen bonding.
?(c)
12. Benzoic acid exists as dimer due to presence of inter molecular hydrogen bonding.
?(a)
13. (c) 14. (a)
15. (a) 16. (d)
17. (d) 18. (a)
19. (c) 20. (d)
[Type text]

Gaseous state
The Gas Laws
Boyle's Law: It relates the volume and the pressure of a given mass of a gas at constant
temperature.

The relationship between the volume and the pressure of a gas was studied by Robert Boyle in
1662. He found that increasing the pressure at constant temperature on a sample of a gas causes
the volume of the gas to decrease proportionately, i.e., if the pressure is doubled, the volume
becomes half and so on. Boyle's law states that, “At constant temperature, the volume of a
sample of a gas varies inversely with the pressure”.

1
∴ P∝ (when temperature and number of moles are kept constant)
V

The proportionality can be changed into an equality by introducing a constant k, i.e.,

k
P= or PV = k
V

Alternatively, Boyle's law can also be stated as follows:-

“Temperature remaining constant, the product of pressure and volume of a given mass of a gas is
constant”.

The value of the constant depends upon the amount of a gas and the temperature.

Mathematically, it can be written as,

P1V1 = P2 V2 = P3 V3 = ...

Boyle’s law can be verified by any one of the following three ways graphically.
Temperature-constant Temperature-constant Temperature-constant
Mass-constant Mass-constant Mass-constant

V V PV

P 1/P P
(A) (B) (C)
The first curve shows the variation of volume of a given mass of gas with pressure at constant
temperature. The shape of the curve is rectangular hyperbola. This curve is also called isotherm.
[Type text]

The second curve showing the relationship between volume and reciprocal of pressure is a
straight line. It confirms the statement that at constant temperature, volume of a given mass of
gas is inversely proportional to the pressure. The third curve shows a straight line parallel to
pressure-axis. This confirms that the product of pressure and volume of a given mass of a gas at
constant temperature is constant.

Location of straight line and curve changes with temperature in the isotherm shown in the
following figure.
P T1 P PV
T3
T1 > T2 > T3
T2 T2
T3
T1 (T3 > T2 > T1)
T1
T2
T3

1/V V P
(D) (E) (F)

According to Boyle’s law, PV = Constant at constant temperature

∴ log P + log V = constant


log P = –log V = constant
log P

log V
(G)

Illustration 1: A gas is present at a pressure of 2 atm. What should be the increase in


pressure so that the volume of the gas can be decreased to 1/4th of the initial
value if the temperature is maintained constant.

Solution: PV = constant for a given mass of gas at constant pressure


P1V1 = P2 V2
P1 = 2atm
Let, V1 = V
V2 = V/4
V
Now, 2 × V = P2 ×
4
P2 = 8 atm
∴ Pressure should be increased from 2 to 8 atm
[Type text]

∴ total increase = 8 – 2 = 6 atm.


BRAIN TEASER 1:
A sample of gas occupies 10 litre under a pressure of 1 atmosphere. What will be its
volume if the pressure is increased to 2 atmospheres? Assume that the temperature of
the gas sample does not change.

Charles’ Law: It relates the volume and temperature of a given mass of a gas at constant
pressure.

Experiments have shown that when 273 mL sample of a gas at 0°C is heated to 1°C, its volume
increases by 1 mL, i.e., it becomes 274 mL. At 5°C, the volume increases to 278 mL if the
pressure remains constant in both cases. Similarly, when 273 mL sample of gas at 0°C is cooled
to –1°C, its volume decreases to 272 mL while at –5°C, the volume decreases to 268 mL if the
pressure remains constant.

Thus, all gases expand or contract by the same fraction of their volume at 0°C per degree change
of temperature, i.e., for each degree change of temperature, the volume of a sample of a gas
1
changes by the fraction of its volume at 0°C.
273

Let the volume of a given amount of gas be V0 at 0°C. The temperature is increased by t°C and
the new volume becomes Vt

V0  t 
Thus, Vt = V0 + × t = V0  1 + 
273  273 
 273 + t 
or Vt = V0   … (i)
 273 

A new temperature scale was introduced known as Kelvin scale or absolute scale named after the
British physicist and mathematician Lord Kelvin. The lower limit of the scale is called absolute
zero which corresponds to –273°C. At absolute zero or –273°C, all molecular motions would
stop and the volume of the gas would become zero. The gas would become a liquid or solid.
Thus, absolute zero is that temperature at which no substance exists in the gaseous state. The
temperature in absolute scale is always obtained by adding 273 to the temperature expressed
in °C.
K = (t°C + 273)

The concept of absolute zero of temperature can also be derived from the laws of
thermodynamics. According to the first law of thermodynamics, the absolute zero of temperature
is the lowest possible temperature on the thermodynamic scale and according to the second law
of thermodynamics, this temperature can't be attained.

This new temperature scale may be used for deducing Charles' law.

By substituting T for 273 + t and T0 for 273 in Eq. (i).


[Type text]

V0 × T
Vt =
T0
Vt V0
or =
T T0
V
or = constant, if pressure is kept constant
T

2.0
Volume (litre)

1.0

0 -200 -100 0 100 200 300 °C


73 173 273 373 473 573 K
Temperature

This is Charles’ law. It can be stated as follows:

“The volume of a given amount of a gas at constant pressure varies directly as its absolute
temperature”.

V ∝ T (if pressure is kept constant)

Charles' law can be verified experimentally by plotting the values of volumes of a given amount
of a gas under respective absolute temperature at constant pressure. The straight line confirms
the above statement.

Pressure-Temperature Law: It relates the pressure and absolute temperature of a given


mass of a gas at constant volume.

1
Volumes remaining constant, the pressure of given mass of a gas increases or decreases by
273
of its pressure at 0°C per degree change of temperature.

P0 × t
Pt = P0 +
273
 t 
or Pt = P0  1 + 
 273 
 273 + t  T
or Pt = P0   = P0
 273  T0
Pt P0
or =
T T0
[Type text]

or P ∝ T (if volume and number of moles are kept constant)


At constant volume, the pressure of a given amount of a gas is directly proportional to its
absolute temperature.

Avogadro’s Law
In 1812, Amadeo Avogadro stated that samples of different gases which contain the same
number of molecules (any complexity, size, shape) occupy the same volume at the same
temperature and pressure. It follows from Avogadro's hypothesis that V ∝ n (T and P are
constant).

Ideal Gas Equation: Combining all these gas laws, a simple equation can be derived at,
which relates P, V, n and T for a gas

Eq. PV = nRT (for n moles of gas)

This is called ideal Gas Equation.

P is the pressure of the gas and can be expressed in atm or Pa. Correspondingly, the volume must
be expressed in litres or m3 respectively. n is the number of moles and T is the temperature in
Kelvin. R is called the universal gas constant.

Numerical Values of R

i) In litre atmosphere = 0.0821 litre atm deg–1 mole–1


ii) In ergs = 8.314 ×107 erg deg –1 mole –1
iii) In calories = 1.937 cal deg –1 mole –1

Illustration 2: An open vessel at 27°C is heated until 3/5th of the air in it has been expelled.
Assuming that the volume of the vessel remains constant find
a) the air escaped out if vessel is heated to 900K.
b) temperature at which half of the air escapes out.

Solution: One should clearly note the fact that on heating a gas in a vessel, there are the
number of moles of gas which go out, the volume of vessel remains constant.
Let initial no. of moles of gas at 300 K be `n’. On heating 3/5 moles of air
escape out at temperature T.
 3  2n
∴ Moles of air left at temperature T =  n – n  =
 5  5

a) On heating vessel to 900 K, let n1 moles be left, again n1T1 = n2T2


? n1 × 900 = 300 × n
1
⇒ n1 = n
3
[Type text]

n 2
∴ moles escaped out = n – = n moles
3 3
b) Let n/2 moles escape out at temperature T, then
n1 T1 = n2 T2
n
× T = n × 300
2
T = 600 K
BRAIN TEASER 2:
5g of ethane is confined in a bulb of one litre capacity. The bulb is so weak that it will
burst if the pressure exceeds 10 atm. At what temperature will the pressure of gas
reach the bursting value?

Relation between Molecular Mass and Gas Densities

w
Actual density: For an ideal gas PV = nRT or PV = RT , where w = mass of the gas in gms
M
and M = Molecular wt. in gms.
w
∴ PM= RT
V
w
or PM = ρ RT, (where ρ is the density of the gas =
V
PM
∴ ρ=
RT

Illustration 3: The density of an unknown gas at 98°C and 0.974 atm is 2.5 × 10–3 g/ml.
What is the mol wt. of gas?

Solution: Density = 2.5 × 10 –3 g/ml = 2.5 g/L


∴ PM = ρ RT
0.974 × M = 2.5 × 0.0821 × 371
⇒ M = 78.18.

Vapour Density: For gases another term which is often used is vapour-density. Vapour density
of a gas is defined as the ratio of the mass of the gas occupying a certain volume at a certain
temperature and pressure to the mass of hydrogen occupying the same volume at the same
PVM
temperature and pressure i.e. W (gas) = .
RT

PV × 2
and WH 2 = (∴ mol. wt. of hydrogen is 2)
RT
Wgas M
∴ = = (Vapour density of gas)
WH2 2
[Type text]

Vapour density of a gas is same at any temperature, pressure and volume.


BRAIN TEASER 3:
When 3.2 g of sulphur is vapourised at 450°C and 723 mm pressure, the vapours
occupy a volume of 780 ml. What is the mol. formula of sulphur vapours under these
conditions? Calculate the vapour density also.

Molecular Weight and Effective Molecular Weight


Molecular weight of a gas mixture or effective molecular weight:Suppose we
have to find the molecular weight of air and we are told that air contains 79% nitrogen and 21%
oxygen (by mole or volume).

First of all let us understand what is meant by molecular wt. of a gas. Molecular wt. of a gas is
the weight in gms of one mole of the gas (hence the unit gm/mole). Now if we take one mole of
air, it would contain 79/100 moles of nitrogen and 21/100 moles of oxygen. The weight of one
mole of air would be 0.79 × 28 + 0.21 × 32 = 28.84 gm/mole.

No. of moles of the gas


∴ (Mole fraction) of a gas is =
Total no. of moles of all the gas

Dalton's Law of Partial Pressures: The total pressure of a mixture of non-reacting gases
is equal to the sum of their partial pressures. By the partial pressure of a gas in a mixture is
meant, the pressure that the gas will exert if it occupies alone the total volume of the mixture at
the same temperature.

Consider n1 moles of gas 1 and n2 moles of gas 2 occupying a vessel of volume V at temperature
T K and exerting a total pressure P.

Let n1 moles of gas 1 alone occupy the same vessel of capacity V at the same temperature T K
and exert a pressure P1. Then by definition P1 is partial pressure of gas 1 in the mixture. Let n2
moles of gas 2 alone occupy the same volume V at the same temperature TK and exert a pressure
P2. The P2 is the partial pressure of gas 2 in the mixture.

By Dalton’s Law P = P1 + P2

Derivation: n = n1 + n2 + …
PV
n=
RT

PV P1V P2 V
= + + ...
RT RT RT

P = P1 + P2 + …

Assumption: Volume of all the gases is same.


[Type text]

Relationship between partial pressure and number of moles

Important formula
 n1 
i) P1 =   P = x1P where x1 = mole fraction of gas
 n1 + n 2 
Volume of the gas
ii) Partial pressure of a gas in the mixture = ×P
Total Volume

Partial pressure and aqueous tension: Dalton's law is used to calculate the pressure of
a dry gas when it is collected over water at atmospheric pressure. By Dalton's law,

Pressure of dry gas = atmospheric pressure - aqueous tension

Aqueous tension depends on temperature. It increases with temperature and becomes 760 mm at
100°C.

Illustration 4: A gaseous mixture contains 55% N2, 20% O2, and 25% CO2 by mass at a
total pressure of 760 mm. Calculate the partial pressure of each gas.

Solution: Total mass of the gases = 100 g


Mass of N2 = 55g,
Mass of O2 = 20 g,
Mass of CO2 = 25g
Moles of N2 = 55/28 = 1.964
Moles of O2 = 20/32 = 0.625
Moles of CO2 = 25/44 = 0.568
Total moles = 3.157
1.964 × 760
PN 2 = = 472.8 mm
3.157
0.625 × 760
PO 2 = = 150.46 mm
3.157
0.568 × 760
PCO2 = = 136.74 mm
3.157
BRAIN TEASER 4:
A mixture containing 1.6 g of O2, 1.4g of N2 and 0.4 g of He occupies a volume of 10
litre at 27°C. Calculate the total pressure of the mixture and partial pressure of each
compound.

Graham’s Law of Diffusion


Diffusion is the tendency of any substance to spread throughout the space available to it.
Diffusion will take place in all direction and even against gravity. So gases diffuse through firm
[Type text]

substances and through small holes. The streaming of gas molecules through a small hole is
called effusion.

According to Graham, the rate of diffusion (or effusion) of a gas at constant pressure and
temperature is inversely proportional to the square root of its molecular mass.

1
r∝ , at constant P and T
M
r1 M2
∴ = , at constant P and T
r2 M1
r1 d2
Since molecular mass of gas = 2 × vapour density, = , at constant P and T
r2 d1
The rate of diffusion (or effusion) r of two gases under different pressure can be given by
r1 M 2 P1
= × at constant T only.
r2 M1 P2
Volume diffused (V) moles diffused (n)
r= =
time taken time taken

distance travelled in a narrow tube of uniform cross-sectional area (x)


r=
time taken

Therefore, according to Graham’s law of diffusion (effusion) at constant P and T.

V1 t 2 d2 M2
× = = ;d1 and d2 are the respective densities
t1 V2 d1 M1

where V1 and V2 are volumes diffused (effused) in time t1 and t2 .

n1 t 2 d2 M2
× = =
t1 n 2 d1 M1

where n1, n2 are moles diffused (effused) in time t1 and t2.

x1 t 2 d2 M2
× = =
t1 x 2 d1 M1

where x1 and x2 are distances travelled by molecules in narrow tube in time t1 and t2.

Illustration 5: Pure O2 diffuses through an aperture in 224 seconds, whereas mixture of O2


and another gas containing 80% O2 diffuses from the same in 234 sec under
[Type text]

similar condition of pressure and temperature. What is molecular wt. of


gas?

Solution: The gaseous mixture contains 80% O2 and 20% gas.


32 × 80 + 20 × mmix
∴Average molecular weight of mixture (M mix ) = … (i)
100
Now for diffusion of gaseous mixture and pure O2
rO2 M mix VO2 t mix M mix
= or × =
rm M O2 Vmix t O2 32

1 234 M mix
or × = … (ii)
224 1 32
∴ M mix = 34.92
By (i) and (ii) mol weight of gas (m) =46.6.
BRAIN TEASER 5:
235 238
Calculate the relative rates of diffusion of UF6 and UF6 in the gaseous state
(Atomic mass of F = 19).

The Kinetic Theory of Gases


In order to derive the theoretical aspect of the various gas laws based on simple experiment facts,
Maxwell proposed the following postulates under the heading of kinetic theory of gases:

The postulates of kinetic theory of gas are

i) Each gas is made up of a large number of small (tiny) particles known as molecules.

ii) The volume of a molecule is so small that it may be neglected in comparison to total
volume of gas.

iii) The molecules are never in stationary state but they are believed to be in chaotic
(random) motion. They travel in straight line in all possible directions with altogether
different but constant velocities. The direction of motion is changed by the collision with
container or with the other molecules.

iv) The collision between molecules is perfectly elastic i.e., there is no change in the energies
of the molecules after collision.

v) The effect of gravity on molecular motion is negligible.

vi) The kinetic energy of the gases depends on the temperature.

vii) The pressure of the gas arises due to collision of molecules with the walls of the
container.
[Type text]

The Kinetic Equation: Maxwell also derived an equation on the basis of above
assumptions as
1
PV = mnu 2
3
where P = Pressure of gas
V = Volume of gas
m = mass of one molecule of gas
n = no. of molecules of gas
u = root mean square velocity of molecules
For 1 mole n = N (Avogadro number)
m × N = Molecular mass M.
1 3PV 3RT 3PV 3RT
∴ PV = Mu 2 or u 2 = = or u = =
3 M M M M

Distribution of Molecular Velocities


Maxwell and Boltzmann proposed that gas molecules are always in rapid random motion
colliding with each other and with the walls of container. Due to such collisions, their velocities
always changes. A fraction of molecules have a particular molecular velocity at a time. James
Clark Maxwell calculated the distribution of velocity among fraction of total number of
molecules, on the basis of probability.

The distribution of velocities of different gas molecules may be shown by the following curve.
Most probable
velocity
Fraction of Molecules

Average velocity
273 K
1273 K

2773 K

Molecular velocity

From the curve it may be concluded that

i) Only a small fraction of molecules have either very low or very high velocity.

ii) Curve becomes flat when temperature is raised i.e. distribution around average velocity
becomes wider. Average molecular velocity increases with rise in temperature.

iii) Most of the molecules have velocity close to most probable velocity represented by the
top of curve.

iv) At higher temperature greater number of molecules have high velocity, while few
molecules have lower velocity.
[Type text]

Average Velocity: As per kinetic theory of gases, each molecule is moving with altogether
different velocity. Let `n’ molecules be present in a given mass of gas, each one moving with
velocity u1,u2, u3, …,un. The average velocity or U aV = average of all such velocity terms.

u1 + u 2 + u 2 + ...u n
Average velocity =
n
8RT 2.55RT
U av = =
πM M

Root Mean Square Velocity: Maxwell proposed the term Urms as the square root of
means of square of all such velocities.

u12 + u 22 + u 32 + ...
U 2rms =
n
3RT 3PV 3P
Also U rms = = = where, D = density of the gas.
M M D

Most probable velocity: It is the velocity possessed by maximum no. of molecules.

2RT
U mpv =
M

2RT 8RT 3RT


Furthermore U mpv : Uav : U rms :: : :
M πM M
8
= 2: : 3 = 1 : 1.128 : 1.224
π
Also Uav = Urms × 0.9213
1
Kinetic Energy of Gas: As per kinetic equation PV = mn u 2rms .
3
For 1 mole m × n = Molecular Mass (M)
1 2 1 2 K.E. 3
∴ PV = Mu 2rms = × Mu 2 = × K.E. / mole or = RT
3 3 2 3 mole 2
3 RT 3  R
Also KE per molecule = = kT. Where k is the Boltzmann constant  k = 
2 n 2  n
5
Illustration 5: Calculate rms speed of O2 at 273 K and 1 × 10 Pa pressure. The density of
O2 under these conditions is 1.42 kg m–3.
Solution: Data are given in SI units
3P 3 ×105
C= = = 459.63 m sec –1
d 1.42
[Type text]

Illustration 6: At what temperature will the r.m.s. velocity of oxygen be one and half times
of its value at N.T.P.?

1 2 3
Solution: mc = kT
2 2
3
Suppose the temperature required is T′ then the velocity will be C
2
1 3
? mc′2 = kT′
2 2
3 / 2C T′ 9
? = or, T′ = × 273 = 614.25° K
C T 4
BRAIN TEASER 6:
Calculate the average and total kinetic energy of 0.5 mole of an ideal gas at 0°C.
Deviations from Ideal Behaviour: An ideal gas is one which obeys the gas laws of the
gas equation PV = RT at all pressure and temperatures. However no gas in nature is ideal.
Almost all gases show significant deviations from the ideal behaviour. Thus the gases H2, N2 and
CO2 which fail to obey the ideal-gas equation are termed as non-ideal or real gases.

Compressibility Factor: The extent to which a real gas departs from the ideal behaviour
may be depicted in terms of a new function called the compressibility factor, denoted by Z. It is
defined as
PV
Z=
nRT

The deviations from ideality may be shown by a plot of the compressibility factor Z, against P.

For an ideal gas, Z = 1 and it is independent of temperature and pressure. 2


N2
The deviations from ideal behaviour of a real gas will be determined by the H2
value of Z being greater or less than 1. The difference between unity and the CO2

value of the compressibility factor of a gas is a measure of the degree of non Ideal gas
1
ideality of the gas. Z

0
P

For a real gas, the deviations from ideal behaviour depends on (i) pressure; and (ii) temperature.
This will be illustrated by examining the compressibility curves of some gases discussed below
with the variation of pressure and temperature.
[Type text]

Effect of Pressure Variation on Deviations: Figure shows 200K


the compressibility factor, Z, plotted against pressure for H2, N2 and 500K
CO2 at a constant temperature.
1000K
Z
At very low pressure, for all these gases Z is approximately equal to
1.0
one. This indicates that at low pressures (upto 10 atm), real gases
exhibit nearly ideal behaviour. As the pressure is increased, H2 shows a
continuous increase in Z (from Z = 1).Thus the H2 curve lies above the P (atm)
ideal gas curve at all pressure.

For N2 and CO2, Z first decrease (Z < 1). It passes through a minimum and then increases
continuously with pressure (Z > 1). For a gas like CO2 the dip in the curve is greatest as it is
most easily liquified.

Effect of Temperature on Deviations: Figure shows plots of Z or PV/RT against P for


N2 at different temperatures. It is clear from the shape of the curves that the deviations from the
ideal gas behaviour become less and less with increase of temperature. At lower temperature, the
dip in the curve is large and the slope of the curve is negative. That is, Z < 1. As the temperature
is raised, the dip in the curve decreases. At a certain temperature, the minimum in the curve
vanishes and the curve remains horizontal for an appreciable range of pressures. At this
temperature, PV/RT is almost unity and the Boyle’s law is obeyed. Hence this temperature for
the gas is called Boyle temperature. The Boyle temperature of each gas is characteristic e.g., for
N2 it is 332 K.

Conclusion: From the above discussions we conclude that:

1. At low pressure and fairly high temperatures, real gases show nearly ideal behaviour and
the ideal-gas equation is obeyed.

2. At low temperatures and sufficiently high pressures, a real gas deviates significantly from
ideality and the ideal-gas equation is no longer valid.

3. The closer the gas is to the liquefication point, the larger will be the deviation from the
ideal behaviour.

Compressibility factor Z: Real and ideal gases can be compared at various pressures and
various temperatures by noting the extent to which the value of PV/RT deviates from 1. The
PV PV
quantity is given by the symbol Z and the name compressibility factor. That is Z =
RT RT

Greater is the departure of Z from unity, more is the deviation from ideal behaviour.

i) When Z < 1, this implies that gas is more compressible.


ii) When Z > 1, this means that gas is less compressible.
[Type text]

iii) When Z = 1, the gas is ideal.

Vander Waals Equation of State for a Real Gas: The equation of state generated by
Vander Waals in 1873 reproduces the observed behaviour with moderate accuracy. For n moles
of gas, the Vander Waals equation is

 n 2a 
 P +  (V – nb) = nRT
 V2 

where a and b are constants characteristic of a gas. This equation can be derived by considering a
real gas and converting it to an ideal gas.

Volume correction: We know that for an ideal gas P × V = nRT. Now in a real gas the
molecular volume cannot be ignored and therefore let us assume that `b’ is the volume excluded
(out of the volume of container) for the moving gas molecules per mole of a gas. Therefore due
to n moles of a gas the volume excluded would be nb.

∴ a real gas in a container of volume V has only available volume of (V – nb) and this can
be thought of, as an ideal gas in a container of volume (V – nb)

Pressure correction: Let us assume that the real gas exerts a pressure P. The molecules that
exert the force on the container will get attracted by molecules of the immediate layer which are
not assumed to be exerting pressure.

It can be seen that pressure the real gas exerts would be less than the pressure an ideal gas would
have exerted. The real gas experiences attractions by its molecules in the reverse direction.
Therefore if a real gas exerts a pressure P, then an ideal gas would exert a pressure equal to P +
p(p is the pressure lost by the gas molecules due to attractions). This small pressure p would be
directly proportional to the extent of attraction between the molecules which are hitting the
container wall and the molecules which are attracting these.

n
Therefore p ∝ (concentration of molecules which are hitting the container’s wall)
V
n n2
p∝ (concentration of molecules which are attracting these molecules ) ⇒ p ∝ 2
V V
an 2
∴ p = 2 where a is the constant of proportionality which depends on the nature of gas.
V
Higher value of `a’ reflects the increased attraction between gas molecules.

The Vander Waals constant b (the excluded volume) is actually 4 times the volume of a single
molecule. i.e. b = 4 NV where N ? Avogadro number.

4 3
∴ b = 4 × 6.023 × 1023 π r , where r is the radius of a molecule.
3
[Type text]

The constant a and b: Vander Waals constant for attraction (a) and volume (b) are
characteristic for a given gas. Some salient features of ‘a’ and ‘b’ are:

i) For a given gas Vander Waal’s constant of attraction `a’ is always greater than Vander
Waals constant of volume (b).

ii) The gas having higher value of `a’ can be liquified easily and therefore H2 and He are not
liquified easily.

iii) The units of a = litre2 atm mole–2 and that of b = litre mole–1

iv) The numerical values of a and b are in the order of 10–1 to 10–2 to 10–4 respectively.

v) Higher is the value of `a’ for a given gas, easier is the liquification.

Explanation of deviation by Van der Waals equation

i) At lower pressure:`V’ is large and `b’ is negligible in comparison with V. Then


Vander Waals equation reduces to :

 a 
 P + 2  V = RT
 V 
a
PV + = RT
V
a
PV = RT – or PV < RT
V

this accounts for the dip in PV vs P isotherm at low pressure.

a
ii) At fairly high pressures: may be neglected in comparison with P.
V2
The Vander Waals equation becomes
P ( V–b) = RT
PV – Pb = RT
PV = RT + Pb or PV > RT

This accounts for the rising parts of the PV vs P isotherm at high pressures.
a
iii) At very low pressure:V becomes so large that both b and become negligible and
V2
the Vander Waals equation reduces to PV = RT. This shows why gases approach ideal
behaviour at very low pressures.

iv) Hydrogen and Helium:These are two lightest gases known. Their molecules have
very small masses. The attractive forces between such molecules will be extensively
[Type text]

a
small. So is negligible even at ordinary temperatures. Thus PV > RT. In this way
V2
Vander Waals equation explains quantitatively the observed behaviour of real gases and
so is an improvement over the ideal gas equation.

Vander Waals equation accounts for the behaviour of real gases.

At low pressures, the gas equation can be written as,

 a 
P + 2  (Vm ) = RT
 Vm 
a
RT −
PVm Vm  a 
or Z = = Q PV = RT − at low pressure 
RT RT  V 

where Z is known as compressibility factor. Its value at low pressure is less than 1 and it
decreases with increase of P. For a given value of Vm, Z has more value at higher
temperature.

At higher pressure, the gas equation can be written as

P (Vm – b) = RT
PVm RT + Pb Pb
Z= = = 1+ {Q PVm = RT + Pb at high pressure}
RT RT RT

Here, the compressibility factor increases with increase of pressure at constant


temperature and it decreases with increase of temperature at constant pressure. For the
gases H2 and He, the above behaviour is observed even at low pressures, since for these
gases, the value of `a’ is extremely small.

Critical phenomenon & Liquification of gases: The phenomena of converting a gas


into liquid is known as liquifaction. The liquifaction of gas is achieved by controlling P and T as
follows:

1. Increasing pressure: An increase in pressure results in an increase in attraction


among molecules.

2. Decreasing temperature: A decrease in temperature results in decrease in kinetic


energy of molecules.

Critical temperature (Tc): It is defined as the characteristic temperature for a given gas
below which a continuous increase in pressure will bring liquification of gas and above which no
liquefaction is noticed although pressure may be increased e.g. Tc for CO2 is 31.2°C.
8a
Tc =
27 Rb
[Type text]

Critical pressure (Pc): It is defined as the minimum pressure applied on 1 mole of gas
placed at critical temperature, to just liquefy the gas
a
Pc =
27b2

Critical Volume (Vc): The volume occupied by 1 mole of gas placed at critical conditions.

Vc = 3b (i.e. P = Pc and T = Tc)

Gas Eudiometry
The relationship amongst gases, when they react with one another, is governed by two laws,
namely Gay-Lussac’s law and Avogadro’s law.

Gaseous reactions for investigation purposes are studied in a closed graduated tube open at one
end and the other closed end of which is provided with platinum terminals for the passage of
electricity through the mixture of gases. Such a tube is known as Eudiometer tube and hence the
name Eudiometry also used for Gas analysis.

During Gas analysis, the Eudiometer tube filled with mercury is inverted over a trough
containing mercury. A known volume of the gas or gaseous mixture to be studied is next
introduced, which displaces an equivalent amount of mercury. Next a known excess of oxygen is
introduced and the electric spark is passed, whereby the combustible material gets oxidized. The
volumes of carbon dioxide, water vapour or other gaseous products of combustion are next
determined by absorbing them in suitable reagents. For example, the volume of CO2 is
determined by absorption in KOH solution and that of excess of oxygen in an alkaline solution of
pyrogallol. Water vapour produced during the reaction can be determined by noting contraction
in volume caused due to cooling, as by cooling, the steam formed during combustion, forms
liquid (water) which occupies a negligible volume as compared to the volumes of the gases
considered. The excess of oxygen left after the combustion is also determined by their difference
if other gases formed during combustion have already been determined. From the data thus
collected a number of useful conclusions regarding reactions amongst gases can be drawn.
a) Volume-volume relationship amongst Gases or simple Gaseous reaction.
b) Composition of Gaseous mixtures.
c) Molecular formula of Gases.
d) Molecular formula of Gaseous Hydrocarbons.
The vapours reagents used for absorbing different gases are
O 3 → turpentine oil
O 2 
→ alkaline pyrogallol
NO 
→ FeSO 4 solution
CO 2 ,SO 2 
→ alkali solution (NaOH, KOH, Ca (OH)2, HOCH2CH2NH2 etc).
NH 3  → acid solution or CuSO4 solution
Equation for combustion of hydrocarbons
[Type text]

 y y
C x H y +  x +  O 2 
→ xCO 2 + H 2 O
 4 2

General Assumptions: In all problems, it is assumed that the sparking occurs at room
temperature. This implies that water formed would be in liquid state and that nitrogen gas is inert
towards oxidation.

Illustration 9: A gaseous hydrocarbon requires 6 times its own volume of O2 for complete
oxidation and produces 4 times its volume of CO2. What is its formula?

Solution: The balanced equation for combustion


 y y
C x H y +  x +  O 2 
→ xCO 2 + H 2 O
 4 2
 y y
1 volume means  x +  volume ∴ x + = 6 (by equation)
 4 4
or 4x + y = 24 … (1)
Again x = 4 since evolved CO2 is 4 times that of hydrocarbon
∴ 16 + y = 24 or y = 8 ∴ formula of hydrocarbon C4H8.

Answer to Brain Teaser


1. 5 litre
2. 730.8 K
3. S8, 128
4. Ptotal = 0.4926 atm, PN 2 = PO2 = 0.123 atm PHe = 0.246 atm
5. r1 : r2 : : 1.0043 : 1.0000
6. 1.702 kJ
[Type text]

Solved Problems
SUBJECTIVE

Problem 1: Calculate the total pressure in a 10 L cylinder which contains 0.4 g of


helium, 1.6g of oxygen and 1.4 g of nitrogen at 27°C. Also calculate the
partial pressure of helium gas in the cylinder. Assume ideal behaviour for
gases.

Solution: Moles of He = w/M = 0.4/4 = 0.10


Moles of O 2 = w/M = 1.6/32 = 0.05
Moles of N 2 = w/M = 1.4/28 = 0.05
Total moles = 0.2
PV = nRT
P ×10 = 0.2 × 0.082 × 300
P = 0.492 atm

Problem 2: One litre flask contains O2 at 0°C and 7.6 × 10–10 mm Hg. Calculate the
number of molecules of O2 present.

Solution: PV = nRT
7.6 ×10–10
P= atmosphere
760
= 10 –12 atmosphere
V1 = 1 litre n=?
R = 0.082 litre L atm K –1mol –1 , T = 273 K
∴ 10 –12 × 1 = n × 0.082 × 273
∴ n = 4.46 ×10 –12 moles
Molecules = 4.46 × 10 –14 × 6.02 × 10 23 = 2.68 × 1010

Problem 3: Assuming that N2 molecules is spherical and its radius is 2×10–10 meter,
calculate the empty space in one mole of N2 gas at NTP.

4
Solution: Volume of one molecule of N 2 = πr 3
3
4 × 3.14(2 × 10 )
–10 3
= = 3.35 ×10–29 m3 = 3.35 ×10 –23 cm3
3
[Type text]

Volume of one mole N 2 molecules = 6.02 × 10 23 × 3.35 × 10 –23 cm 3


= 20.167 cm3
Volume of one mole N 2 at NTP = 22400 cc.
Hence empty space = 22400 – 22379.833 cm3
Empty volume × 100 22379.833 ×100
% empty space = = = 99.9%
Total Volume 22400

Problem 4: A glass bulb of one litre contains 2 ×1021 molecules of nitrogen exerting
pressure of 7.57 ×103 Nm–2. Calculate the root mean square velocity and
temperature of the gas molecules. The ratio of U mpv to Urms is 0.82.
Calculate also Umpv of gas molecules at this temperature.

Solution: We will calculate first the temperature of the gas from the given data.
PV = nRT
All the values are to be taken in MKS system.
P = 7.57 ×103 Nm –2
V = 1litre = 1×10 –3 meter 3 ,
2 ×1021
n= moles, R = 8.31 JK –1mol –1
6.02 ×1023
2 ×1021 × 8.31× T
∴ 7.57 ×103 ×10 –3 =
6.02 ×1023
T = 274.2 K
3RT
Now, U rms =
M
All the values are to be taken in MKS system,
R = 8.31, M = 28 × 10 –3 kg for N2
T = 274.2 K
3 × 8.31× 274.2
∴ U rms =
28 ×10 –3
= 494.2 meter/sec.
Again , U mp = 0.82 × U rms
= 494.2 × 0.82
= 405.26 meter/sec.
[Type text]

Problem 5: Using Vander Waals equation calculate the pressure exerted by one mole of
CO2. Its volume at 373 K is 0.05 dm3. Given a = 3.592 atm L2mol–2 and
b = 0.0426 L mol–1.

 an 2 
Solution:  P +  (V – nb) = nRT
 V2 
P = ?, n = 1, V = 0.05 litre,
a = 3.592, b = 0.04267, T = 373 K
R = 0.0821 L atm K –1 mol –1 ,
 3.592 
P +  (0.05 – 0.04267) = 1× 0.0821× 373 ⇒ P = 1620.95 atm
 (0.05)2 

Problem 6: Density of a mixture of CO and CO2 at 303 K and 73 cm of Hg is 1.5


gram/litre. What is the mole percent of two gases in the mixture.

wRT
Solution: PV =
M
wRT
or P =
M×V
dRT
or P =
M
73
P = atm, d = 1.5 g/litre, M = ?
76
T = 303 K, R = 0.0821 L atm K –1 mol –1
73 1.5 × 0.0821× 303
∴ =
76 M
M = 38.85 (molecular mass of the mixture)
Suppose, Total moles = 100
Moles of CO = a, Moles of CO 2 = 100 – a
Mass of CO2 = (100 – a)44
Total mass = 28a + (100 – a) 44
28a + (100 – a)44
Molecular mass of mixture =
100
But molecular mass of mixture is 38.85
28a + (100 – a)44
38.85 =
100
a = 32.19
Mole percent of CO = 31.19
Mole percent of CO2 = 67.81
[Type text]

Problem 7: Two grams of gas A are introduced in a evacuated flask at 25°C. The
pressure of the gas is 1 atm. Now 3g of another gas B is introduced in the
same flask, the total pressure becomes 1.5 atm. Calculate (a) the ratio of
molecular mass M A and M B (b) volume of the vessel, if A is O2.

Solution: For gas A : Its mass = 2g; PA = 1 atm , T = 298 K


For gas B : Its mass = 3g, PB = 0.5 atm. T = 298 K
According to Dalton’s law of partial pressure
P = p A + pB
1.5 = 1 + p B
∴ pB = 0.5 atm
wRT
Now, PV =
m
For gas A : p A = 1 atm, m = M A , w = 2g
2 × RT
1× V = … (1)
MA
for gas B : p B = 0.5 atm, m = M B , w = 3g
3 × RT
0.5 × V = … (2)
MB
Dividing the equation (1) and (2)
3× MA
0.5 =
2 MB
MA 1
or =
MB 3
(b) If A is O2 then M A = 32, T = 298 K, V = ?
Put these values in equation (1)
2 × 0.0821× 298
V= = 1.53 litres
32

Problem 8: One litre O2 and one litre H2 are taken in a vessel of 2 litre at STP. These
gases are made to react to form water. Calculate (a) moles and weight of
water formed. (b) amount of gas left in the vessel. (c) Total pressure of the
gas at 100°C.

Solution: 2H 2 + O 2 = 2H 2O
Volume before reaction (L) 1 1 0
Volume after reaction (L) 0 0.5 1
a) For H 2O : PV = nRT
At STP P = 1 atm, V = 1 litre,
[Type text]

∴ Moles of H 2 O(n) = 4.46 × 10 –2


Weight of H 2 O = 4.46 × 10 –2 × 18 = 0.803 gram

b) Gas left in the vessel = 0.5 litre at STP


∴ 22.4 litre O 2 at STP weigh 32 g
32 × 0.5
∴ 0.5 litre O2 at STP weigh 32g = = 0.714 gram
22.4
c) Moles of H 2O formed = 4.46 × 10–2
O2 left = 0.5 litre at STP
= 2.23 ×10 –2 moles
Total moles in the vessel = 4.46 ×10 –2 + 2.23 ×10–2 = 6.69 ×10 –2
PV = nRT
P = ? V = 2 litre,
n = 6.69 × 10 –2 , T = 373 K
P × 2 = 6.69 × 10 –2 × 0.0821 × 373
∴ P = 1.02 atm

Problem 9: At 27°C vapour density of the mixture of NO2 and N2O4 is 38.3. Calculate
the moles of NO2 in 100 g mixture.

Solution: Vapour density of mixture = 38.3


Molecular mass of mixture = 2 × 38.3
Mass of mixture = 100 g
100
Number of moles present = … (1)
2 × 38.3
Suppose mass of NO 2 = a gram
Mass of N 2O4 = (100 – a)gram
Moles of NO2 = w / M = a / 46
Moles of N 2 O4 = w / M = (100 – a) / 92
a 100 – a
Total moles + … (2)
46 92
Equating equation (1) and (2)
100 a 100 – a
= +
2 × 38.3 46 92
∴ Moles of NO 2 = w / M = 20.1/ 46 = 0.43

Problem 10: A given volume of O2 gas diffuses in 224 seconds. Under identical
conditions same volume of a mixture of O2 and another unknown gas
diffuses in 234 seconds. If the mixture contains 80% O2, calculate the
molecular mass of the unknown gas.
[Type text]

Solution: Mixture contains O2 = 80%


Unknown gas = 20%
Suppose,
Molecular mass of unknown gas = m
According to Graham’s law
rO2 Mg
=
rg M O2
rO2 = V / 224 , M O2 = 32
80 × 32 + 20m 234 80 × 32 + 20m
rg = V / 234, M g = ∴ =
100 224 100 × 32
m = 46.6 (molecular mass of unknown gas)
[Type text]

KVPY
1. The plot that is not valid for an ideal gas where P is the pressure and V is the volume of the gas, is –
(2007)

PV V P
(A) (B) (C) (D)
1/V
1/P V

o
2. One mole each of the two gases X and Y are stored separately in two cylinders at 25 C at pressure
1 atm. and 2 atm, respectively. The difference in the compressibilities of the two gases. ( k x − k y ) is
(2008)
(A) 0.1 atm−1 (B) 0.5 atm−1 (C) 1.0 atm−1 (D) 2.0 atm−1

3. An ideal gas is subjected to a cyclic change as shown in the P-V diagram below : (2008)
A

V
The step in which the gas will cool down is along
(A) AB (B) BC (C) Both AB and CA (D) Both BC
and CA

4. Consider two sealed jars of equal volume. One contains 2 g of hydrogen at 200 K and the other
contains
28 g of nitrogen at 400 K. The gases in the two jars will have (2010)
(A) The same pressure (B) The same average kinetic energy
(C) The same number of molecules (D) The same average molecular speed

5. Two balloons A and B containing 0.2 mole and 0.1 mole of helium of room temperature and 2.0 atm,
respectively, are connected. When equilibrium is established, the final pressure of He in the system
is (2011)
(A) 1.0 atm (B) 1.5 atm (C) 0.5 atm (D) 2.0 atm

6. At 298 K, assuming ideal behaviour, the average kinetic energy of a deuterium molecule is (2011)
(A) two times that of a hydrogen molecule (B) four times that of a hydrogen
molecule
(C) half of that of a hydrogen molecule (D) same as that of hydrogen molecule

7. As isolated box, equally partitioned contains two ideal gases A and B as shown (2011)

A B
1 atm, 25°C 1 atm, 25°C

When the partition is removed, the gases mix. The changes in enthalpy ( ∆H ) and entropy ( ∆S) in
the process, respectively, are
(A) zero, positive (B) zero, negative (C) positive zero (D) negative,
zero
[Type text]

8. A specific volume of H2 requires 24 s to diffuse out of a container. The time required by an equal
volume of O2 to diffuse out under identical conditions, is (2013)
(A) 24 s (B) 96 S (C) 384 S (D) 192 s

9. At room temperature the average seed of Helium is higher than that of Oxygen by a
factor of (2014)
(A) 2 (B) 6/ (C) 8 (D) 6

Answer Key
1:D 2:B 3:B 4:C
5:D 6:D 7:A 8:B
9;A
[Type text]

Assignments
SECTION – I Single Choice Questions

1. A large cylinder of helium filled at 2000 mm of Hg had a small orifice through which
helium escaped into evacuated space at the rate of 6.4 moles/hour. How long would it
take for 10 mole of CO to leak through a similar orifice if the CO were confined at the
same pressure?
(a) 2.1 hour (b) 4.2 hour
(c) 5.6 hour (d) 11.2 hour.

2. When 150 ml of ozonized oxygen was passed through red hot tube, the volume increased
by 10 ml, then the volume percentage of ozone in the sample is
(a) 25% (b) 51%
(c) 13.33% (d) 17.84%.

3. A real gas at a very high pressure occupies


(a) more volume that that of an ideal gas under identical conditions
(b) less volume than that of an ideal gas under identical conditions
(c) same volume that that of an ideal gas under identical conditions
(d) can’t predict.

4. A flask filled with CCl4 vapour was weighed at a temperature and pressure. The flask was
then filled with oxygen at the same temperature and pressure. The mass of CCl4 vapour
would be about
(a) the same as that of the oxygen (b) one-fifth as heavy as oxygen
(c) five times as heavy as oxygen (d) twice as heavy as oxygen.

5. Hydrogen gas diffuses four times as rapidly as a mixture of C2H4 and CO2. The molar
ratio of C2H4 to CO2 in the mixture is
(a) 1 : 1 (b) 2 : 1
(c) 3 : 1 (d) 3 : 2.

6. At 100oC and 1 atm, if the density of liquid water is 1.0 g/cc and that of water vapour is
0.0006 g/cc, then the volume occupied by water molecule in one litre of steam at that
temperature is
(a) 6 cc (b) 60 cc
(c) 0.6 cc (d) 0.06 cc

7. The K.E. of N molecule of O2 is x Joules at –123°C. Another sample of O2 at 27°C has a


KE of 2x Joules. The latter sample contains
(a) N molecules of O2 (b) 2N molecules of O2
[Type text]

(c) N/2 molecules of O2 (d) N/4 molecule of O2

8. Helium atom is two times heavier than a hydrogen molecule. At 298 K, the average
kinetic energy of a Helium atom is
(a) two times that of hydrogen molecule (b) same as that of a hydrogen molecule
(c) four times that of a hydrogen molecule (d) half that of a hydrogen molecule
9. The temperature of an ideal gas is increased from 140 K to 560 K. If at 140 K the root
mean square velocity of the gas molecule is V, at 560 K it becomes
(a) 5V (b) 2V
(c) V/2 (d) V/4
10. The value of Van der Waal’s constant `a’ for the gases O2, N2, NH3 and CH4 are 1.360, 1.390,
2 –2
4.170 and 2.253 litres atm. mole . The gas which can most easily be liquiefied is
(a) O2 (b) N2
(c) NH3 (d) CH4.

11. 1.0 litre of N2 and 7/8 litre of O2 at the same temperature and pressure were mixed
together. What is the relation between the masses of the two gases in the mixture?
(a) M N 2 = 3M O 2 (b) M N 2 = 8M O 2
(c) M N 2 = M O 2 (d) M N 2 = 16M O2

12. The density of a gas A is twice that of a gas B at the same temperature. The molecular
weight of gas B is thrice that of A. The ratio of the pressures acting on A and B will be
(a) 1:6 (b) 7:8
(c) 2:5 (d) 1:4
13. Two vessels A and B have volumes V and 4V respectively. Both vessels contain some
water. The pressure in the space above water is p1 for vessel A and p2 for vessel B. What
will be the relation between p1 and p2?
(a) p1 = 4p2 (b) 4p1= p2
(c) p1 = p2 (d) p1 = 16p2

14. Let the most probable velocity of hydrogen molecules at a temperature t°C is V0. Suppose all
the molecules dissociate into atoms when temperature is raised to (2t + 273)°C then the
new r.m.s. velocity is

(a) 2 / 3V0 (b) 3(2 + 273 / t)V0

(c) 2 3V0 (d) 6 V0 .

15. The rms velocity of hydrogen is 7 times the rms velocity of nitrogen. If T is the
temperature of the gas then
(a) T (H2) = T(N2) (b) T(H2) > T(N2)
(c) T(H2) > T(N2) (d) T(H2) = 7T(N 2 )

16. Equal weights of CH4 and H2 are mixed together in an empty container at 25°C. The
[Type text]

fraction of the total pressure exerted by H2 is


(a) ½ (b) 8/9
(c) 1/9 (d) 16/17

17. The volumes of two vessels are in the ratio of 2 : 3 one vessel contains H2 and other N2 at
600 mm and 900 mm respectively. The final pressure when they are conncected together
is;
(a) 650 mm (b) 760 mm
(c) 780 mm (d) 800 mm
18. Under same conditions of temperature and pressure a hydrocarbon of molecular formula
CnH2n–2 was found to diffuse 3 3 times slower than hydrogen. Find the value of n?
(a) n = 2 (b) n = 4
(c) n = 3 (d) n = 1

19. A container contains certain gas of mass “m” at a high pressure. A little amount of the
gas has been allowed to escape from the container and after sometime the pressure of the
gas becomes half and its absolute temperature 2/3rd. The amount of the gas escaped is;
m
(a) 2m/3 (b)
2
m m
(c) (d)
4 6
20. A vessel contains 1 lole of O2 at 27°C and 1 atm pressure. A certain amount of the gas
was withdrawn and the vessel was heated to 327°C to maintain the pressure of 1 atm. The
amount of gas removed was;
(a) 0.2 mole (b) 0.5 mole
(c) 0.25 mole (d) 0.1 mole
21. The ratio of velocities of diffusion of two gas A and B is 1 : 4. If the ratio of their masses
in the mixture is 2 : 3, the ratio of their mole fraction is;
1 1
(a) (b)
10 12
1 1
(c) (d)
24 15
22. Two mole each of hydrogen and oxygen are filled in a balloon and a small hole is made
after one hour 0.4 mole of oxygen effused out. The mole fraction of hydrogen in the
balloon is
(a) 0.1 (b) 0.2
(c) 0.4 (d) 0.5
23. A mixture of hydrogen and oxygen is 3 : 1 volume ratio is allowed to diffuse through a
process partition. What should be the composition of the initial gas diffusing out of the
vessel?
(a) 1 : 12 (b) 12 : 1
(c) 6 : 1 (d) 4 : 1
24. An unspecified quantity of an ideal gas has a volume of 30 litres at 20°C until the
[Type text]

pressure has doubled and then, the temperature is raised to 100°C, while the pressure is
kept constant what is the final volume of the gas?
(a) 25 lit (b) 30 lit
(c) 19 lit (d) 40 lit

25. There are 201 equidistant rows of spectators sitting in a hall. A magician releases
laughing gas N2O from the front and the tear gas (mol. Wt. = 176) from the rear of the
hall simultaneously which row spectators will have a tendency to smile and weep
simultaneously?
(a) 130 (b) 134
(c) 120 (d) 100
SECTION – II May be more than one choice

1. Liquified natural gas (LNG) is mainly methane. A 10 m 3 tank is constructed to store


LNG at –164°C and 1 atm pressure, under this condition density of LNG is 415 kg/ m 3 .
The volume of storage tank capable of holding … mass of LNG as a gas at 20°C and 1
atm pressure will be
(a) 1250 m 3 (b) 5280 m 3
(c) 6230 m 3 (d) 9870 m 3 .

2. The rate of effusion of two gases `a’ and `b’ under identical conditions of temperature
and pressure are in the ratio of 2 : 1. What is the ratio of rms velocity of their molecules if
Ta and Tb are in the ratio of 2 : 1?
(a) 2 : 1 (b) 2 :1
(c) 2 2 :1 (d) 1: 2 .

3. One litre of an ideal gas diffuses in 10 minutes at STP. How much time it would take for
the same diffusion at 1 atm and 273°C?
(a) 7.1 min (b) 10 min
(c) 14.1 min (d) can’t say.

4. When 100 ml sample of methane and ethane along with excess of O 2 is subjected to
electric spark, the contraction in volume was observed to be 212 ml. When the resulting
gases were passed through KOH solution, further contraction in volume was
(a) 60 ml (b) 96 ml
(c) 108 ml (d) 124 ml.

5. A 50 ml sample of a hydrogen-oxygen mixture was placed in a gas burette at 18°C and


confined at 1 atm. A spark was passed through the sample so that the formation of water
could go to completion. The resulting pure gas had a volume of 10 ml at atmospheric
pressure. The initial mole fraction of hydrogen in the mixture would be
(a) 0.734 (b) 0.534.
(c) 0.734 or 0.534 (d) can’t be predicted.

6. For two gases A and B with molecular weights MA and MB, it is observed that at a certain
[Type text]

temperature T1 the mean velocity of A is equal to the root mean square velocity of B, thus
the mean velocity of A can be made equal to the mean velocity of B if
(a) A is at temperature T and B at T?, T > T?
(b) A is lowered to a temperature T2 , T2< T while B is at T
(c) Both A and B are raised to a higher temperature
(d) Both A and B are placed at lower temperature

7. The circulation of blood in human body supplies O2 and releases CO2. The concentration
of O2 and CO2 is variable but on an average, 100 ml blood contains 0.02 g of O2 and 0.08
g of CO2. The volume of O2 and CO2 at 1 atm and at body temperature 37oC, assuming
10 litre blood in human body, is
(a) 2 lt, 4 lt (b) 1.5 lt, 4.5 lt
(c) 1.59 lt, 4.62 lt (d) 3.82 lt, 4.62 lt
8. Which of the following statements is not true?
(a) The ratio of the mean speed to the rms speed is independent
of the temperature.
(b) The square of the mean speed of the molecules is equal to the square of rms speed at
a certain temperature.
(c) Mean kinetic energy of the gas molecules at any given temperature is independent of
the mean speed.
(d) The difference between rms speed and mean speed at any temperature for different
gases diminishes as larger and yet larger molar masses are considered.

PV
9. Consider the equation Z = . Which of the following statements is correct?
RT
(a) When Z ? 1, real gases are easier to compress than the ideal gas.
(b) When Z = 1, real gases get compressed easily.
(c) When Z ? 1, real gases are difficult to compress.
(d) When Z = 1, real gases are difficult to compress.

10. An open vessel at 27C is heated until 3/8th of the air in it has been expelled. Assuming
that the volume remains constant, calculate the temperature at which the vessel was
heated.
(a) 307°C (b) 107°C
(c) 480°C (d) 207°C

11. The vapour pressure of water at 80°C is 355 mm of Hg. A 100 mL vessel contains water
saturated with O2 at 80°C, the total pressure being 760 mm of Hg. The contents of the
vessel were pumped into a 50 mL vessel at the same temperature. What is the partial
pressure of O2?
(a) 1115 mm (b) 810mm
[Type text]

(c) 405mm (d) 355mm

12. 40.0 mL of a gaseous mixture of CO and C2H2 is mixed with 100.0 mL of O2 and burnt.
The volume of the gas after the combustion is 10.5 mL. Calculate the composition of the
original mixture
(a) 25 mL of CO and 15 mL of C2H2 (b) 15 mL of CO and 25 mL of C2H2
(c) 10 mL of CO and 30 mL of C2H2 (d) 20 mL of CO and20 mL of C2H2

13. 80 mL of oxygen is added to 50 mL of a mixture of H2, C2H2 and CO, after which the
total mixture is burnt. The volume of the cooled mixture after combustion measures 65
mL. This is reduced to 15 mL by treatment with a KOH solution. Calculate the volume of
each gas in the original mixture.
(a) 20 mL of H2, 20 mL of C2H2, 10 mL of CO
(b) 10 mL of H2, 20 mL of C2H2, 20 mL of CO
(c) 15 mL of H2, 15 mL of C2H2, 20 mL of CO
(d) 20 mL of H2, 25 mL of C2H2, 5 mL of CO
14. I, II, III are three isotherms respectively at T1, T2 and T3.
Temperature will be in order:
(a) T1 = T2 = T3
P
(b) T1< T2< T3 I
(c) T1> T2> T3 II
(d) T1> T2 = T3 III
O V

15. An under water bubble with a radius of 0.5 cm at the bottom of a tank, whose the temperature
is 5°C and the pressure is 3 atm, rises to the surface where the temperature is 25°C and
pressure is 1 atm. What will be the radius of the bubble when it reaches the surface?

(a) 0.53 cm (b) 0.73 cm

(c) 0.40 cm (d) 5.3 cm

16. A vessel of volume 100 ml contains 10% of oxygen and 90% of an unknown gas. The gases
diffuse in 86 seconds through a small hole of the vessel. If pure oxygen under the same
conditions diffuses in 75 seconds. What is the molecular mass of the unknown gas?

(a) 42 (b) 44

(c) 40 (d) 48
[Type text]

17. There are 13 equidistant rows of benches of students in a class-room. A teacher releases N2O
from the front of the class-room, while a student releases tear gas C6H11OBr (mol. Wt. =
176) from the back of the class-room simultaneously. Students of which row will laugh and
weep simultaneously.

th th
(a) 9 (b) 6

(c) 5th (d) 4th

18. An unspecified quantity of an ideal gas has an initial pressure of 5 atm and temperature is
30°C. The gas is expanded at 30°C until the volume has increased by 60% of the initial
value. Next, the quantity of the gas in the vessel is increased by 20% of the initial value
while the volume is maintained constant. Finally, the temperature is adjusted at constant
volume until the gas pressure is again 5 atm, what is the final temperature?

(a) 400 K (b) 405 K

(c) 100 K (d) 50 K

19. The pressure exerted by 12 gm of an ideal gas at t°C in a vessel of volume v litre is one atom.
When the temperature is increased by 10°C at the same volume the pressure increased by
10%. Calculate the temperature t and volume v? (Mol. Wt. = 120°)

(a) 100°C, 0.82 lit (b) 200°C, 80 lit

(c) 40°C, 80 lit (d) 400°C, 100 lit

20. Two containers A and B contain the same gas. If the pressure volume and temperature of the
gas in A are two times as compared to that of B and if the mass of the gas B is x gm, the
mass of the gas in A will be;

(a) 4 x gm (b) x/2 gm

(c) 2x gm (d) x gm

–1
21. The average speed of an ideal gas molecule at 27°C is 0.3 ms . Calculate average speed at
927°?
[Type text]

–1 –1
(a) 0.6 ms (b) 6 ms

–1 –1
(c) 60 ms (d) 8 ms

22. An evacuated glass vessel weighs 50 gm when empty 148.0 gm filled with a liquid of density
–1
0.98 gm ml and 50.5 gm when filled with an ideal gas at 760 mm of Hg at 300 K. What is
the mol. Wt. of the gas?

(a) 100 (b) 110

(c) 122 (d) 90

23. The factor which measure the deviation from ideal behavior of a gas are
(a) collision frequency (b) collision diameter
(c) compressibility factor (d) vander Waal constant ‘a’

24. Which of the following expression is correct for an adiabatic process ?


γ −1 γ −1/ r
T  V  P T 
(a)  2  =  1  (b) 2 =  1 
 T1   V2  P1  T2 
(c) P2 V2γ = P1V1γ (d) P1V1γ−1 = P2 V2γ−1

25. The correct statement(s regarding graphite allotrope of carbon is(are):


(a) It consists of planar hexagonal layers of carbon atoms held together by relative week
van der Waal’s forces.
(b) It is a good conductor of electricity in directions along the plane of carbon atoms.
(c) It is thermodynamically less stable isotrope than the diamond
(d) Graphite is anisotropic

SECTION – III Comprehension Type Questions

? Write-up I

T1
No. of molecules

T2
T3

Molecular speed

Graph shows typical Maxwell speed distribution curves for N2 gas at three different
temperatures T1, T2 and T3. The peak of each curve represents the most probable speed.
On increasing temperature, curve begins to flatten out and shifts toward right. But at
[Type text]

constant temperature T, same nature of graph is obtained for three gases A, B and C. In
an apparatus a beam of atoms (or molecules) exits from an oven at a known temperature
and passes through a pinhole. Two circular plates mounted on the same shaft is rotated
by a motor. The first plate is called chopper which allows small bursts of atoms and
second plate is detector. Eventually a layer of deposit will accumulate on detector. The
speed of the deposit on the detector gives the range of molecular speed.

1. During an experiment in two different cases, the density of the deposit accumulated in
case II ? Case I. If speed of the motor remains constant, then which of the following is
correct?
(a) Molecular velocity of beam of atoms in case I ? case II
(b) Molecular velocity of beam of atoms in case I ? case II
(c) Molecular velocity of beam of atoms in case I = case II
(d) None of the above

2. A student measures the fraction of molecules by the apparatus in which chopper bursts
the molecules. Chopper has square and circular faces in two cases respectively. Ratio of
fraction of molecules obtained will be:
1
(a) (b) π
π
1
(c) 2? (d)

3. Root-mean square speed will be minimum for the gas


(a) A (b) B
(c) C (d) All of the above are equal

? Write-up II

Compressibility factor (Z): Real gases deviate from ideal behavior due to the following two
faulty assumptions of kinetic theory of gases.
i) Actual volume occupied by the gas molecule is negligible as compared to the total
volume of the gases.
ii) Forces of attraction and repulsion among the gas molecules are negligible.
the, extent of deviation of the real gas from ideal behaviour, is explained in terms of
compressibility factor (Z), which is function of pressure and temperature for real gas.
For ideal gas, Z = 1
For real gases, Z ?? 1 or Z ? 1
When Z ? 1, then it is less compressible because force of repulsion dominates over force
of attraction when Z ? 1, force of attraction dominates over the force repulsion.
Graph in between Z & P is shown as under
[Type text]

at 0o C H2
He N2
CH 4 CO 2

Z =1 ideal gas

P
On increasing temperature, Z increases and approaches to unity. Graph between
Z and P at different temperature for the same gas is shown as under:
T4

T3
T2
T1

Z =1 Ideal gas

P
The van der Waal’s equation of state for 1 mole of gas is as under:
 a 
 P + 2  (V − b) = RT …(1)
 V 
Where a and b are van der Waal’s constants.
van der Waal’s constant “a” measures the amount of the force of attraction among the
gas molecules. Higher the value of “a”, higher will be the ease of liquefaction.
Case (1) For H2 and He then equation into –I will reduce P(V – b) = RT Case (2) When
pressure is too low i.e. for N2 or CH4 or, CO2 then equation (–I) reduces into
 a 
 P + 2  V = RT
 V 

4. What is the correct increasing order of “a” of the gases shown as in the above graph?
(a) H2? He ? N2? CO2 (b) CO2? N2? He ? H2
(c) N2? CO2 ? H2? He (d) N2? H2? He ? CO2

5. Which of the following is the correct increasing order of temperature? For given gas as
shown in the above graph in between Z vs P?
(a) T1 ? T2? T3? T4 (b) T2? T1? T3? T4
(c) T3? T4? T1? T2 (d) T4? T3? T2? T1

6. Which of the following statement is correct as shown in the above graph?


b
(a) The slope of Z vs P at constant temperature for all real gases, is
RT
b
(b) The slope of Z vs P at constant temperature for both He and H2 is .
RT
b
(c) The slope of Z vs P at low pressure for all real gases, at constant temperature is .
RT
[Type text]

(d) The slope of Z vs P at high pressure and at constant temperature for real gases is
b
− .
RT

7. Which of the following statements is correct for a gas X having molar mass 5g and
density 0.3g/litre at 0.5 atmospheric pressure at 300K.
(a) The gas “X” will behave ideally
(b) The force of attraction will dominate over the force of repulsion among the gas
molecules.
(c) The force of repulsion will dominate over the force of attraction among the gas
molecules
(d) None

Answers to Assignments
SECTION - I

1. (b) 2. (c) 3. (a) 4. (c)


5. (c) 6. (c) 7. (a) 8. (d)
9. (b) 10. (c) 11. (c) 12. (a)
13. (a) 14. (d) 15. (c) 16. (b)
17. (c) 18. (b) 19. (c) 20. (b)
21. (c) 22. (b) 23. (b) 24. (c)
25. (b)

SECTION - II

1. (c) 2. (c) 3. (a) 4. (d)


5. (c) 6. (b) 7. (c) 8. (b)
9. (c) 10. (d) 11. (b) 12. (a)
13. (a) 14. (c) 15. (b) 16. (a)

17. (a) 18. (b) 19. (a) 20. (c)

21. (a) 22. (c) 23. (c), (d) 24. (a), (c)
25. (a), (c)

SECTION - III

1. (b) 2. (a) 3. (a) 4. (a)


[Type text]

5. (d) 6. (b) 7. (b)

S-ar putea să vă placă și